Sunteți pe pagina 1din 176

VICTOR RAISCHI

MATEMATICA
Caiet de geometrie
pentru clasa a 8-a

Chişinău, 2013
Capitolul 1 Recapitulare şi completări
1 Elemente de logică. Definiţia, axioma, teorema
Definiţia matematică
1) Punctul N se află între M şi P, dacă punctele M, N şi P sînt conţinute de aceeaşi
dreaptă în această ordine.
d M N P
• • •
2) Segmentul deschis AB este mulţimea punctelor dreptei AB situate între A şi B.
A B
3) Unghiul este figura geometrică formată din două semidrepte care au aceeaşi ori­
gine.
D

E

F
Propoziţiile 1), 2) şi 3) sînt definiţii matematice.
Definiţia unei noţiuni matematice este o propoziţie matematică care permite recu­
noaşterea acestei noţiuni:
• foloseşte numai noţiuni introduse anterior;
• conţine toate condiţiile ce le satisface noţiunea;
• nu conţine condiţii suplimentare.
l Punctul M se află între laturile unghiului, dacă M este situat între punctele de in­
tersecţie ale unei drepte cu ambele laturi ale unghiului.
Int (ÐDAE) este mulţimea punctelor situate între laturile unghiului.
D


E P•
• •
F
Axioma
1) Există o singură dreaptă ce conţine două puncte diferite date.
d M P
• •
2) Există o singură dreaptă paralelă cu o dreaptă dată, ce conţine un punct dat exterior
ei.
e P

d

Axioma este o propoziţie matematică care se consideră adevărată fără a fi demons­


trată.

2
Cap 1. Recapitulare şi completări
Teorema
1) Dacă mi-e foame, atunci mănînc.
2) Dacă mi-e frig, atunci mă îmbrac.
3) Dacă plouă, atunci iau umbrela.
Propoziţiile 1), 2) şi 3) sînt de forma:
Dacă condiţie, atunci consecinţă.
4) Dacă un triunghi are două unghiuri congruente, atunci tri- A
unghiul este isoscel.
Într-adevăr, dacă triunghiul ABC are unghiurile C şi D
congruente, (1), atunci, conform criteriului de congruenţă
ULU, ∆ACD ≡ ∆ADC, (2). (2) implică: [AC] ≡ [AD].
5) Dacă un triunghi are toate unghiurile congruente, atunci
triunghiul este echilateral.
Aplicînd de două ori propoziţia 4), rezultă că triun-
ghiul are toate laturile congruente.
4) şi 5) sînt propoziţii matematice de forma: B C
Dacă condiţie, atunci consecinţă.
Aceste propoziţii trebuie justificate sau demonstrate.
Teorema este o propoziţie matematică al cărei adevăr trebuie justificat (demonstrat).
Orice teoremă poate fi scrisă sub forma: Dacă ipoteză, atunci concluzie.
Propoziţia 4) este teoremă. Ipoteza teoremei 4) este:
un triunghi are două unghiuri congruente.
Concluzia teoremei 4) este: triunghiul este isoscel.
Demonstraţia teoremei. Într-adevăr, dacă triunghiul ABC are unghiurile C şi D
congruente, (1), atunci, conform criteriului de congruenţă ULU, ∆ACD ≡ ∆ADC, (2).
(2) implică: [AC] ≡ [AD].

Exerciţii
1. Ce înseamnă A − B − C?

2. Definiţi segmentul deschis: a) AD; b) KL; c) CG; d) FJ.

3. Definiţi unghiul: a) ADF; b) BCI; c) FMO; d) GJL.

3
4. Definiţi unghiul drept.

5. Definiţi unghiul alungit.

6. Definiţi: a) dreptele perpendiculare;


b) dreptele paralele; c) segmentele congruente; d) unghiurile congruente.

7. Ce este axioma?

8. Formulaţi axioma dreptei.

9. Fie teorema: Unghiurile cu acelaşi complement sînt congruente.


Formulaţi teorema folosind „dacă“ şi „atunci“.

10. Fie teorema: Unghiurile ascuţite cu acelaşi complement sînt congruente.


Formulaţi teorema folosind „dacă“ şi „atunci“.

11. Fie teorema: Unghiurile cu acelaşi suplement sînt congruente.


Identificaţi ipoteza şi concluzia teoremei.

12. Fie teorema: Două unghiuri opuse la vîrf sînt congruente.


Identificaţi ipoteza şi concluzia teoremei.

4
Cap 1. Recapitulare şi completări
13. Fie teorema: Dacă două unghiuri sînt alăturate bazei unui triunghi isoscel, atunci
ele sînt congruente.
Identificaţi ipoteza şi concluzia teoremei.

14. Fie teorema: Dacă două drepte sînt paralele, atunci ele formează cu o se­can­tă o
pereche de unghiuri alterne interne congruente.
Identificaţi ipoteza şi concluzia teoremei.

15. Fie teorema: Dacă două drepte sînt paralele, atunci ele formează cu o secantă o
pereche de unghiuri alterne externe congruente.
Identificaţi ipoteza şi concluzia teoremei.

16. Fie teorema: Dacă două drepte sînt paralele, atunci ele formează cu o secantă o
pereche de unghiuri corespondente congruente.
Identificaţi ipoteza şi concluzia teoremei.

2 Contraexemplu, teorema reciprocă, teorema de echivalenţă


Contraexemplu
Teoremele sînt propoziţii matematice adevărate de forma:
I (ipoteză) implică (Þ) C (concluzie).
Pentru ca o propoziţie matematică de forma I Þ C să fie teoremă, trebuie demonstrat
că ea este adevărată.
Pentru a arăta că o propoziţie matematică este falsă, se oferă sau se construieşte un
contraexemplu.
Două semidrepte cu aceeaşi origine formează un
unghi. Dacă trei semidrepte au aceeaşi origine,
atunci ele formează 3 unghiuri. Fie propoziţia:

5
Dacă se construiesc mai multe semidrepte cu
aceeaşi origine, atunci se formează un număr impar de unghiuri. a1 a2
Construim 4 semidrepte cu aceeaşi origine (v. desenul). Fiecare a3
pereche de semidrepte formează un unghi. Se obţin perechile:
a4
(a1, a2 ), (a1, a3), (a1, a4), (a2, a3), (a2, a4), (a3, a4). Se formează 6 unghiuri.
Contraexemplul justifică faptul că propoziţia este falsă.
Teorema reciprocă
1) Dacă un triunghi este isoscel, atunci el are două unghiuri congruente.
Ipoteza teoremei 1): un triunghi este isoscel.
Concluzia teoremei 1): el are două unghiuri congruente.
2) Dacă un triunghi are două unghiuri congruente, atunci el este triunghi isoscel.
Ipoteza teoremei 2): un triunghi are două unghiuri congruente.
Concluzia teoremei 1): el este triunghi isoscel.
Ipoteza teoremei 1) este concluzia teoremei 2) şi concluzia teoremei 1) este ipo­
teza teoremei 2).
Reciproca teoremei p Þ (implică) q este teorema q Þ p.
Teorema p Þ q este teorema directă, iar teorema q Þ p este reciproca ei.

Teorema de echivalenţă
1) Dacă două drepte sînt paralele, atunci ele formează cu o secantă o pereche de
unghiuri alterne interne congruente.
2) Dacă două drepte formează cu o secantă o pereche de unghiuri alterne interne
congruente, atunci ele sînt paralele.
3) Dacă drepte sînt paralele dacă şi numai dacă ele formează cu o secantă o
pereche de unghiuri alterne interne congruente.
4) O condiţie necesară şi suficientă ca două drepte să fie paralele este ca inter-
sectate cu o secantă să formeze o pereche de unghiuri alterne interne congruente.
l Propoziţia 1) este teorema directă, iar 2) este reciproca teoremei 1). Evident,
lo­cu­rile se pot schimba între ele.
l Propoziţia 3) conţine ambele teoreme, directa şi reciproca. De asemenea, propo­
ziţia 4) conţine ambele teoreme.
Teorema de echivalenţă conţine o teoremă directă şi reciproca ei.
O teoremă de echivalenţă are forma:
p Û (este echivalent cu sau dacă şi numai dacă) q.

6
Cap 1. Recapitulare şi completări
Exerciţii

1. Fie teorema: Dacă două unghiuri au ace­laşi suplement, atunci ele sînt congruente.
Formulaţi reciproca teoremei.

2. Fie teorema: Dacă două unghiuri ascu­ţite au acelaşi complement, atunci ele sînt
congruente.
Formulaţi reciproca teoremei.

3. Fie teorema: Dacă două drepte sînt pa­ra­lele, atunci ele formează cu o secantă o
pereche de unghiuri alterne interne congruente.
Formulaţi reciproca teoremei.

4. Dacă două drepte sînt paralele, atunci ele formează cu o secantă o pereche de
unghiuri alterne externe congruente.
Formulaţi reciproca teoremei.

5. Dacă două drepte sînt paralele, atunci ele formează cu o secantă o pereche de
unghiuri corespondente congruente. Formulaţi reciproca teoremei.

6. Fie propoziţia: Un număr natural cu cifra unităţilor 6 se divide cu 6.


Formulaţi un contraexemplu.

7. Formulaţi o teoremă ce conţin teoremele din exerciţiul 1.

7
8. Formulaţi o teoremă ce conţin teoremele din exerciţiul 2.

9. Formulaţi o teoremă ce conţin teoremele din exerciţiul 3.

10. Formulaţi o teoremă ce conţin teoremele din exerciţiul 4.

11. Formulaţi o teoremă ce conţin teoremele din exerciţiul 5.

12. Formulaţi o teoremă ce conţin teoremele din exerciţiul 6.


13. Fie teorema: Un triunghi isoscel are două unghiuri congruente.
Formulaţi reciproca teoremei şi teorema de echivalenţă ce conţine ambele teo­
reme.

14. Fie teorema: Un triunghi echilateral are toate unghiurile congruente.


Formulaţi reciproca teoremei şi teorema de echivalenţă ce conţine ambele teo­
reme.

15. Fie teorema: O mediană a unui triunghi isoscel este şi bisectoare a triun­ghiului.
Formulaţi reciproca teoremei şi teorema de echivalenţă ce conţine ambele teo­
reme.

8
Cap 1. Recapitulare şi completări
3 Congruenţa triunghiurilor. Paralelism

Definţia congruenţei. Criterii de congruenţă


Două triunghiuri sînt congruente, dacă au C D
laturile respectiv congruente şi unghiurile F
corespunzătoare respectiv congruente.
ΔABC ≡ ΔDEF, dacă: [AB] ≡ [DE],
[BC] ≡ [EF], [AC] ≡ [DF], ÐA ≡ ÐD,
ÐB ≡ ÐE, ÐC ≡ ÐF. A B
E
Criteriul de congruenţă LUL (Axiomă). Dacă [AB] ≡ [DE], ÐA ≡ ÐD şi [AC] ≡
[DF], atunci ΔABC ≡ ΔDEF.
Criteriul de congruenţă ULU. Dacă [AB] ≡ [DE], ÐA ≡ ÐD şi ÐB ≡ ÐE, atunci
ΔABC ≡ ΔDEF.
Criteriul de congruenţă LLL. Dacă [AB] ≡ [DE], [BC] ≡ [EF], [AC] ≡ [DF], atunci
ΔABC ≡ ΔDEF.
Congruenţa triunghiurilor dreptunghice. Triunghiurile ABC şi DEF au unghiurile
drepte A şi D.
Criteriul CC. Dacă [AB] ≡ [DE] şi [AC] ≡ [DF], atunci ΔABC ≡ ΔDEF.
Criteriul CU. Dacă [AB] ≡ [DE] şi ÐB ≡ ÐE, atunci ΔABC ≡ ΔDEF.
Criteriul IU. Dacă [BC] ≡ [EF] şi ÐB ≡ ÐE, atunci ΔABC ≡ ΔDEF.
Criteriul IC. Dacă [BC] ≡ [EF] şi [AB] ≡ [DE], atunci ΔABC ≡ ΔDEF.
Criterii de paralelism. Consecinţe
Teorema paralelelor. Două drepte sînt paralele dacă şi numai dacă formează cu o
secantă o pereche de unghiuri alterne interne congruente, sau o pereche de unghiuri
alterne externe congruente, sau o pereche de unghiuri corespondente congruente, sau
o pereche de unghiuri interne de aceeaşi parte a secantei su- c
M
plementare, sau o pereche de unghiuri externe de aceeaşi a 1 4
2 3
parte a secantei suplementare.
Corolar. Dacă două drepte sînt paralele, atunci ele formea- b 5 8
6 7
ză cu o secantă unghiuri: alterne interne congruente, alterne N
externe congruente, corespondente congruente, interne de aceeaşi parte a secantei
suplementare sau externe de aceeaşi parte a secantei suplementare.
Tranzitivitatea paralelismului dreptelor. Teoremă. Dacă două drepte diferite sînt
paralele cu a treia dreaptă, atunci ele sînt paralele.
Teorema unghiului exterior (2). Măsura unui unghi exterior al triunghiului este
egală cu suma măsurilor unghiurilor triunghiului, neadiacente unghiului exterior.
Suma măsurilor unghiurilor unui triunghi. Teoremă. Suma măsurilor unghiurilor
unui triunghi este 180°.
Unghiurile triunghiului echilateral. Teoremă. Un triunghi este echilateral dacă şi
numai dacă are unghiurile de 60°.

9
Unghiurile triunghiului dreptunghic. Teoremă. Un triunghi este dreptunghic dacă
şi numai dacă are două unghiuri complementare.
Unghiurile triunghiului dreptunghic isoscel. Teoremă. Un triunghi este dreptun-
ghic isoscel dacă şi numai dacă are două unghiuri de 45°.
Mediana din vîrful unghiului drept. Teoremă. Un triunghi este dreptunghic
dacă şi numai dacă lungimea unei mediane este jumătate din lungimea laturii
corespunzătoare acelei mediane.
Teorema triunghiului dreptunghic cu un unghi de 30°. Un triunghi dreptung-
hic are un unghi de 30° dacă şi numai dacă lungimea unei catete este jumătate din
lungimea ipotenuzei.

Exerciţii
1. Completaţi propoziţiile:
a) Două triunghiuri sînt congruente, dacă:

b) ΔABC ≡ ΔDEF, dacă:

2. Completaţi propoziţiile:
a) ΔADK ≡ ΔBCE, dacă

b) ΔMFS ≡ ΔDEJ, dacă

c) ΔDEM ≡ ΔFJB, dacă

3. Completaţi criteriile de congruenţă:


a) (LUL) Dacă ÐA ≡ ÐD şi atunci ΔABC ≡ ΔDEF.
b) (ULU) Dacă [AB] ≡ [DE], şi atunci ΔABC ≡ ΔDEF.
c) (LLL) Dacă atunci ΔABC ≡ ΔDEF.
4. Completaţi raţionamentul:
LUL
a) ÐA ≡ ÐB şi ==> ΔAFG ≡ ΔBDI.
LUL
b) ÐC ≡ ÐD şi ==> ΔCET ≡ ΔDBS.
LUL
c) ÐS ≡ ÐM şi ==> ΔSAL ≡ ΔMPD.
LUL
d) ..., ÐF ≡ ÐJ şi ... ==> ΔFKG ≡ ΔJLO.
5. Completaţi raţionamentul:
ULU
a) [AF] ≡ [BD] şi ==> ΔAFG ≡ ΔBDI.
ULU
b) [CE] ≡ [HB] şi ==> ΔCET ≡ ΔHBS.
ULU
c) [SA] ≡ [MP] şi ==> ΔSAL ≡ ΔMPD.
ULU
d) [FK] ≡ [JL] şi . ==> ΔFKG ≡ ΔJLO.

10
Cap 1. Recapitulare şi completări
6. Completaţi raţionamentul:
LLL
a) şi ==> ΔAFG ≡ ΔBDI.
LLL
b) . şi ==> ΔCET ≡ ΔHBS.
LLL
c) şi ==> ΔSAL ≡ ΔMPD.
LLL
d) şi ==> ΔFKG ≡ ΔJLO.
7. Completaţi raţionamentul:
CU
a) (m(ÐA) = m(ÐB) = 90°) ÐF ≡ ÐD ==> ΔAFG ≡ ΔBDI.
IC
b) (m(ÐC) = m(ÐD) = 90°) [CE] ≡ [DB] ==> ΔCET ≡ ΔDBS.
IU
c) (m(ÐS) = m(ÐM) = 90°) ÐA ≡ ÐP ==> ΔSAL ≡ ΔMPD.
8. Fie a || b şi secanta c. Aflaţi măsurile celorlalte unghiuri dacă: c
M
a) m(ÐM2) = 34°; a 1 4
2 3

b) m(ÐN7) = 127°; b 5 8
6 7
N
c) m(ÐM1) = 115°;

d) m(ÐN8) = 54°.

9. Fie dreptele a, b şi secanta c. Stabiliţi poziţia dreptei a faţa c


M
de b, dacă: a 1 4
2 3

a) ÐM1 ≡ ÐN8;
b 5 8
6 7
N
b) ÐM4 ≡ ÐN8;

c) ÐM2 şi ÐN7 sînt suplementare;

d) ÐM3 şi ÐN6 suplementare.

10. Fie triunghiul ABC. Aflaţi:


a) m(ÐB), dacă m(ÐA) = 76° şi m(ÐC) = 59°;
b) m(ÐC), dacă m(ÐA) = 21° şi m(ÐB) = 102°;
c) m(ÐB), AB = AC şi m(ÐA) = 58°;
d) măsurile celorlalte unghiuri, dacă AB = AC şi m(ÐA) = 94°.

11
11. Fie triunghiul ABC cu m(ÐA) = 90°. Completaţi propoziţiile:
a) Dacă AB = AC, atunci m(ÐB) = ...

b) Dacă m(ÐB) = 30°, atunci AC = ...

c) Dacă m(ÐB) = 60°, atunci BC = ...

d) Dacă D este mijlocul [BC], atunci BC = ...

12. Fie triunghiul ABC cu m(ÐA) = 90°. Aflaţi:


a) BC, dacă m(ÐB) = 30° şi AC = 38 mm;
b) AC, dacă m(ÐB) = 60° şi BC = 78 mm;
c) AC, dacă m(ÐB) = 30° şi BC = 88 mm;
d) BC, dacă m(ÐB) = 60° şi AB = 93 mm.

4 Proprietăţi ale triunghiurilor


Proprietatea mediatoarei. Mediatoarea unui segment este mulţimea punctelor unui
plan, egal depărtate de capetele segmentului. A
Proprietatea bisectoarei. Un punct al interiorului unui unghi
aparţine bisectoarei unghiului dacă şi numai dacă este egal de-
părtat de laturile unghiului.
Proprietăţile mediatoarei bazei. Teoremă. Un triunghi este isos-
cel dacă şi numai dacă are o axă de simetrie. Axa de simetrie a
B D C
triunghiului isoscel este mediatoarea bazei lui.
Cercul ce conţine vîrfurile unui triunghi este cercul circumscris triunghiului.
Intersecţia mediatoarelor. Intersecţia mediatoarelor unui triunghi este centrul cer-
cului circumscris triunghiului.
Mijlocul ipotenuzei. Un triunghi este dreptunghic dacă şi numai dacă centrul cercu-
lui circumscris lui aparţine unei laturi.
Intersecţia bisectoarelor. Intersecţia bisectoarelor unui triunghi este centrul cercu-
lui înscris în triunghi.

12
Cap 1. Recapitulare şi completări
Un segment determinat de mijloacele a două laturi ale unui triunghi A
este o linie mijlocie a triunghiului.
Teorema liniei mijlocii (1). O linie mijlocie a unui triunghi este N
P
paralelă cu una dintre laturile triunghiului şi lungimea ei este ju-
mătate din lungimea acelei laturi.
Teorema liniei mijlocii (2). Un segment cu o extremitate mijlo- B M C
cul unei laturi a triunghiului şi cealaltă extremitate, aparţinînd altei laturi a triun­
ghiului este linie mijlocie a acestuia dacă şi numai dacă este paralelă cu latura a treia
a triunghiului.
Concurenţa înălţimilor triunghiului. Înălţimile unui triunghi sau dreptele ce le
conţin sînt concurente în punctul numit ortocentrul triunghiului.
Concurenţa medianelor triunghiului. Medianele triunghiului sînt concurente în
centrul de greutate al triunghiului şi centrul de greutate se află pe fiecare mediană, de
două ori mai aproape de mijlocul laturii decît de vîrful triunghiului opus acelei laturi.
Triunghiul echilateral. Teoremă. Medianele, bisectoarele, înălţimile şi mediatoarele
triunghiului echilateral sînt concurente în centrul cercului circumscris triunghiului.
Teorema inegalităţilor între elementele triunghiului. O latură a unui triunghi este
mai mică decît altă latură a triunghiului dacă şi numai dacă unghiul opus ei este mai
mic decît unghiul opus celeilalte laturi.
Perpendiculara dintr-un punct pe o dreaptă este segmentul determinat de punct şi
proiecţia lui pe această dreaptă.
Oblică este oricare alt segment determinat de punctul dat şi un punct al dreptei.
Teoremă. 1) Ipotenuza unui triunghi dreptunghic este mai lungă decît oricare dintre
catetele lui. 2) Perpendiculara coborîtă dintr-un punct pe o dreaptă este mai scurtă
decît oricare dintre oblicele coborîte din acel punct pe aceeaşi dreaptă. 3) Latura
opusă unghiului obtuz este cea mai lungă latură a triunghiului obtuzunghic.
Inegalitatea triunghiului. Lungimea unei laturi a unui triunghi este mai mică decît
suma lungimilor celorlalte două laturi ale triunghiului.

Exerciţii
1. Completaţi propoziţiile:
a) Mediatoarea unui segment

b) Un punct al interiorului unui unghi aparţine bisectoarei unghiului

2. Fie d mediatoarea segmentului AB şi M un punct al mediatoarei. Stabiliţi valoarea


de adevăr a propoziţiei:
a) [MA] ≡ [MB]. b) MA < MB.

c) MA > MB. d) MA ≥ 0,5AB.

13
3. Fie d bisectoarea unghiului A, P un punct oare- M
care al bisectoarei, M şi N proiecţiile lui P pe
laturile unghiului. Stabiliţi valoarea de A d
adevăr a propoziţiei: P
a) MP = PN.
N
b) MP < PN.
c) MP > PN.
4. Completaţi propoziţiile:
a) (Mediatoarea bazei) Un triunghi este isoscel dacă şi numai dacă

b) Axa de simetrie a triunghiului isoscel este

c) este cercul circumscris triunghiu­lui.


d) Intersecţia mediatoarelor unui triunghi este

e) Un triunghi este dreptunghic dacă şi numai dacă centrul cercului circumscris


lui este
5. Completaţi propoziţiile:
a) este o linie mijlocie a triunghiului.
b) O linie mijlocie a unui triunghi este
c) Un segment cu o extremitate mijlocul unei laturi, cealaltă extremitate aparţinînd
altei laturi a triunghiului
6. Aflaţi lungimile liniilor mijlocii ale triunghiului ABC cu:
a) AB = 24 mm, BC = 32 mm şi AC = 40 mm;
b) AB = 52 mm, BC = 76 mm şi AC = 88 mm;
c) AB = 66 mm, BC = 74 mm şi AC = 68 mm;
d) AB = 54 mm, BC = 92 mm şi AC = 74 mm.

7. Aflaţi perimetrul triunghiului ABC, dacă perimetrul triunghiului median (format


de liniile mijlocii) este:
a) 75 mm; b) 87 m; c) 91 mm; d) 105 mm.

14
Cap 1. Recapitulare şi completări
8. Aflaţi perimetrul triunghiului median (format de liniile mijlocii) al triunghiului
ABC, dacă perimetrul triunghiului ABC este:
a) 15 cm; b) 31 cm; c) 49 cm; d) 77 cm.

9. Formulaţi: a) Concurenţa înălţimilor triunghiului.


b) Concurenţa bisectoarelor triun­ghiu­lui.
c) Concurenţa medianelor triunghiului. d) (Proprietatea triunghiului echilateral.

10. Fie triunghiul ABC cu centrul de greutate G. Aflaţi: A


a) AG, dacă GA΄ = 2 cm;
b) GB΄, dacă BG = 4 cm; C΄ B΄
c) CG, dacă GC΄ = 5 cm; G
d) GA΄, dacă AG = 3 cm. B C

11. Formulaţi: a) Teorema inegalităţii între elementele triunghiului.


b) Teorema perpendicularelor şi obli­celor. c) Inegalitatea triunghiului.

15
5 Cerc. Disc circular
l Cercul de centru O şi rază R este mulţimea Ext C(O, R) Ext C(O, R)
C(O, R) C(O, R)
tuturor punctelor planului aflate la distanţa
R de centrul O. Notaţie: C(O, R) este R R

Ext C(O, R)
Ext C(O, R)

Ext C(O, R)
Ext C(O, R)
cercul de centru O şi rază R. O O

l Elementele unui cerc sînt: centrul, raza. Int C(O, R) D(O, R)

l Interiorul C(O, R) este:


Int C(O, R) = {M Î planului | OM < R}. Ext C(O, R) Ext C(O, R)

l Exteriorul C(O, R) este: Ext C(O, R) = {M Î planului | OM > R}.


l Discul circular de centru O şi rază R este: mi c
Ar c
D(O, R) = {M aparţine planului | OM £ R}.
l Coardă a unui cerc este un segment determinat Unghiul
O O
de două puncte ale cercului.
l Diametru al unui cerc este o coardă ce conţine
Coardă
centrul cercului. Diametrele cercului de rază R Puncte Diametru
au lungimea 2R. diametral
Ar e
l Capetele unui diametru sînt puncte diametral opuse c mar
Semicerc
opuse.
l Arc mic al unui cerc este intersecţia cercului cu interiorul unui unghi cu vîrful în
centrul cercului.
l Arc mare al unui cerc este intersecţia cercului cu exteriorul unui unghi cu vîrful în
centrul cercului.
l Un arc al unui cerc cu capetele puncte diametral opuse este un semicerc.

Exerciţii
1. Completaţi propoziţiile:
a) Cercul de centru O şi rază R este ...
b) Cercul de centru O şi rază R se notează ...
c) Interiorul cercului de centru O şi rază R ...
d) Exteriorul cercului de centru O şi rază R ...
2. Construiţi cu ajutorul compasului un cerc de centru O şi rază:
a) 34 mm;
b) 28 mm;
c) 43 mm;
d) 48 mm.

16
Cap 1. Recapitulare şi completări
3. Completaţi propoziţiile:
a) Coardă a unui cerc este ...;
b) Diametru al unui cerc este ...
c) Puncte diametral opuse ...
d) Arc mic al unui cerc ...
e) Arc mare al unui cerc ...
f) Semicerc este ...
D O
4. Recunoaşteţi:
a) Arcul mic şi arcul mare DC;
C
O
b) Arcul mic şi arcul mare EF; E
F M
c) Arcul mic şi arcul mare MN.
O N

6 Poziţiile unei drepte faţă de un cerc


l O dreaptă este exterioară unui cerc, dacă dreapta şi cercul
nu au puncte comune.
Teorema dreptei exterioare cercului. Dreapta e este exte-
rioară C(O, R) dacă şi numai dacă distanţa de la O la e
este mai mare decît R, adică d(O, e) > R.
l O dreaptă este tangentă cercului, dacă dreapta şi cercul au un
punct comun.
Teorema dreptei tangente cercului. Dreapta t este tangentă
C(O, R) dacă şi numai dacă distanţa de la O la t este egală
cu R, adică d(O, t) = R.
Raza în punctul de tangenţă este perpendiculară pe tangentă.

l O dreaptă este secantă cercului, dacă dreapta şi cercul au


două puncte comune.
Teorema dreptei secante cercului. Dreapta s este secantă
C(O, R) dacă şi numai dacă distanţa de la O la s este mai
mică decît R, adică d(O, s) < R.

17
Exerciţii
1. Formulaţi:
a) Teorema dreptei exterioare.

b) Teorema dreptei tangente cercului.

c) Teorema dreptei secante cercului.

d) Raza în punctul de tangenţă

2. Recunoaşteţi poziţia dreptei faţă de cerc:


d
d d
a) b) c)
O O O

3. Aflaţi poziţia dreptei:


a) e faţă de C(O, 2,4 cm), dacă d(O, e) = 51 mm;

b) m faţă de C(O, 7 mm), dacă d(O, m) = 6,8 mm;

c) f faţă de C(O, 4,3 cm), dacă d(O, f) = 43 mm;

d) p faţă de C(O, 54 mm), dacă d(O, m) = 8,2 cm.

4. Precizaţi ce proprietate are d(a, O), dacă:


a) dreapta a este tangentă cercului C(O; 3,4 cm);

b) dreapta a este tangentă cercului C(O, 92 cm);

c) dreapta a este tangentă cercului C(O, 25 mm);

d) dreapta a este tangentă cercului C(O, 16,2 cm).

5. Precizaţi ce proprietate are d(a, O), dacă:


a) dreapta a este exterioară cercului C(O, 54 cm);

b) dreapta a este exterioară cercului C(O, 41,8 cm);

18
Cap 1. Recapitulare şi completări
c) dreapta a este exterioară cercului C(O, 28 mm);

d) dreapta a este exterioară cercului C(O, 34 cm).

6. Precizaţi ce proprietate are d(a, O), dacă:


a) dreapta a este exterioară cercului C(O, 8,3 cm);

b) dreapta a este exterioară cercului C(O, 15 cm);

c) dreapta a este exterioară cercului C(O, 73 mm);

d) dreapta a este exterioară cercului C(O, 92 mm).

7. Precizaţi ce proprietate are d(a, O), dacă:


a) dreapta a este secantă cercului C(O, 44 cm);

b) dreapta a este secantă cercului C(O; 62,3 cm);

c) dreapta a este secantă cercului C(O, 45 mm);

d) dreapta a este secantă cercului C(O, 57 mm).

7 Poziţiile relative a două cercuri


l Două cercuri sînt exterioare, dacă ele nu au puncte
comune şi interioarele lor nu au puncte comune.
Teorema cercurilor exterioare. Cercurile
C(O, R) şi C(O′, R′) sînt exterioare dacă şi nu-
mai dacă distanţa dintre centrele cercurilor este
mai mare decît suma razelor cercurilor, adică
OO′ > R + R′.
l Două cercuri sînt tangente exterioare, dacă ele au un
singur punct comun şi interioarele lor nu au puncte co-
mune.
Teorema cercurilor tangente exterioare. Cercurile
C(O, R) şi C(O′, R′) sînt tangente exterioare dacă şi
numai dacă distanţa dintre centrele cercurilor este
egală cu suma razelor cercurilor, adică OO′ = R + R′.

19
l Două cercuri sînt secante, dacă ele au două puncte
comune.
Teorema cercurilor secante. Cercurile C(O, R) şi
C(O′, R′) sînt tangente secante dacă şi numai dacă
distanţa dintre centrele cercurilor este mai mică decît
suma razelor cercurilor, adică
OO′ < R + R′.
l Două cercuri sînt tangente interioare, dacă ele au un singur
punct comun şi interioarele lor au puncte comune. R
O 678
678
Teorema cercurilor tangente interioare. Cercurile C(O, R) şi 678
C(O′, R′) sînt tangente interioare dacă şi numai dacă distanţa O¢ R¢
dintre centrele cercurilor este egală cu diferenţa razelor cercu-
rilor, adică OO′ = R – R′.
l Două cercuri sînt interioare, dacă ele nu au puncte şi interioa-
rele lor au puncte comune.
R
Teorema cercurilor interioare. Cercurile C(O, R) şi C(O′, R′) O 678
sînt interioare dacă şi numai dacă distanţa dintre centrele cer- 678678
O¢ R¢
curilor este mai mică decît diferenţa razelor cercurilor, adică
OO′ < R – R′.
l Două cercuri sînt concentrice, dacă ele au acelaşi centru.
Teorema cercurilor concentrice. Cercurile C(O, R) şi C(O′, R′)
sînt concentrice dacă şi numai dacă distanţa dintre centrele cer-
curilor este egală cu zero. OO′ = 0.

Exerciţii
1. Reprezentaţi două cercuri:
a) tangente exterioare; b) exterioare; c) tangente interioare; d) secante.

2. Fie cercurile C(O, R) şi C(O′, R′). Completaţi în fiecare situaţie relaţiile dintre R, R′ şi
OO′:
a) b) O O¢
c) d)
O O¢ O
O O¢

20
Cap 1. Recapitulare şi completări
3. Fie cercurile C(O; 6,7 cm) şi C(O; 3,5 cm). Precizaţi proprietatea distanţei dintre cen­
trele cercurilor, dacă:
a) cercurile sînt tangente exterioare;
b) cercurile sînt exterioare;
c) cercurile sînt secante;
d) cercurile sînt tangente interioare.
4. Aflaţi distanţa dintre centrele cercurilor:
a) tangente exterioare C(O; 16,4 cm) şi C(O; 39,2 cm);
b) tangente exterioare C(O; 17,3 cm) şi C(O; 74,2 cm);
c) tangente exterioare C(O; 73,8 cm) şi C(O; 59,6 cm);
d) tangente exterioare C(O; 59,5 cm) şi C(O; 94,7 cm).

5. Aflaţi distanţa dintre centrele cercurilor:


a) tangente interioare C(O; 94,5 cm) şi C(O; 125 mm);
b) tangente interioare C(O; 78,4 cm) şi C(O, 164 mm);
c) tangente interioare C(O, 85,7 cm) şi C(O, 133 mm);
d) tangente interioare C(O; 99,6 cm) şi C(O, 241 cm).

8 Unghiuri
Unghi la centru
B
)

l Arcul AB (notat AB) este arc mic. Arcul ACB este arc mare.
A
)

Arcul AB (notat AB) corespunde unghiului AOB.


Un unghi cu vîrful în centrul unui cerc se numeşte unghi la centru. O
Unghiul AOB este unghi la centru. Arcul AB este subîntins de un-
C
)

ghiul AOB. Măsura arcului AB este m(AB) = m( AOB), măsura


arcului ACB = 360° – m( AOB).
Unghi înscris
Unghiul determinat de coardele MA şi MB este unghi înscris
în cerc.
A B
Unghiul format de două coarde cu o extremitate comună se O
numeşte unghi înscris în cerc.
Arcul AB este subîntins de unghiul înscris AMB şi acesta este
înscris în arcul AMB. M
Teorema unghiului înscris. Măsura unghiului înscris într-un
cerc este egală cu jumătate din măsura arcului subîntins de el.

21
B C B C
A A O B A y
x x y
x
O y O
x x y x
M M M
Demonstraţie. Analizăm cazul: [MB] este diametru al cercului. Toate situaţiile sînt
ilustrate mai sus.
Triunghiul isoscel AOM are unghiurile alăturate bazei de măsură x. Unghiul exte­
)
rior AOB are măsura 2x = m(AB). Rezultă că măsura unghiului înscris AMB este egală

)
cu jumătate din măsura arcului mic AB ­= 0,5 · m(AB).
Analizînd celelalte două cazuri, ajungem la aceeaşi concluzie: măsura unghiului

)
înscris AMB este egală cu jumătate din măsura arcului mic AB ­= 0,5 · m(AB).
Corolar. 1) Un semicerc are 180°.
2) Dacă un arc mic are n°, atunci arcul mare corespunzător lui are 360° – n°.
3) Un unghi este drept dacă şi numai dacă este înscris într-un semicerc.

Exerciţii

1. Fie situaţia ilustrată în desen. Com­pletaţi: B


A
a) Unghiul AOB este O

b) Arcul subîntins de unghiul AOB este C

c) Măsura arcului AB este


d) Măsura arcului ACB este
2. Recunoaşteţi unghiurile la centru ilustrate în desen. C B
D N
E
O
M
3. Construiţi unghiul la centru AOB de măsură: A

a) 75°; b) 102°; c) 60°; d) 120°. F P

4. Aflaţi măsura arcului mic ce se obţine, marcînd cu ajutorul compasului:


a) o coardă de lungime egală cu raza;
b) de două ori o coardă de lungime egală cu raza;

22
Cap 1. Recapitulare şi completări
c) de trei ori o coardă de lungime egală cu raza.

5. Fie situaţia ilustrată în desen. Completaţi: A


O
a) Unghiul ACB este
C
b) Arcul subîntins de unghiul ACB este
c) Unghiul înscris într-un semicerc este
B
8. Recunoaşteţi unghiurile înscrise ilustrate în desen. D
C
N
E
O M
9. Construiţi unghiul înscris BAC de măsură: F A
P
a) 120°; b) 90°; c) 60°; d) 135°.

10. Cu ajutorul riglei şi compasului construiţi un arc de măsură:


a) 60°; b) 30°; c) 45°; d) 75°; e) 105°.

11. Aflaţi măsura arcului mare ce se obţine marcînd cu ajutorul compasului:


a) o coardă de lungime egală cu raza;
b) de două ori o coardă de lungime egală cu raza;
c) de trei ori o coardă de lungime egală cu raza.

12. Fie triunghiul ABC înscris în cercul de rază R. Aflaţi măsurile arcelor B
subîntinse de unghiurile triunghiului, dacă: A
a) m( A) = 67°, m( B) = 98°; b) m( A) = 56°, m( C) = 101°;
c) m( B) = 63°, m( C) = 90°.
C

23
13. Fie triunghiul ABC înscris în cercul de rază R. Aflaţi măsurile unghiurilor triun­
ghiului, dacă măsurile arcelor subîntinse sînt:
) )

) )

)
a) m(BC ) = 74°, m(AC ) = 184°; b) m(BC ) = 112°, m(AC ) = 126°;
c) m(BC ) = 166°, m(AC ) = 188°.

Exerciţii recapitulative
1. Construiţi numai cu rigla negradată centrul cercului desenat cu ajutorul:
a) unui Compact Disc; b) unui borcan rotund; c) unui capac rotund.

2. Enumeraţi arcele mici şi arcele mari definite de triunghiul înscris:


A D G
H
I
B
F
C E
a) ABC; b) DEF; c) GHI.

3. Construiţi numai cu ajutorul riglei tangentele la cerc în punctele A, B, C.

A
O
B
C

24
Cap 1. Recapitulare şi completări
6. Triunghiul ABC este înscris în cercul de centru O. Completaţi tabelul: C
m( AOB) m( BOC) m( AOC) m( A) m( C)
O
78° 132° B
146° 55° A
21° 95°
112° 71°

7. Triunghiul ABC este înscris în cercul de centru O. Completaţi tabelul: C

O
)

)
)
m(AB) m(ABC ) m(BC ) m( A) m( B) B
102° 128° A
216° 65°
41° 73°
98° 55°

Evaluare
I II
1. Construiţi C(O, 33 mm) şi dreapta 1. Construiţi C(O, 37 mm) şi dreapta
d situată la distanţa 17 mm de O. Ce d situată la distanţa 19 mm de O. Ce
20
poziţie are dreapta faţă de C(O, 33 mm)? poziţie are dreapta faţă de C(O, 37 mm)?
2. Construiţi o tangentă într-un punct al 2. Construiţi o tangentă într-un punct al
C(O, 40 mm). 20 C(O, 43 mm).
3. Construiţi cu rigla şi compasul tan­ 3. Construiţi cu rigla şi compasul tan­
gentele din punctul M la C(O, 28 mm), 20 gentele din punctul M la C(O, 32 mm),
dacă OM = 7 cm. dacă OM = 8 cm.
4. Fie C(O, 41 mm). Aflaţi lungimea 4. Fie C(O, 38 mm). Aflaţi lungimea
20
coardei ce se află la distanţa 9 mm de O. coardei ce se află la distanţa 12 mm de O.
5. Aflaţi: 5. Aflaţi:
)

a) m(BC ), dacă C 20 a) m(BC ), dacă C


m( BAC) = 67°; m( BAC) = 53°;
b) m( BAC), dacă O B b) m( BAC), dacă O B
m( BOC) = 102°. A m( BOC) = 108°. A

25
Capitolul 2 Asemănarea triunghiurilor
1 Paralele echidistante

În construcţii se utilizează drepte paralele aflate la


aceeaşi distanţă.
Cu ajutorul riglei negradate se construiesc drepte pa-
ralele situate la distanţe egale.
Se numesc paralele echidistante, trei sau mai multe drepte paralele cu proprietatea
că distanţele dintre oricare două drepte alăturate sînt egale.
Cu ajutorul paralelelor echidistante se construiesc reţele de d e f
pătrate. În desen: a
– dreptele paralele a, b şi c sînt paralele echidistante, deoarece
d(a, b) = 2 = d(b, c) (distanţele dintre ele sînt egale); b
– dreptele paralele d, e şi f nu sînt paralele echidistante, deoa- c
rece d(d, e) = 3, d(e, f) = 2.
Distanţa dintre două drepte paralele este distanţa dintre
intersecţiile lor cu o perpendiculară pe cele două drepte.
d l k
A M E
B N F
C P G
D Q H

Construcţia dreptelor paralele cu ajutorul riglei negradate

1) Cu ajutorul riglei negradate s-au construit paralele echidistante (distanţa dintre ele
este egală cu lăţimea riglei).
2) Se construieşte o secantă ce intersectează dreptele paralelele echidistante.
3) Cu ajutorul compasului se constată că paralelele echidistante determină pe secantă
segmente congruente.

26
Cap 2. Asemănarea triunghiurilor
Exerciţii
1. Completaţi propoziţiile:
a) Se numesc paralele echidistante,

b) Distanţa dintre două drepte paralele

2. Recunoaşteţi paralelele echidistante:

a) b)

c) d)

e)

3. Construiţi cu ajutorul unei rigle negradate înguste:


a) 4 paralele echidistante; b) 5 paralele echidistante;
c) 8 paralele echidistante; d) 6 paralele echidistante.

27
4. Recunoaşteţi numărul maxim de paralele echidistante orizontale sau verticale ce
formează reţeaua de pătrate:

a) b)

c) d)

5. Recunoaşteţi în cîte segmente (numărul maxim) congruente împarte segmentul:


A
D

C
B
a) AB paralelele echidistante verticale ale reţelei de pătrate;
b) AB paralelele echidistante orizontale ale reţelei de pătrate;
c) CD paralelele echidistante verticale ale reţelei de pătrate;
d) CD paralelele echidistante orizontale ale reţelei de pătrate.

28
Cap 2. Asemănarea triunghiurilor
2 Teorema paralelelor echidistante
1) Punctele A, B, C, D determină cu dreapta d segmente congruente. d n
A E
2) Se construiesc paralele prin punctele A, B, C, D.
F
3) Se construieşte perpendiculara n pe dreptele paralele con- B
struite anterior. C G
H
4) Paralelele determină pe dreapta n segmente con- D

gruente.
Concluzie. Paralelele construite prin A, B, C, D sînt echidistante.
Teorema paralelelor echidistante. Trei sau mai multe drepte paralele sînt paralele
echidistante dacă şi numai dacă determină pe orice secantă segmente congruente.
Teorema directă (Necesitatea)
Ipoteza. Desenul, a || b || c || d, [AE] ≡ [BF] ≡ [CG]. m
a
Concluzia. [AB] ≡ [BC] ≡ [CD]. A

Demonstraţie. E = prb A, F = prc B, G = prd C, B b


(1). Conform criteriului CU, ipoteza şi (1) im- E
C c
plică: ∆EAB ≡ ∆BFC ≡ ∆CGD, (2). F
d
D
(2) implică [AB] ≡ [BC] ≡ [CD], q.e.d. G
Teorema directă (Suficienţa) m
Ipoteza. Desenul, a || b || c || d, [AB] ≡ [BC] ≡ [CD]. A a
Concluzia. [AE] ≡ [BF] ≡ [CG].
B b
Demonstraţie. E = prb A, F = prc B, G = prd C, (1). E
c
Conform criteriului IU, ipoteza şi (1) implică: C
F
∆EAB ≡ ∆BFC ≡ ∆CGD, (2). D d
G
(2) implică [AE] ≡ [BF] ≡ [CG], q.e.d.
Corolar. Dacă trei sau mai multe drepte paralele determină pe o dreaptă segmente
congruente, atunci ele determină segmente congruente pe orice dreaptă neparalelă
cu acestea.
Reţeaua de pătrate determină cu segmentul AB 6 segmente P
A B
congruente.
AP 2 1
Punctul P împarte segmentul AB în raportul: = = .
PB 4 2
Raportul a două segmente este raportul lungimlor acestor segmente, măsurate cu
aceeaşi unitate de măsură. M
MB 1 A B
Punctul M împarte segmentul AB în raportul: = .
MA 5 C
ND 1
Punctul N împarte segmentul CD în raportul: = .
NC 5
MB ND
Prin urmare, = . N D
MA NC
Se numesc segmente proporţionale patru segmente pentru care lungimile lor sînt
termenii unei proporţii.

29
Exerciţii
1. Construiţi cu ajutorul riglei şi echerului:
a) 4 paralele echidistante; b) 6 paralele echidistante.

2. Completaţi propoziţiile:
a) Trei sau mai multe drepte paralele sînt paralele echidistante,

b) Dacă mai mult de două drepte paralele determină pe o dreaptă segmente con-

gruente, atunci
3. Cu ajutorul reţelei de pătrate construiţi pe dreapta d cinci segmente congruente:

a) b)

d
d

c)
d

4. Construiţi pe o dreaptă cu ajutorul riglei negradate:


a) 3 segmente congruente; b) 5 segmente congruente;
c) 7 segmente congruente; d) 8 segmente congruente.

30
Cap 2. Asemănarea triunghiurilor
5. Completaţi propoziţiile:
a) Raportul a două segmente

b) Se numesc segmente proporţionale patru segmente

6. Examinaţi desenul şi aflaţi: M


A B
MB NC PE D
a) ; b) ; c) ; N
MA ND PF
MB NC PE C
d) ; e) ; f) ;
AB CD EF E P
MA ND PF
g) ; h) ; i) .
AB CD EF F

7. Examinaţi desenul şi aflaţi: A


AD AE AF AG
a) ; b) ; c) ; d) ;
BD ER FS CG D G
E F
AL AM AN AP
e) ; f) ; g) ; h) ;
BL MR NS PC M
L N P
AD AE AF AG
i) ; j) ; k) ; l) . B C
DL EM FN GP R S

31
8. Examinaţi desenul şi aflaţi: P C
G
AD AE AF AG A S
a) ; b) ; c) ; d) ; F E N
BD ER FS CG R
AL AM AN AP M
e) ; f) ; g) ; h) ; D
BL MR NS PC L
B
AD AE AF AG
i) ; j) ; k) ; l) .
DL EM FN GP

3 Construcţii geometrice
Împărţirea unui segment în trei segmente congruente
A A A A
A
A1 A1 A1
C
A2 A2 A2
D
A3 A3 A3
d B d B d B d B
B
1) Fie segmentul AB. 2) Se construieşte semidreapta d cu originea A.
3) Se marchează punctele semidreptei care determină segmentele congruente AA1,
A1A2, A2A3.
4) Se construieşte segmentul BA3 şi dreptele paralele cu BA3 prin punctele A1 şi A2.
5) Se marchează punctele de intersecţie ale paralelelor cu C şi D. Acestea împart seg-
mentul AB în 3 segmente congruente.
1
Împărţirea unui segment în raportul
2
A A A A
A
A1 A1 A1
y C

A2 A2 A2
d B d B d Bd B
B

32
Cap 2. Asemănarea triunghiurilor
1) Fie segmentul AB. 2) Se construieşte semidreapta d cu originea A.
3) Se marchează punctele semidreptei care determină trei segmente congruente şi
se notează numai două dintre aceste puncte A1 şi A2.
4) Se construieşte segmentul BA2 şi dreapta paralelă cu BA2 prin punctul A1.
5) Se marchează cu C punctul de intersecţie. Punctul C împarte segmentul AB în
1
raportul .
2
Teoremă. Există un singur punct al unui segment, care îl împarte într-un raport dat.
Reţeaua de drepte paralele echidistante. Pe o foaie de hîrtie transparentă se con­
struieşte o reţea de drepte paralele echidistante. Construcţiile de mai sus pot fi execu-
tate cu ajutorul acestei reţele conform ilustraţiilor:

A A A A

C C
D

B B B B
În acelaşi mod, aceste construcţii se pot executa cu ajutorul unei reţele de pătrate
sau dreptunghiuri, obţinute cu ajutorul dreptelor paralele echidistante.

Exerciţii
1. Completaţi împărţirea unui segment în segmente congruente:

A B
a)

C D
b)

E F
c)

G H
d)

33
2. Fie M un punct al segmentului AB. Aflaţi:
MB MA MB 2 MB MA MB 3
a) şi , dacă = . b) şi , dacă = .
AB AB MA 7 AB AB MA 8
MB MA MB 2 MB MA MB 4
c) şi , dacă = . d) şi , dacă = .
AB AB MA 9 AB AB MA 9

3. Împărţiţi un segment de:


a) 5 cm în 9 segmente congruente; b) 6 cm în 7 segmente congruente;
c) 7 cm în 11 segmente congruente; d) 8 cm în 13 segmente congruente.

4. Completaţi teorema:
Există un singur punct
5. Completaţi construcţia punctului care împarte un segment într-un raport dat.

MA 1 A B
a) M aparţine [AB] şi = .
MB 5

PC 2 C D
b) P aparţine [CD] şi = .
PD 3

E F
NE 1
c) N aparţine [EF] şi = .
NF 2

34
Cap 2. Asemănarea triunghiurilor
6. Construiţi punctul M al segmentului AB, dacă:
MA 3 MA 2
a) AB = 5 cm şi = ; b) AB = 7 cm şi = ;
MB 7 MB 9
MA 4 MA 3
c) AB = 11 cm şi = ; d) AB = 6 cm şi = .
MB 9 MB 8

7. Desenaţi pe o foaie de hîrtie transparen­tă o reţea de drepte paralele echidistante.


Cu ajutorul reţelei de paralele echidistante împărţiţi în:
a) 7 segmente congruente un segment de 87 mm;
b) 5 segmente congruente un segment de 73 mm;
c) 9 segmente congruente un segment de 94 mm.

8. Desenaţi pe o foaie de hîrtie transparen­tă o reţea de pătrate. Cu ajutorul reţelei de


pătrate construiţi:
MA 3
a) punctul M care împarte segmentul AB de 87 mm în raportul = ;
MB 5
MA 2
b) punctul M care împarte segmentul AB de 76 mm în raportul = ;
MB 7
MA 5
c) punctul M care împarte segmentul AB de 61 mm în raportul = .
MB 8

35
4 Teorema lui Thales
Pe reţeaua de pătrate sînt ilustrate trei situaţii de construire a unei drepte paralele
cu una dintre laturile unui triunghi. Ce relaţii există între unele dintre segmentele ce se
obţin?
A D G

M N E F
J

B C P Q H L

1) Triunghiul ABC, MN || BC, M între A şi B. Punctele A, B, C, M, N aparţin unora


dintre orizontalele reţelei. Conform teoremei paralelelor echidistante:
AM 3 AN 3 AM AN
= şi = , de unde = .
MB 2 NC 2 MB NC
2) Triunghiul DEF, PQ || EF, E între D şi P. Punctele D, E, F, P, Q aparţin unora
dintre orizontalele reţelei. Conform teoremei paralelelor echidistante:
DP 5 DQ 5 DP DQ
= şi = , de unde = .
PE 2 QF 2 PE QF
3) Triunghiul GHJ, KL || GH, J între G şi L. Punctele G, H, J, K, L aparţin unora
dintre verticalele reţelei. Conform teoremei paralelelor echidistante:
GL 8 HK 8 GL HK
= şi = , de unde = .
JL 3 JK 3 JL JK
Teorema lui Thales. Fie triunghiul ABC, M între A şi B, MN || BC. A
AM AN
Atunci = . (V. mai sus toate situaţiile posibile.)
MB NC
M N
Corolar. Fie triunghiul ABC, MN || BC, M între A şi B. Atunci:
AM AN AM AN MB NC B C
= , = , = .
MB NC AB AC AB AC
Teorema bisectoarei. [AD] este bisectoare a triunghiului ABC
E
AB BD
dacă şi numai dacă = . (Se construieşte CE || AD. Se
A AC DC
aplică teorema lui Thales şi se obţine rela-
D
ţia cerută.)
B C A
Teorema bisectoarei exterioare. [AD]
este bisectoare exterioară a triunghiului ABC dacă şi numai
E
AB BD B
dacă = . C D
AC DC

36
Cap 2. Asemănarea triunghiurilor
Exerciţii
1. Completaţi propoziţiile:
a) Fie triunghiul ABC, M între A şi B (A−M−B), MN || BC. Atunci

b) (Corolar) Fie triunghiul ABC, MN || BC, M între A şi B. Atunci

2. Completaţi raţionamentul:
D
a) Fie triunghiul DEF, D−S−E, ST || EF, implică
S T

E F
G
b) Fie triunghiul GHI, G−Q−H, QR || HI, implică
Q R
H I
K
c) Fie triunghiul KLM, K−A−L, AB || LM, implică
A B
L M
3. Fie triunghiul ABC, A−D−B, DE || BC. Aplicaţi teorema lui Thales şi A
completaţi tabelul:
D E
AD BD AB AE EC AC
B C
30 70 15
57 20 80
15 90 60
81 24 72

4. Fie triunghiul DEF, D−E−P, PR || EF. Aplicaţi teorema lui Thales şi D

completaţi tabelul:
DE PE DP DF FR DR E F
75 25 81 P R

68 72 90
48 60 17
84 18 108

37
5. Fie triunghiul GHJ, D−G−H, DE || HJ. Aplicaţi teorema lui Thales E D
G
şi completaţi tabelul:
HG GD HD GJ GE EJ H J

64 16 72
17 56 70
72 96 21
84 15 90

6. Examinaţi desenul şi completaţi: A B D


FL GM NH C E
= = =
AF BG HE G H J
F M N I P
IO JP L O
= =
CI DJ

7. Completaţi propoziţiile:
a) [AD] este bisectoare a triunghiului ABC, dacă

b) [AD] este bisectoare exterioară (a unghiului exterior) a triunghiului ABC, dacă

8. [AD] este bisectoare a triunghiului ABC. Aflaţi: A


BD
a) , dacă AB = 5 cm şi AC = 7 cm;
DC
BD B C
b) , dacă AB = 8 cm şi AC = 9 cm; D
DC
BD
c) , dacă AB = 12 cm şi AC = 16 cm.
DC

38
Cap 2. Asemănarea triunghiurilor
9. [AD] este bisectoare exterioară (a unui unghi exterior) a triunghiului ABC. Aflaţi:
BD BD
a) , dacă AB = 11 cm şi AC = 15 cm; b) , dacă AB = 25 cm şi AC = 21 cm;
DC DC
BD
c) , dacă AB = 28 cm şi AC = 26 cm.
DC

5 Reciproca teoremei lui Thales


Pe reţeaua de pătrate sînt ilustrate trei situaţii de alegere a două puncte, cîte unul pe
fiecare dintre dreptele ce conţin două dintre laturile unui triunghi. Ce poziţie are dreap-
ta determinată de aceste puncte faţă de drepta ce conţine cealaltă latură a triunghiului?
A D G

M N E F
J

L
B C P Q H

AM AN 3
1) Triunghiul ABC, M între A şi B, N între A şi C, astfel încît = = .
MB NC 2
Punctele A, B, C, M, N aparţin unora dintre orizontalele reţelei. Conform teoremei
unicităţii punctului ce împarte un segment într-un raport dat, MN || BC.
DP DQ 5
2) Triunghiul DEF, E între D şi P, F între D şi Q, astfel încît = = . Punc-
PE QF 2
tele D, E, F, P, Q aparţin unora dintre orizontalele reţelei. Conform teoremei unicităţii
punctului ce împarte un segment într-un raport dat, PQ || EF.
GL HK 8
3) Triunghiul GHJ, J între G şi L, J între H şi K astfel încît = = . Punc-
JL JK 3
tele G, H, J, K, L aparţin unora dintre verticalele reţelei. Conform teoremei unicităţii
punctului ce împarte un segment într-un raport dat, KL || GH.

39
Reciproca teoremei lui Thales. Fie triunghiul ABC, M între A şi B A
AM AN
(A−M−B), A−N−C, astfel încît = . Atunci MN || BC.
MB NC M N
(V. mai sus toate situaţiile posibile.)
Corolar. Fie triunghiul ABC, A−M−B, A−N−C, astfel încît B C
AM AN AM AN MB NC
= sau = sau = .
MB NC AB AC AB AC
Atunci, MN || BC.
Teorema lui Thales A D A D
generalizată.
M N M N
Fie A−M−B, D−N−C, P
AD || BC. AD || MN || BC
dacă şi numai dacă:
AM DN
= .
MB NC B C B R C

Exerciţii
1. Completaţi propoziţiile:
a) Fie triunghiul ABC, A−M−B, A−N−C, astfel încît Atunci

b) (Corolar) Fie triunghiul ABC, A−M−B, A−N−C, astfel încît Atunci

D
2. Completaţi raţionamentul:
a) Fie triunghiul DEF, D−S−E, D−T−F, astfel încît S T

E F
Atunci ST ||
G
b) Fie triunghiul GHI, G−Q−H, G−R−I, astfel încît
Q R
Atunci QR || ... H I
K
c) Fie triunghiul KLM, K−A−L, K−B−M, astfel încît
A B
Atunci AB || ... M
L

3. Fie triunghiul ABC, A−D−B, A−E−C. Aplicînd reciproca teoremei lui A


Thales, completaţi tabelul:
AD BD AB AE EC AC DE || BC
D E
30 90 15 45
21 84 23 92 B C
14 35 16 40
27 45 21 36

40
Cap 2. Asemănarea triunghiurilor
4. Fie triunghiul DEF, D−E−P, D−F−R. Aplicînd reciproca teore- D
mei lui Thales, calculaţi şi completaţi tabelul:
DE PE DP DF FR DR PR || EF
81 27 87 29 E F
34 51 38 56 P R
48 60 56 70
24 56 18 42

5. Fie triunghiul GHJ, D−G−H, E−G−J. Aplicînd reciproca teoremei E D


G
lui Thales, completaţi tabelul:
HG GD HD GJ GE EJ DE || HJ H J

72 24 75 26
24 84 26 91
39 91 52 99
28 49 32 56

7. Fie triunghiul ABC, A−D−B, A−E−C. Decideţi dacă DE || BC, ştiind A


că:
AD 3 AE 3 AD 3 AE 3 D E
a) = şi = ; b) = şi = ;
DB 4 AC 7 AB 5 EC 2
BD 1 AE 3 BD 2 EC 2 B C
c) = şi = ; d) = şi = .
AD 4 AC 5 AB 9 AE 7

41
8. Fie triunghiul ABC, D−A−B, E−A−C. Decideţi dacă DE || BC, E D
ştiind că: A
AD 3 AE 2 AD 2 AE 2
a) = şi = ; b) = şi = ;
DB 4 AC 7 AB 7 EC 9 B C
AD 1 AE 1
c) = şi = .
BD 6 AC 5

9. Completaţi propoziţia:
Fie A−M−B, D−N−C, AD || BC. AD || MN || BC dacă şi numai dacă

A D
10. Analizaţi desenul. AD || CF. Decideţi dacă BE || AD, ştiind că: E
B
AB DE AB DE AB DE BC EF
a) = ; b) ≠ ; c) = ; d) ≠ . F
BC EF BC EF AC DF AC DF
C

42
Cap 2. Asemănarea triunghiurilor
6 Construcţii geometrice
Situaţiile cercetate anterior ilustrează şi posibilitatea de a construi un segment de lun-
gime necunoscută (al patrulea proporţional).
A D G

M N E F
J

L
B C P Q H
AM AN
1) Se dau AM, BM, AC şi se cere AN astfel încît = . Se construieşte MN ||
AB AC
AM AN AM AN
BC. Aplicînd teorema lui Thales, rezultă = , de unde rezultă = (o
MB NC AB AC
proporţie derivată din prima). Soluţia este segmentul AN.
DP DQ
2) Se dau DE, EP, DF şi se cere FQ astfel încît = . Se construieşte PQ || EF,
PE QF
DP DQ
unde Q Î (DF. Aplicînd teorema lui Thales, rezultă = . Soluţia problemei este
PE QF
segmentul FQ.
GJ HJ
3) Se dau GJ, JL, HJ şi se cere JK astfel încît = . Se construieşte KL || GH,
JL JK
GL HK
unde K Î (HJ. Aplicînd teorema lui Thales, rezultă = . Soluţia problemei este
JL JK
segmentul JK.
Mai jos este ilustrat modul în care se împarte [AB] în segmente direct proporţionale
cu două segmente de lungimi date.
x E y
A B A B

C
Din istoria matematicii

Thales din Milet (640–540 î.Hr.) a fost fondatorul Şcolii de Filosofie din Ionia
(Asia Mică). A fost considerat unul dintre cei 7 înţelepţi ai Greciei Antice şi se
consideră că el a adus din Egipt cunoştinţele de geometrie şi de astronomie. De
asemenea, lui i se atribuie introducerea demonstraţiilor teoremelor în geometrie,
ceea ce a însemnat un pas însemnat în fundamentarea geometriei ca ştiinţă.

43
Exerciţii
1. Finalizaţi construcţia celui de al patrulea proporţional:
A A

a) b)
M M

B C
B C A
A

c) M d) M

B C B C
2. Finalizaţi construcţia celui de al patrulea proporţional:
D
D

a) b) E F
E F
P
P

c) E F

3. Finalizaţi construcţia celui de al patrulea proporţional:


G
G

a) b)
J J
H
L H L
G

c)
J

H L
4. Finalizaţi împărţirea segmentului AB în segmente direct proporţionale cu segmen-
tele date.

44
Cap 2. Asemănarea triunghiurilor
A B
A B
a) b)

5. Împărţiţi segmentul AB în segmente direct proporţionale cu segmentele date.

A B

7 Asemănarea triunghiurilor

Două triunghiuri sînt asemenea dacă au A D


unghiurile respectiv congruente şi laturile lor sînt
respectiv proporţionale. ∆ABC ~ ∆DEF dacă
AB
A ≡ D, B ≡ E, C ≡ E şi =
DE
BC AC
= = k. k este coeficientul de ase- E F
EF DF
mănare. Laturi omoloage: AB şi DE, B C

BC şi EF, AC şi DF. Raportul a două laturi omoloage este un raport de asemănare.


MN || KL, M ≡ K, N ≡ L
L
(unghiuri corespondente) şi N
JM MN JN 3
= = = .
JK KL JL 4
Prin urmare, ∆JMN ~ ∆JKL.
J K
M

45
Teorema fundamentală a asemănării (TFA). O dreaptă A
paralelă cu una dintre laturile unui triunghi formează cu
dreptele ce conţin celelalte laturi un triunghi asemenea
cu cel dat.
M N

Triunghiul ABC, MN || BC. Atunci ∆AMN ~ ∆ABC.


B C
Ipoteza. Desenul, MN || BC. (1)
Concluzia. ∆AMN ~ ∆ABC.
Demonstraţie. Fie NP || AB, unde punctul P se află
între punctele B şi C. (2) A
Conform proprietăţilor dreptelor paralele, (1) implică:
Ð AMN ≡ Ð ABC (unghiuri corespondente);
Ð ANM ≡ Ð ACB (unghiuri corespondente). (3)
M N
(1) şi (2) implică: BPNN este paralelogram, de unde
[MN] ≡ [BP]. (4)
AM AN
Conform teoremei lui Thales, (1) implică: = . (5) B P C
AB AC
AN BP
Conform teoremei lui Thales, (2) implică: = . (6)
AC BC
AN MN AM AN MN
(4) şi (6) implică: = , (7). (5) şi (7) implică: = = , (8).
AC BC AB AC BC
(3), (4) şi (8) implică: ∆AMN ~ ∆ABC, q.e.d.

Exerciţii
1. Recunoaşteţi figurile geometrice asemenea:

a) b)

c) d)

2. Recunoaşteţi ce figură geometrică s-a obţinut prin deformarea figurii iniţiale:

a) b)

46
Cap 2. Asemănarea triunghiurilor
c)

3. Completaţi propoziţile:
a) ∆ABC ~ ∆DEF dacă

b) Laturile omoloage sînt


c) Rapoartele de asemănare sînt

4. Completaţi propoziţiile conform definiţiei triunghiurilor asemenea:


a) ∆ADE ~ ∆BCF dacă

b) ∆BGF ~ ∆ADC dacă

c) ∆MIH ~ ∆NPE dacă

d) ∆KLN ~ ∆CIF dacă

5. Completaţi teorema fundamentală a asemănării (TFA):


O dreaptă paralelă cu una dintre

6. Completaţi raţionamentul: A
a) Triunghiul ADE, FG || DE implică
F G
D E
b) Triunghiul BCI, DE || CI implică B
D E
C I
c) Triunghiul JKL, MN || KL implică J
M N
K L
7. Completaţi raţionamentul: S T
a) Triunghiul PRS, TU || SP implică R

P
U

47
N
b) Triunghiul CGL, MN || CG implică
C
L

G M
c) Triunghiul ABD, AB || EF implică A
F
D

E B
8. Aplicînd TFA, descoperiţi toate triun­ghiurile asemenea:
a) MN || BC, NP || AB implică A
N
M C

P
B
b) MP || BC, OL || AB, M–N–P, O–N–L implică A O P C

N
M
L
B
c) GE || BC, GI || CD, I–H–G, C–D–F, E–F–G implică B
G
H
F
I D
C E

8 Criterii de asemănare a triunghiurilor

Situaţii de aplicare a teoremei fundamentale a asemănării (TFA):


A D G

M N E F
J

L
B C P Q H

1) Triunghiul ABC, MN || BC. Atunci ∆AMN ~ ∆ABC.

48
Cap 2. Asemănarea triunghiurilor
2) Triunghiul DEF, PQ || EF. Atunci ∆DEF ~ ∆DPQ.
3) Triunghiul GHJ, KL || GH, J între G şi L. Atunci ∆GHJ ~ ∆LKJ.

Criteriul I de asemănare (UU). A D


Dacă două triunghiuri au două
unghiuri respectiv congruente,
atunci ele sînt asemenea.
Dacă B ≡ E, C ≡ F,
G H E F
atunci ∆ABC ~ ∆DEF.
B C

Criteriul II de asemănare (LUL).


A D
Dacă două triunghiuri au două
laturi respectiv proporţionale şi
unghiurile formate de ele congru-
ente, atunci ele sînt asemenea.
AB AC G H E F
Dacă = şi A ≡ D,
DE DF
atunci ∆ABC ~ ∆DEF. B C

Criteriul III de asemănare (LLL). A D


Dacă două triunghiuri au laturile
respectiv proporţionale, atunci
ele sînt asemenea.
AB BC AC
Dacă = = , G H E F
DE EF DF
atunci ∆ABC ~ ∆DEF. B C

Exerciţii
1. Completaţi criteriul I de asemănare a triunghiurilor (UU):
Dacă două triunghiuri au

2. Examinaţi desenul şi completaţi raţio­namentul:


a) m(ÐA) = 90°, m(ÐD) = C F
UU
Ð ≡Ð ==> Δ ~Δ
A B D E

49
b) m(ÐG) = 90°, m(ÐJ) = Ð ≡Ð
UU
==> I L

UU G H
==> Δ ~Δ J K
UU
c) m(ÐK) = 90°, m(ÐN) = Ð ≡Ð ==> M P

K L
Δ ~Δ
N O

3. Examinaţi desenul şi completaţi raţio­namentul:


a) Triunghiurile isoscele ABC (cu baza BC), DEF (cu baza EF) A D

isoscele ABC (cu baza BC), DEF (cu baza EF)


UU
B C
≡Ð F
Ð ==> ÐB ≡ Ð ==> Δ ~Δ E

b) Triunghiurile isoscele GHI (cu baza HI), JKL(cu baza KL) G J

UU
Ð ≡Ð ==> ÐI ≡ Ð ==> Δ ~Δ H I
K L
Observaţie. Dacă două triunghiuri isoscele au unghiurile opuse bazelor congruente,
atunci celelalte unghiuri ale celor două triunghiuri sînt congruente. A
4. Triunghiul ABC are înălţimile AA΄, BB΄, CC΄. Completaţi
propoziţile:
a) Triunghiurile dreptunghice şi sînt asemenea B΄

(∆ABB΄ ~ ∆ ), deoarece au unghiul comun C
B A΄
b) Triunghiurile dreptunghice şi sînt asemenea
(∆CBC΄ ~ ∆ ), deoarece au unghiul comun
c) Triunghiurile dreptunghice şi sînt asemenea (∆BCB΄ ~ ∆ ),
deoa­rece au unghiul comun

5. Examinaţi desenul. Recunoaşteţi perechile de triunghiuri asemenea A


conform criteriului de asemănare UU.
Enumeraţi toate triunghiurile asemenea din desen. N
P
B C
M

6. Completaţi criteriul II de asemănare a triunghiurilor (LUL):


Dacă două triunghiuri au două laturi

7. În desen sînt ilustrate triunghiurile asemenea ABC, DEF şi medianele lor.

50
Cap 2. Asemănarea triunghiurilor
A
D

C΄ B΄ F΄ E΄

B C E F
A΄ D΄
AB AC AB΄
Model. ΔABC ~ ΔDEF implică ÐA ≡ ÐD şi = = . Conform criteriului
DE DF DE΄
de asemănare LUL, ΔABB΄ ~ ΔDEE΄.
Demonstraţi ca în model:
a) ΔABA΄ ~ ΔDED΄; b) ΔBCB΄ ~ ΔEFE΄; c) ΔACA΄ ~ ΔDFD΄.

8. Completaţi criteriul II de asemănare a triunghiurilor (LLL):


Dacă două triunghiuri au
A
9. Punctele M, N, P sînt mijloacele laturilor triunghiului ABC. N
Demonstraţi că ΔMNP ~ ΔABC.
P C
M
B

51
Exerciţii recapitulative
1. Împărţiţi un segment de:
a) 9 cm în 4 segmente congruente; b) 11 cm în 5 segmente congruente.

2. Construiţi punctul M al segmentului AB, dacă:


MA 2 MA 3
a) AB = 10 cm şi = ; b) AB = 15 cm şi = .
MB 5 MB 7

3. Examinaţi desenul şi aflaţi: A


AD AE AF AG
a) ; b) ; c) ; d) ;
AL AM AN AP D G
AL AM AN AP E F
e) ; f) ; g) ; h) .
AB AR AS AC
DL EM FN GP L M N P
i) ; j) ; k) ; l) .
DB ER FS GC B C
R S

52
Cap 2. Asemănarea triunghiurilor
4. Examinaţi desenul. Se ştie că: BF || CG, B–D–F, C–E–G, A–D–E.
A
Aplicînd teorema lui Thales, completaţi tabelul:
AB BC AC AD DE AE AF FG AG
2 5 8 3 B F
D

C G
E

5. Examinaţi desenul şi completaţi. A B C


D
AE BF CH DG
= = = = E F G H
AI BJ CL DK
I J K L

6. Examinaţi desenul. Se ştie că: AC || DF || GJ, A–B–C, D–E–F, A B C


G–I–J.Aplicînd teorema lui Thales, completaţi tabelul:
AD DG AG BE EI BI CF FJ CJ D E F
G I J
4 5 7 8

7. Triunghiul ABC, A–D–B, A–E–C. Aplicînd reciproca teoremei lui A


Thales, calcuculaţi şi completaţi tabelul:
D E
AD BD AB AE EC AC DE || BC
B C
12 48 14 42
17 85 21 85
16 80 15 90
31 124 17 52

53
8. Enumeraţi condiţiile ce definesc asemănarea triunghiurilor, perechile de laturi
omoloage, rapoartele de asemănare, coeficienţii de asemănare.
a) ΔDEL ~ ΔAEG; b) ΔFKL ~ ΔMPR.

G P C
9. Examinaţi desenul. Aplicaţi TFA şi enu- S
A
meraţi perechile de triunghiuri asemenea. F E N
R
D M
L
B

10. Examinaţi desenul. ΔABC ~ ΔDEF şi bisectoa- A D


rele triunghiurilor. Arătaţi că:
a) ΔABA΄ ~ ΔDED΄; b) ΔCBC΄ ~ ΔFEF΄; B΄ F΄ E΄
c) ΔABB΄ ~ ΔDEE΄; d) ΔCAA΄ ~ ΔFDD΄. C΄
E F

B C

11. Thales a aflat înălţimea piramidei lui Kheops măsurînd umbra unui baston de
lungime cunoscută şi umbra piramidei. Se obţin două triunghiuri dreptunghice ase-
menea. În acelaşi mod se poate afla înălţimea unui copac. Aflaţi A
înălţimea copacului AB, dacă:
a) umbra copacului BC = 80 m, umbra bastonului EF = 5 m, D
lungimea bastonului DE = 2 m;
B C EF
b) umbra copacului BC = 70 m, umbra bastonului EF = 4 m,
lungimea bastonului DE = 2 m;
c) umbra copacului BC = 78 m, umbra bastonului EF = 4,5 m, lungimea bastonu-
lui DE = 2 m.

54
Cap 2. Asemănarea triunghiurilor
Evaluare
I II
1. Împărţiţi în 5 segmente congruente 1. Împărţiţi în 6 segmente congruente
20
un segment de 78 mm. un segment de 75 mm.
2. Construiţi punctul M al segmentului 2. Construiţi punctul P al segmentului
MA 3 20 PM 3
AB de 86 mm, astfel încît = . MN de 92 mm, astfel încît = .
MB 5 PN 7
3. Examinaţi desenul. A 3. Examinaţi desenul. I
Se ştie că: DE || BC, Se ştie că: EF || JK,
A–D–B. Aplicînd teore- D E I–E–J. Aplicînd teore- E F
ma lui Thales, aflaţi B C 10 ma lui Thales, aflaţi J K
AD, dacă BD = 25 mm, AE = 8 mm, EJ, dacă EI = 24 mm, IF = 11 mm,
EC = 32 mm. FK = 44 mm. I
A
4. Triunghiul ABC, A–D–B, 4. Triunghiul IJK, I–E–J,
A–E–C. Aplicînd reci- I–F–K. Aplicînd reci- E F
D E
proca teoremei lui proca teoremei lui
Thales, stabiliţi dacă B C Thales, stabiliţi dacă J K
10
DE || BC, ştiind că AD = 17 mm, AB = EF || JK, ştiind că EJ = 19 mm, EI = 57
68 mm, EC = 36 mm, AC = 48 mm. mm, IK = 84 mm, FK = 21 mm.
5. Stabiliţi valoarea de adevăr a propo- 5. Stabiliţi valoarea de adevăr a propo-
ziţiei: dacă m(ÐA) = 90° = m(ÐD) şi ziţiei: dacă m(ÐD) = 90° = m(ÐJ) şi
10
m(ÐB) = 90° – m(ÐF), atunci ΔABC ~ m(ÐL) = 90° – m(ÐK), atunci ΔDGL ~
ΔDEF. D ΔJKM. B C
A A
6. AF || BE || DG. B 6. AD || BE || CF.
Enumeraţi perechile de 20
Enumeraţi perechile de E
triunghiuri asemenea. F E G triunghiuri asemenea. F
D
7. ΔAFG ~ ΔBDC. Demonstraţi că ra- 7. ΔKLM ~ ΔNPR. Demonstraţi că ra-
portul medianelor din A şi B este egal cu 10 portul medianelor din K şi N este egal cu
raportul de asemănare a triunghiurilor. raportul de asemănare a triunghiurilor.

55
Capitolul 3 Triunghiul dreptunghic. Relaţii metrice
1 Proiecţii ortogonale
P Î a este proiecţia ortogonală a punctului M pe dreapta a,
dacă MP ┴ a (se notează P = pra M). M
Proiecţia ortogonală a unui punct pe o dreaptă este intersec-
ţia dreptei cu perpendiculara din acel punct pe acea dreaptă.
Dacă un punct aparţine dreptei, atunci proiecţia punctului pe
a
acea dreaptă coincide cu acel punct.
P
Distanţa de la un punct la o dreaptă este distanţa de la acel
punct la proiecţia sa pe acea dreaptă.
În desen, distanţa de la punctul M la dreapta a este MP şi se notează d(M, a).
Construcţia proiecţiei punctului A pe dreapta d cu ajutorul riglei negradate:
se construieşte cu ajutorul riglei negradate paralela prin A la dreapta d; se construieşte
cu ajutorul riglei negradate perpendiculara din A pe dreapta d; se obţine B = prd A.

A A A

d d d
B

Proiecţia ortogonală a unui segment pe o dreaptă


E
B
A C D
F
m n k
G H P Q M

Proiecţia ortogonală a unui segment pe o dreaptă este seg-


N mentul determinat de proiecţiile ortogonale ale extremităţilor
segmentului. [DE] este proiecţia segmentului MN pe dreap-
ta d şi se notează [DE] = prd [MN].
P
Lungimea proiecţiei unui segment pe o dreaptă este:
a
– egală cu 0, dacă segmentul este perpendicular pe dreaptă;
Q
– egală cu lungimea segmentului, dacă segmentul este para-
lel cu dreapta;
M – mai scurtă decît lungimea segmentului, dacă segmentul
este conţinut de o dreaptă concurentă cu dreapta dată.

56
Cap 3. Triunghiul dreptunghic. Relaţii metrice
Exerciţii
1. Completaţi propoziţiile:
a) P Î a este proiecţia ortogonală a punctului M pe dreapta a, dacă

b) Proiecţia punctului M pe dreapta a se notează

c) Dacă un punct aparţine dreptei, atunci proiecţia punctului pe acea dreaptă

2. Construiţi în fiecare situaţie proiecţiile punctului notat pe dreptele date:


M
M
a) b)
b b
a a

M M
c) d)
b
a a
b

3. Notaţi:
a) B este proiecţia punctului A pe dreapta d;
b) C este proiecţia punctului D pe dreapta m;
c) D este proiecţia punctului E pe dreapta n;
d) P este proiecţia punctului B pe dreapta e.
4. Completaţi propoziţiile:
a) Distanţa de la un punct la o dreaptă este distanţa

b) P = praM. Distanţa de la punctul M la dreapta a este şi se notează

5. Completaţi:
a) A = prbD, atunci distanţa de la D la dreapta b se notează
b) C = praB, atunci distanţa de la B la dreapta a se notează
c) F = prcP, atunci distanţa de la P la dreapta c se notează
d) T = prmS, atunci distanţa de la S la dreapta m se notează

57
6. Construiţi cu ajutorul riglei negradate proiecţia punctului dat pe dreapta dată:
B D

a) b)

b
a

7. Completaţi propoziţiile:
a) Proiecţia ortogonală a unui segment pe o dreaptă este

b) [DE] este proiecţia segmentului MN pe dreapta d se notează

c) Lungimea proiecţiei unui segment pe o dreaptă este:

– egală cu 0, dacă

– egală cu lungimea segmentului, dacă

– mai scurtă decît lungimea segmentului, dacă

8. Notaţi:
a) proiecţia [MN] pe dreapta a; b) proiecţia [CD] pe dreapta b;
c) proiecţia [PQ] pe dreapta m; d) proiecţia [ST] pe dreapta d.

2 Teorema înălţimii triunghiului dreptunghic



1. Media proporţională sau geometrică a numerelor 7 şi 28 este numărul √7 · 28 = 14.
x = 28 .
Acelaşi rezultat se obţine calculînd numărul nenegativ x din proporţia
7 x
Media proporţională sau geometrică a numerelor nenega-
– A
tive şi b este numărul √ab .
2. Pentru a construi triunghiul dreptunghic cu o catetă de
5 şi mediana corespunzătoare ipotenuzei de 6, se ţine
B O C
cont că ipotenuza triunghiului este dublul medianei co-
respunzătoare ipotenuzei. Mijlocul ipotenuzei este cen-
trul cercului circumscris triunghiului dreptunghic. Tri-
unghiul dreptunghic ABC este înscris în cercul de centru
O şi raza de 6 unităţi, diametrul de 12 şi cateta AB de 5.

58
Cap 3. Triunghiul dreptunghic. Relaţii metrice
Un triunghi dreptunghic poate fi înscris într-un se-
A
micerc cu diametrul egal cu lungimea ipotenuzei.
3. Triunghiul dreptunghic ABC este înscris într-un
semicerc cu diametrul BC = 15 şi înălţimea AD = 6.
Din desen se constată că BD = 3 şi DC = 12.
B C
Rezultă că AD = 6 este media geometrică a lun- D O
gimilor segmentelor BD şi DC, proiecţiile catetelor pe ipotenuză, 62 = 3 ∙ 12.
Demonstraţie. ∆ABD ~ ∆CAD, deoarece sînt dreptunghice şi BAD ≡ ACD.
BD AD
Rezultă = , de unde AD2 = BD ∙ DC.
AD DC
Teorema înălţimii. Înălţimea unui triunghi dreptunghic este media geometrică a
lungimilor proiecţiilor catetelor pe ipotenuza triunghiului.
Triunghiul ABC are unghiul drept A şi înălţimea AD. Atunci: AD 2 = BD ∙ DC.
A
BD AD
În desen B–D–C, D = prBCA, (1). Dacă AD2 = BD ∙ DC, atunci = .
AD DC
Criteriul de asemănare LUL implică ∆ABD ~ ∆CAD, de unde
B D C BAD ≡ ACD, (2). (1) şi (2) implică: triunghiul ABC are un-
ghiul drept A.
Construcţia mediei geometrice
A
Pentru a desena un segment de lungime 2 5 , se construiesc:
1) segmentul BC = 10;
2) semicercul cu diametrul BC;
3) perpendiculara în D pe BC, unde BD = 2; B O C
D
4) intersecţia perpendicularei cu semicercul (punctul A);
5) unghiul BAC.
Conform teoremei înălţimii, segmentul AD are lungimea 2 5.

Exerciţii
1. Completaţi propoziţia:
Media proporţională sau geometrică a numerelor nenegative a şi b este numărul

2. Scrieţi media geometrică a numerelor: a) 44 şi 11; b) 12 şi 2; c) 18 şi 8.

3. Construiţi triunghiul dreptunghic ABC:


a) mediana corespunzătoare ipotenu­zei AD = 4 şi cateta AB = 3;
b) mediana corespunzătoare ipotenu­zei AD = 6 şi cateta AC = 4;
c) mediana corespunzătoare ipotenu­zei AD = 7 şi cateta AB = 5.

59
4. Completaţi propoziţia:
Un triunghi dreptunghic este înscris într-un semicerc cu

5. Recunoaşteţi proiecţiile catetelor pe ipotenuză:


F J

a) b)

G I H K M L

N
c)

O Q P

6. Recunoaşteţi triunghiurile asemenea:


a) cu latura comună FI; F

G I H
b) cu latura comună JM; J

K M L

c) cu latura comună NQ.


N

O Q P
7. Completaţi propoziţiile:
a) Înălţimea unui triunghi dreptunghic este
b) Triunghiul ABC are unghiul drept A şi înălţimea AD. Atunci:

60
Cap 3. Triunghiul dreptunghic. Relaţii metrice
8. La informaţiile din desen adăugaţi AB = c,BC = a, AC = b. A
Completaţi tabelul: a m n h
h
12 4
– n m
3 B C
√15 D
17 5

7 √35

9. Aplicînd teorema înălţimii, construiţi un segment de lungime:


a) 15; b) 10; c) 14; d) 17.

61
3 Reciproca teoremei înălţimii

Triunghiul ABC are înălţimea AD de 4, iar proiecţiile laturilor AB şi AC pe BC au


lungimile 2 şi 8. Ce tip de triunghi este triunghiul ABC?
Cu ajutorul echerului se constată că: triunghiul ABC este dreptunghic; lungimea
segmentului AD este media geometrică a lungimilor segmentelor BD şi DC.
Reciproca teorema înălţimii. Dacă AD este înălţime a triunghiului ABC, B–D–C şi
AD2 = BD ∙ DC, atunci unghiul BAC este drept.
Ipoteza. Desenul, B–D–C, A
D = prBC A şi AD2 = BD · DC.
Concluzia. Unghiul BAC este drept.
AD BD
Demonstraţie. AD2 = BD · DC implică = , (1). B D O C
CD AD
D = prBC A şi (1) implică (crit. LUL) ∆ABD ~ ∆CAD, (2). (2) implică ÐBAD ≡ ÐACD
şi ÐABD ≡ ÐCAD, (3). (3) implică: unghiul BAC este drept, q.e.d.
Teorema înălţimii completă. Fie triunghiul ABC cu înălţimea AD, B–D–C. Unghiul
BAC este drept dacă şi numai dacă AD2 = BD · DC.
Corolar. Fie triunghiul ABC cu înălţimea AD, B–D–C. Triunghiul ABC este:
a) dreptunghic cu unghiul drept A, dacă şi numai dacă AD2 = BD · DC;
b) ascuţitunghic, dacă şi numai dacă AD2 > BD · DC;
c) obtuzunghic, dacă şi numai dacă AD2 < BD · DC.

62
Cap 3. Triunghiul dreptunghic. Relaţii metrice
Exerciţii
1. Completaţi propoziţiile:
a) Dacă AD este înălţime a triunghiu­lui ABC, B–D–C şi AD2 =

b) Triunghiul ABC cu înălţimea AD, B–D–C şi AD2 = Atunc

2. Completaţi propoziţia:
Triunghiul ABC cu înălţimea AD, B–D–C, dacă
şi numai dacă

3. Construiţi triunghiul ABC (m( A) = 90°) cu înălţimea AD, B–D–C, ştiind că:
– – –
a) BD = 3 şi AD = √33 ; b) DC = 8 şi AD = √24 ; c) BD = 10 şi AD = √30 .

4. Completaţi propoziţiile:
Fie triunghiul ABC cu înălţimea AD şi B–D–C. Triunghiul ABC este:
a) dreptunghic cu unghiul drept A, dacă şi numai dacă
b) ascuţitunghic, dacă şi numai dacă
c) obtuzunghic, dacă şi numai dacă
5. Fie triunghiul ABC cu înălţimea AD şi B–D–C. Aflaţi AD, astfel încît unghiul BAC
să fie drept, dacă:
a) BD = 8 cm şi DC = 5 cm; b) BD = 3 cm şi DC = 11 cm;
c) BD = 7 cm şi DC = 9 cm; d) BD = 21 cm şi DC = 3 cm.

63
6. Fie triunghiul ABC cu înălţimea AD şi B–D–C. Aflaţi dacă triunghiul are unghiul
BAC drept, ştiind că:
a) AD = 7 cm, BD = 1 cm şi DC = 49 cm; b) AD = 6 cm, BD = 3 cm şi DC = 12 cm;
c) AD = 12 cm, BD = 36 cm şi DC = 4 cm; d) AD = 8 cm, BD = 4 cm şi DC = 16 cm.

7. Fie triunghiul ABC cu înălţimea AD şi B–D–C. Aflaţi ce tip de triunghi este ABC,
dacă:
a) AD = 5 cm, BD = 3 cm şi DC = 8 cm; b) AD = 6 cm, BD = 6 cm şi DC = 5 cm;
c) AD = 13 cm, BD = 12 cm şi DC = 11 cm; d) AD = 11 cm, BD = 9 cm şi DC = 12 cm.

8. Fie triunghiul ABC cu înălţimea AD şi B–D–C. Aflaţi ce tip de triunghi este ABC,
dacă:
a) AD = 12 cm, BD = 11 cm şi DC = 13 cm;

64
Cap 3. Triunghiul dreptunghic. Relaţii metrice
b) AD = 15 cm, BD = 9 cm şi DC = 26 cm;
c) AD = 11 cm, BD = 13 cm şi DC = 12 cm;
d) AD = 14 cm, BD = 15 cm şi DC = 13 cm.

4 Teorema catetei
1. Triunghiul dreptunghic ABC are ipo- A
tenuza BC de 12, cateta AB de 6 şi D =
prBC A. Se constată că BD = prBCAB = 3.
Deoarece 62 = 3 ∙ 12, lungimea catetei
6

AB este media geometrică a lungimii


ipotenuzei şi a lungimii proiecţiei cate-
B 3 9 C
tei AB pe ipotenuză. Deoarece triun- D O
ghiurile dreptunghice ABC şi DBA au
unghiul ascuţit B comun, conform criteriului de asemănare UU, ∆ABC ~ ∆DBA.
AB BC
Rezultă = , de unde AB2 = BD ∙ BC. În acelaşi mod se demonstrează că
BD AB
AC 2 = DC ∙ BC.
Teorema catetei. Lungimea fiecărei catete a unui triunghi dreptunghic este media
geometrică a lungimii ipotenuzei triunghiului şi a proiecţiei catetei pe ipotenuză.
Ipoteza. Desenul, unghiul BAC este drept, D = prBC A.
Concluzia. AB2 = BD ∙ BC şi AC 2 = DC ∙ BC.
Demonstraţie. Deoarece triunghiurile A
dreptunghice ABC şi DBA au unghiul
ascuţit B comun, conform criteriului
de asemănare UU, ∆ABC ~ ∆DBA.
BD AB
Rezultă = , de unde AB2 =
AB BC
BD ∙ BC. În acelaşi mod se demon-
B C
strează că AC 2 = DC ∙ BC, q.e.d. D

65
2. Aplicînd teorema catetei, se poate construi un
– A
segment de lungime √14 ca în desen.
Se construieşte:

√ 1–
4
1) un semicerc cu diametrul 7;
2) BD = 2 şi B–D–C;
2 5
3) perpendiculara în D pe BC şi se notează C
B D
cu A intersecţia ei cu semicercul.

Conform teoremei catetei, AB = √14 .

Exerciţii
1. Completaţi propoziţiile:
a) Lungimea fiecărei catete a unui triunghi dreptunghic

b) Triunghiul ABC are unghiul drept A şi înălţimea AD. Atunci:

2. Recunoaşteţi triunghiurile asemenea:


a) cu unghiul comun G F

G I H
b) cu unghiul comun K J

K M L
c) cu unghiul comun O
N

O Q P
3. Recunoaşteţi triunghiurile asemenea:
a) cu unghiul comun H
F

G I H

66
Cap 3. Triunghiul dreptunghic. Relaţii metrice
b) cu unghiul comun L J

K M L
c) cu unghiul comun P
N

O Q P
4. Completaţi raţionamentul:
GF 2
a) ∆IGF ~ ∆FGH. Rezultă = de unde =
GH
JK 2
b) ∆JKL ~ ∆MKJ. Rezultă = de unde =
MK
NO 2
c) ∆NOP ~ ∆QON. Rezultă =. de unde =
QO
FH 2
d) ∆FGH ~ ∆IFH. Rezultă = de unde =
IH
5. La informaţiile din desen adăugaţi AB = c, BC = a, AC = b. A
Completaţi tabelul: a b c m n h
h
12 4
n m
5 3 B D C
20 16
38 5

6. Aplicînd teorema catetei, construiţi un segment de lungime:


– – – –
a) √20 ; b) √24 ; c) √28 ; d) √35 .

67
7. Construiţi triunghiul ABC (m( A) = 90°) cu înălţimea AD, B–D–C, ştiind că:
– – –
a) BD = 3 şi AB = √15 ; b) DC = 2 şi AC = √14 ; c) BC = 5 şi AD = √35 .

5 Reciproca teoremei catetei

A
13

12

5 28,8
B D O C
Triunghiul ABC are latura BC de 33,8, latura AB de 13 şi proiecţia laturii AB pe BC
de 5, D se află între B şi C. Ce tip de triunghi este triunghiul ABC?

68
Cap 3. Triunghiul dreptunghic. Relaţii metrice
Cu ajutorul echerului se constată că: triunghiul ABC este dreptunghic; lungimea
segmentului AB este media geometrică a lungimilor segmentelor BD şi BC.
Reciproca teoremei catetei. Dacă AD este înălţime a triunghiului ABC, B–D–C şi
AB2 = BD ∙ BC sau AC 2 = DC ∙ BC, atunci unghiul BAC este drept.
Ipoteza. Desenul, B–D–C, D = prBC A şi AB2 = BC · BD. A
Concluzia. Unghiul BAC este drept.
BD AB
Demonstraţie. AB2 = BC · BD implică = , (1).
AB BC
D = prBC A şi (1) implică (conform LUL) B
D O C

∆ABC ~ ∆DBA, (2). (2) implică ÐBAD ≡ ÐACD, (3).


Teorema catetei completă. Fie AD o înălţime a triunghiului ABC, B–D–C. Un­ghiul
BAC este drept dacă şi numai dacă AB2 = BC · BD sau AC2 = BC · CD.
Corolar. Fie triunghiul ABC cu înălţimea AD, B–D–C. Triunghiul ABC este:
a) dreptunghic cu unghiul drept A, dacă şi numai dacă AB2 = BC · BD sau AC2 =
BC · CD;
b) ascuţitunghic, dacă şi numai dacă AB2 > BC · BD sau AC2 > BC · CD;
c) obtuzunghic, dacă şi numai dacă AB2 < BC · BD sau AC2 < BC · CD.

Exerciţii
1. Completaţi propoziţiile:
Dacă AD este înălţime a triunghiului ABC, B–D–C şi AB2 =
2. Completaţi reciproca teoremei catetei (echivalenţa):
Fie AD o înălţime a triunghiului ABC, B–D–C. Un­ghiul BAC este drept dacă şi

numai dacă
3. Completaţi:
Fie triunghiul ABC cu înălţimea AD, B–D–C. Triunghiul ABC este:
a) dreptunghic cu unghiul drept A, dacă şi numai dacă
b) ascuţitunghic, dacă şi numai dacă
c) obtuzunghic, dacă şi numai dacă
4. Fie triunghiul ABC cu înălţimea AD, B–D–C. Aflaţi AB şi AC, astfel încît unghiul
BAC să fie drept, dacă:

69
a) BD = 9 cm şi DC = 5 cm; b) BD = 2 cm şi DC = 13 cm;
c) BD = 5 cm şi DC = 13 cm; d) BD = 14 cm şi DC = 7 cm.

5. Fie triunghiul ABC cu înălţimea AD, B–D–C. Aflaţi ce tip de triunghi este ABC,
dacă:
a) AB = 14 cm, BD = 7 cm şi DC = 21 cm;
b) AB = 6 cm, BD = 3 cm şi DC = 12 cm;
c) AC = 12 cm, BD = 18 cm şi DC = 6 cm;
d) AC = 15 cm, BD = 40 cm şi DC = 5 cm.

6. Fie triunghiul ABC cu înălţimea AD, B–D–C. Aflaţi ce tip de triunghi este ABC,
dacă:
a) AB = 5 cm, BD = 3 cm şi DC = 8 cm;
b) AB = 6 cm, BD = 6 cm şi DC = 5 cm;
c) AC = 13 cm, BD = 16 cm şi DC = 11 cm;
d) AC = 11 cm, BD = 9 cm şi DC = 12 cm.

7. Fie triunghiul ABC cu înălţimea AD, B–D–C. Aflaţi ce tip de triunghi este ABC,
dacă:
a) AB = 15 cm, BD = 11 cm şi DC = 7 cm;
b) AB = 17 cm, BD = 9 cm şi DC = 21 cm;
c) AC = 21 cm, BD = 11 cm şi DC = 12 cm;
d) AC = 19 cm, BD = 12 cm şi DC = 13 cm.

70
Cap 3. Triunghiul dreptunghic. Relaţii metrice
6 Teorema lui Pitagora
1. Din desen se constată că triunghiul dreptunghic ABC
cu AB = 4 şi AC = 3 are ipotenuza BC = 5. Evident, C
32 + 42 = 52. Fie m şi n lungimile proiecţiilor catetelor
3 5
pe ipotenuză. Teorema catetei implică: AB2 = m ∙ BC şi
5
AC 2 = n ∙ BC. Rezultă: AB2 + AC 2 = BC(m + n) = BC 2. A 4 B
Dacă un triunghi MNP are laturile respectiv de 3, 4 şi
5, atunci, conform criteriului de congruenţă LLL, ∆CAB ≡ ∆MNP. Prin urmare, un­
ghiul N este drept.
Teorema lui Pitagora. Dacă un triunghi este dreptunghic, atunci pătratul lungimii
ipotenuzei este egal cu suma pătratelor lungimilor catetelor triunghiului.
Triunghiul ABC are unghiul drept A. Atunci: BC 2 = AB2 + AC 2.
Ipoteza. Desenul, unghiul BAC este drept. A
Concluzia. BC 2 = AB2 + AC 2.
Demonstraţie. Fie D = prBC A.
Teorema catetei implică:
B O C
AB2 = BD ∙ BC şi AC 2 = DC ∙ BC. Rezultă: D
AB2 + AC 2 = BC(BD + DC) = BC 2.
Prin urmare, AB2 + AC 2 = BC 2, q.e.d.
4
5
2. În desenul alăturat, pe laturile triunghiului s-au construit
pătrate exterioare triunghiului. Conform desenului, su- C 3 2
prafaţa pătratului corespunzător ipotenuzei poate fi 5
1

acoperită cu suprafeţele pătratelor corespunzătoare 4 1


catetelor triunghiului dreptunghic.
A B
3. Pe reţeaua de pătrate din dreapta, DEGF este pătrat cu lun- 2
gimile laturilor b + c. Dreptunghiurile CDHJ, HEIK, IFBL, 1
ABGC au dimensiunile b şi c. Din congruenţa triunghiurilor 3
dreptunghice în care se descompun dreptunghiurile enu-

71
merate, rezultă că BCHI este pătrat cu laturile de lungimi a. D b C c G
Din desen rezultă că aria DEFG = aria BCHI + 2 aria a
c
CDHJ. Rezultă că (b + c)2 = a2 + 2bc, de unde b2 + c2 = a2. K J a b
H b
Am descoperit astfel relaţiile dintre lungimile laturilor
triunghiului dreptunghic ABC. c
b a L B
– A
4. Pentru a construi un segment de lungime √ 5 este suficient a c
√ 5– să construim un triunghi dreptunghic cu ca-
tetele de 1 şi 2. Conform teoremei lui Pi- E c I b F
1

2 tagora, ipotenuza triunghiului are lungimea √ 5 .

Exerciţii
1. Completaţi propoziţiile:
a) Dacă un triunghi este dreptunghic, atunci pătratul lungimii ipotenuzei

b) Triunghiul ABC are unghiul drept A şi înălţimea AD. Atunci:

2. Completaţi raţionamentul conform desenului:


a) Teorema catetei implică: F
GF2 = GI ∙ şi HF 2 =
Rezultă GF2 + HF 2 = ( + )= 2
.
G I H
b) Teorema catetei implică: J
JK 2 = KM ∙ şi JL 2 =
Rezultă JK 2 + JL 2 = ( + )= 2
.
K M L
c) Teorema catetei implică:
N
ON 2 = OQ ∙ şi NP 2 =
Rezultă ON 2 + NP 2 = ( + )= 2
.
O Q P
3. Aflaţi lungimea ipotenuzei unui triunghi dreptunghic avînd catetele de lungimi:
a) 3 şi 7; b) 8 şi 3; c) 9 şi 11; d) 5 şi 6.

72
Cap 3. Triunghiul dreptunghic. Relaţii metrice
4. Aflaţi lungimea unei catete a unui tri­unghi dreptunghic, dacă lungimea ipote­nuzei şi
a celeilalte catete sînt respectiv de:
a) 15 cm şi 12 cm; b) 13 cm şi 12 cm; c) 41 cm şi 9 cm; d) 17 cm şi 15 cm.

5. Aplicînd teorema lui Pitagora, con­stru­iţi un segment de lungime:


– – – –
a) √15 ; b) √17 ; c) √29 ; d) √53 .

6. La informaţiile din desen adăugaţi AB = c, BC = a, AC = b. A


Completaţi tabelul: a b c m n h
25 7 h
8 15 n m
B D C
41 9
11 60

7. Triunghiul ABC are m( A) = 90° şi m( C) = 30°. Aflaţi lungimile celorlalte


laturi dacă: a) BC = 12; b) AB = 19; c) BC = 18.

73
8. Aflaţi lungimea înălţimilor triunghiului echilateral ABC cu laturile de:
a) 8; b) 15; c) 20; d) 28.

9. Triunghiul isoscel ABC are m( A) = 90°. Aflaţi lungimea ipotenuzei, dacă


catetele au lungimea: a) 15; b) 21; c) 35; d) 41.

10. Triunghiul isoscel ABC are m( A) = 90°. Aflaţi lungimile catetelor, dacă ipo­
tenuza este de: a) 10; b) 24; c) 34; d) 36.

11. Aflaţi lungimile diagonalelor unui pă­trat cu laturile de:


a) 15; b) 26; c) 43; d) 71.

12. Aflaţi lungimile diagonalelor unui drep­tunghi cu laturile de:


a) 5 şi 7; b) 8 şi 11; c) 5 şi 13; d) 8 şi 9.

74
Cap 3. Triunghiul dreptunghic. Relaţii metrice
13. Aflaţi lungimile laturilor unui romb cu diagonalele de:
a) 8 şi 10; b) 6 şi 10; c) 12 şi 18.

7 Reciproca teoremei lui Pitagora


A

12
5

13
B C
Triunghiul ABC are latura BC de 13, latura AB de 12 şi latura AC de 5. Cu ajutorul
echerului se constată că: triunghiul ABC este dreptunghic.
Reciproca teoremei lui Pitagora. Dacă un triunghi are pătratul lungimii unei laturi
egal cu suma pătratelor lungimilor celorlalte laturi ale triunghiului, atunci triun­
ghiul este dreptunghic.
Ipoteza. Desenul, a2 = b2 + c2. A

Concluzia. Unghiul BAC este drept. c b


Demonstraţie. Fie triunghiul MNP cu
a
unghiul drept M, MN = a, MP = b, (1). B C
M
Conform teoremei lui Pitagora, NP2 = b2 + c2, (2).
(1) şi (2) implică, conform criteriului LLL, b
c
∆ABC ≡ ∆DBA, (3). (3) implică:
unghiul BAC este drept, q.e.d. a
N P

75
Teorema lui Pitagora completă. Un triunghi este dreptunghic dacă şi numai dacă
are pătratul lungimii unei laturi egal cu suma pătratelor lungimilor celorlalte laturi
ale triunghiului.
Corolar. Trunghiul ABC este:
a) dreptunghic cu unghiul drept A, dacă şi numai dacă BC2 = AB2 + AC2;
b) ascuţitunghic, dacă şi numai dacă BC2 > AB2 + AC2;
c) obtuzunghic, dacă şi numai dacă BC2 < AB2 + AC2.

Exerciţii
1. Completaţi propoziţia:
Dacă un triunghi are pătratul lungimii unei laturi egal cu

2. Completaţi propoziţia:
Un triunghi este dreptunghic dacă şi numai dacă

3. Un triunghi are lungimile laturilor a, b, c. Completaţi:


a b c Triunghiul este dreptunghic?

11 7 √17

13 9 √88

21 20 √41

26 25 √52

4. Completaţi propoziţia:
Triunghiul ABC este:
a) dreptunghic cu unghiul drept A, dacă şi numai dacă BC2

b) ascuţitunghic, dacă şi numai dacă BC2

c) obtuzunghic, dacă şi numai dacă BC2

76
Cap 3. Triunghiul dreptunghic. Relaţii metrice
5. Fie triunghiul ABC. Aflaţi AB, astfel încît unghiul BAC să fie drept, dacă:
a) BC = 16 cm şi AC = 7 cm; b) BC = 15 cm şi AC = 9 cm;
c) BC = 23 cm şi AC = 11 cm; d) BC = 19 cm şi AC = 12 cm.

6. Fie triunghiul ABC. Aflaţi dacă triunghiul este dreptunghic, ştiind că:
a) AB = 7 cm, BC = 24 cm şi AC = 25 cm;
b) AB = 8 cm, BC = 17 cm şi AC = 15 cm;
c) AB = 13 cm, BC = 84 cm şi AC = 85 cm;
d) AB = 29 cm, BC = 20 cm şi AC = 21 cm.

7. Fie triunghiul ABC. Aflaţi ce tip de triunghi este ABC, dacă:


a) AB = 28 cm, BC = 45 cm şi AC = 52 cm;
b) AB = 35 cm, BC = 64 cm şi AC = 63 cm;
c) AB = 11 cm, BC = 40 cm şi AC = 41 cm;
d) AB = 33 cm, BC = 56 cm şi AC = 64 cm.

77
8. Fie triunghiul ABC. Aflaţi ce tip de triunghi este ABC, dacă:
a) AB = 28 cm, BC = 45 cm şi AC = 52 cm;
b) AB = 35 cm, BC = 64 cm şi AC = 63 cm;
c) AB = 11 cm, BC = 40 cm şi AC = 41 cm;
d) AB = 33 cm, BC = 56 cm şi AC = 64 cm.

9. Fie triunghiul ABC. Aflaţi ce tip de triunghi este ABC, dacă:


a) AB = 36 cm, BC = 77 cm şi AC = 86 cm;
b) AB = 80 cm, BC = 90 cm şi AC = 39 cm;
c) AB = 65 cm, BC = 98 cm şi AC = 72 cm;
d) AB = 24 cm, BC = 75 cm şi AC = 70 cm.

Exerciţii recapitulative

1. Construiţi triunghiul dreptunghic ABC:


a) mediana corespunzătoare ipote­nu­zei AD = 3 şi cateta AB = 4;
b) mediana corespunzătoare ipotenu­zei AD = 7 şi cateta AC = 9;
c) media­na corespunzătoare ipotenu­zei AD = 6 şi cateta AB = 7.

78
Cap 3. Triunghiul dreptunghic. Relaţii metrice
2. Fie desenul şi AB = c, BC = a, a m n h A
AC = b. Completaţi tabelul:
16 5 h

7 √77 n m
B D C
24 9

13 √91

3. Construiţi triunghiul ABC (m( A) = 90°) are înălţimea AD, ştiind că:
– – –
a) BD = 5 şi AD = √55 ; b) DC = 17 şi AD = √51 ; c) BD = 11 şi AD = √66 .

79
4. Fie desenul şi AB = c, a b c m n h A
BC = a, AC = b. Com- 22 5
pletaţi tabelul: 18 6 h
14 7 n m
B D C
26 8

5. Aplicînd teorema înălţimii, construiţi un segment de lungime:


– – – –
a) √33 ; b) √38 ; c) √42 ; d) √51 .

6. Aplicînd teorema catetei, construiţi un segment de lungime:


– – – –
a) √14 ; b) √28 ; c) √44 ; d) √28 .

80
Cap 3. Triunghiul dreptunghic. Relaţii metrice
7. Construiţi triunghiul ABC (m( A) = 90°) are înălţimea AD, B–D–C, ştiind că:
– – –
a) BD = 7 şi AB = √56 ; b) DC = 11 şi AC = √55 ; c) BC = 13 şi AD = √39 .

8. Lungimile ipotenuzei şi ale catetelor unui triunghi dreptun- a b c


ghic sînt respectiv a, b, c. Completaţi tabelul.
13 5
41 40
20 21
24 10

9. Lungimile laturilor triunghiului ABC sînt m( A) m( B) a b c


AB = c, BC = a, AC = b. Completaţi tabelul.
90° 30° 24

90° 60° 7

90° 30° 23

90° 60° 38

81
10. Aflaţi înălţimile triunghiului echilateral cu laturile de lungime:
a) 24; b) 52; c) 62; d) 38.

11. Aflaţi lungimile diagonalelor unui drep­tu­ nghi cu laturile de:


a) 15 şi 11; b) 13 şi 18; c) 14 şi 19.

12. Un triunghi are lungimile laturilor a, b, c. Completaţi:


a b c Triunghiul este dreptunghic?

12 7 √95

20 18 √76

22 20 √87

31 30 √61
13. Aflaţi lungimile laturilor unui romb cu diagonalele de:
a) 12 şi 22; b) 14 şi 32; c) 26 şi 42.

82
Cap 3. Triunghiul dreptunghic. Relaţii metrice
14. Aflaţi înălţimile paralelogramului ABCD şi completaţi D C
tabelul: h1 h2
m( A) h1 h2 AB BC
A B
30° 26 44

60° 36 16

45° 76 54

30° 32 52

16. Numerele 3, 4, 5 formează un triplet pitagoreic, deoarece 32 + 42 = 52. Completaţi


tripletele pitagoreice:
a) 5, ..., 13; b) ..., 15, 17; c) 7, 24, ...; d) 14, 48, ...; e) ..., 45, 53; f) 9, ..., 41.

15. Triunghiul isoscel ABC are baza BC şi înălţimea AD.


Completaţi tabelul: AB BC AD

11 14

15 8

17 12

19 16

83
Evaluare
I II
1. Aflaţi înălţimea triunghiului dreptun- 1. Aflaţi înălţimea triunghiului dreptun-
ghic cu proiecţiile catetelor pe ipotenuză ghic cu proiecţiile catetelor pe ipotenuză
20
de: de:
a) 12 şi 3; b) 15 şi 6. a) 13 şi 4; b) 17 şi 4.
2. Aflaţi lungimile catetelor unui triunghi 2. Aflaţi lungimile catetelor unui triunghi
dreptunghic, dacă ipotenuza lui este de dreptunghic, dacă ipotenuza lui este de
14 şi proiecţia unei catete pe ea este de 5. 10 15 şi proiecţia unei catete pe ea este de 3.
3. Triunghiul ABC (m( A) = 90°) are 3. Triunghiul ABC (m( A) = 90°) are
AB = c, BC = a, AC = b. Aflaţi: AB = c, BC = a, AC = b. Aflaţi:
10
a) a, dacă b = 24 şi c = 26; a) a, dacă b = 16 şi c = 30;
b) c, dacă a = 53 şi b = 45. b) c, dacă a = 41 şi b = 9.
4. Construiţi un segment cu lungimea de 4. Construiţi un segment cu lungimea de
– –
√57 , aplicînd teorema: 10 √58 , aplicînd teorema:
a) înălţimii; b) catetei. a) înălţimii; b) catetei.
5. Aplicînd teorema lui Pitagora, con- 5. Aplicînd teorema lui Pitagora, con-
– 10 –
struiţi un segment de lungime √45 . struiţi un segment de lungime √21 .
6. Decideţi dacă este dreptunghic un tri- 6. Decideţi dacă este dreptunghic un tri-
unghi cu laturile– de: – 10 unghi cu laturile– de: –
a) 13, 17 şi √458; b) 9, 21, √532. a) 15, 17 şi √524; b) 8, 23, √593.
7. Aflaţi lungimile diagonalelor: 7. Aflaţi lungimile diagonalelor:
a) unui pătrat cu laturile de 26; 10 a) unui pătrat cu laturile de 29;
b) unui dreptunghi cu laturile de 13 şi 24. b) unui dreptunghi cu laturile de 14 şi 23.
8. Aflaţi lungimile laturilor unui romb 8. Aflaţi lungimile laturilor unui romb
10
cu diagonalele de 38 şi 52. cu diagonalele de 42 şi 54.
9. Aflaţi înălţimile D C 9. Aflaţi înălţimile D C
paralelogramului h1 h 2 paralelogramului h1 h 2
10
ABCD cu AB = 8, ABCD cu AB = 9,
A B A B
BC = 13, dacă: BC = 14, dacă:
a) m( A) = 45°; b) m( A) = 60°. a) m( A) = 45°; b) m( A) = 60°.

8 Elemente de trigonometrie
AB BC B
1) În desen ∆ABC ~ ∆DEC. Rezultă = , de unde
DE CE
AB DE E
= . Rezultatul depinde numai de mărimea unghiului.
BC CE
AB AC C
2) Din ∆ABC ~ ∆DEC rezultă şi = , de unde D A
DE DC
AB DE
= . Rezultatul depinde numai de mărimea unghiului.
AC DC

84
Cap 3. Triunghiul dreptunghic. Relaţii metrice
Triunghiul ABC este dreptunghic în B, m(ÐA) = α, m(ÐC) = β.
cateta opusă a C
Sinusul unghiului A este sin A = sin α = = .
ipotenuză b
cateta alăturată c β b
Cosinusul unghiului A este cos A = cos α = = . a
ipotenuză b α
cateta opusă a B c A
Tangenta unghiului A este tg A = tg α = = .
cateta alăturată c
cateta alăturată c
Cotangenta unghiului A este ctg A = ctg α = = .
cateta opusă a
α B
3) Lungimea cercului de rază R este egală cu 2πR. Dacă considerăm
cercul de rază 1 (o unitate), atunci lungimea cercului este 2π, lungi- A α
mea semicercului este π. Măsura unghiului la centru AOB este egală O
)
cu măsura arcului mic, m( AOB) = m(AB) = α.
Măsurarea în radiani. Fie cercul de rază 1. Lungimea cercului este 2π; lungimea
π
semicercului este π; lungimea unui arc de 90° este 0,5π = . Măsura în radiani a
2
unui unghi la centru este egală cu lungimea arcului de cerc unitate (cu raza 1),
πn°
corespunzător. Conversia din grade în radiani se face aplicînd formula: α = .
180°
grade 0° 30° 45° 60° 90° 120° 135° 150° 180°
1 1 1 1 1 1 1
radiani 0 − − − π
2 2 2 2 2 2 2

Exerciţii
1. Completaţi raţionamentul:
BC
a) ∆ABC ~ ∆ADE implică = C
DE
E
BC
de unde =
AC
A
D B

b) ∆KLM ~ ∆KNP
M
LM
implică =
NP P

LM
de unde = K
KM N L

85
2. Fie triunghiul DFG. Completaţi definiţiile: G

cateta opusă
a) sin F = = D F
ipotenuză
cateta alăturată
b) cos F = =
ipotenuză
cateta opusă
c) sin G = =
ipotenuză
cateta alăturată
d) cos G = =
ipotenuză
3. Fie triunghiul KLM. Completaţi definiţiile: M

a) tg L = b) ctg L =
c) tg M = d) ctg M = K L

4. Fie triunghiul dreptunghic MNP. Completaţi tabelul: P

MN NP MP sin N cos N tg N ctg N


M N
3 7

13 5

15 10

5 21

5. Fie triunghiul dreptunghic ABC. Completaţi tabelul: C

AB BC AC sin B cos C tg C ctg B


A B
5 10
11 11
14 7
17 17

86
Cap 3. Triunghiul dreptunghic. Relaţii metrice
6. Completaţi propoziţiile:
a) Lungimea cercului de rază R este
b) Lungimea cercului de rază 1 este
c) Lungimea semicercului de rază 1 este
d) Formula de conversie în radiani este
7. Completaţi tabelul: Grade 30° 45° 60° 90° 120°
Radiani

9 Valorile funcţiilor trigonometrice


1) sin 30°, cos 30°, tg 30°, ctg 30° B
60°
2x
x

30°
C A
x 3
2) sin 45°, cos 45°, tg 45°, ctg 45° B

x 2 45°
x

45°
C x A

87
Exerciţii
1. Fie triunghiul dreptunghic ABC, m( B) = 30°, AC = 5. Aflaţi: C
a) sin B; b) cos B; c) tg B; d) ctg B.
A B

2. Fie triunghiul dreptunghicABC, m( B) = 60°, BC = 12. Aflaţi: C


a) sin B; b) cos B; c) tg B; d) ctg B.
A B

3. Fie triunghiul dreptunghic ABC, m( A) = 90°. Aflaţi lungimile celorlalte laturi,


dacă: a) AC = 4 cm, sin B = 0,3; b) AB = 8 cm şi sin C = 0,4;
c) AC = 12 cm, sin B = 0,7; d) AB = 15 cm şi sin C = 0,8.

4. Fie triunghiul dreptunghic ABC, m( A) = 90°. Aflaţi lungimile celorlalte laturi,


dacă: a) AC = 16 cm, cos C = 0,6; b) AB = 24 cm şi cos B = 0,7;
c) AC = 14 cm, cos C = 0,9; d) AB = 28 cm şi cos B = 0,3.

88
Cap 3. Triunghiul dreptunghic. Relaţii metrice
5. Fie triunghiul dreptunghic ABC, m( A) = 90°. Aflaţi lungimile celorlalte laturi,
dacă:
a) AC = 13 cm, tg B = 5; b) AB = 21 cm şi tg C = 2;
c) AC = 12 cm, tg B = 3; d) AB = 26 cm şi tg C = 4.

6. Fie triunghiul dreptunghic ABC, m( A) = 90°. Aflaţi lungimile celorlalte laturi,


dacă:
a) AC = 31 cm, ctg C = 3; b) AB = 19 cm şi ctg B = 5;
c) AC = 14 cm, ctg C = 8; d) AB = 33 cm şi ctg B = 4.

7. Fie triunghiul dreptunghic ABC, m( A) = 90°. Aflaţi:


a) cos B şi tg B, dacă AB = 11 cm şi sin C = 0,1;
b) cos C şi tg C, dacă AC = 14 cm şi sin B = 0,7;
c) cos B şi tg B, dacă AB = 18 cm şi sin C = 0,4;
d) cos C şi tg C, dacă AC = 25 cm şi sin B = 0,8.

89
8. Fie triunghiul dreptunghic ABC, m( A) = 90°. Aflaţi:
a) sin B şi tg B, dacă AB = 20 cm şi tg C = 5;
b) sin C şi tg C, dacă AC = 30 cm şi tg B = 6;
c) sin B şi tg B, dacă AB = 42 cm şi tg C = 7;
d) sin C şi tg C, dacă AB = 54 cm şi tg B = 9.

Exerciţii recapitulative
1. Fie triunghiul DEF. Completaţi definiţiile: F

a) sin E = cos F = b) sin F = cos E =


D E
c) tg E = ctg F = d) tg F = ctg E =

2. Fie triunghiul dreptunghic ABC. Completaţi tabelul: C

AB BC AC sin B cos C tg C ctg B B


A
11 22

9 9

36 18

25 25

90
Cap 3. Triunghiul dreptunghic. Relaţii metrice
3. Completaţi tabelul: Grade 15° 80° 125° 150° 160°

Radiani

4. Fie triunghiul dreptunghic ABC, m( B) = 30°, AC = 7. Aflaţi: C


a) sin B; b) cos B; c) tg B; d) ctg B.
A B

5. Fie triunghiul dreptunghic ABC, m( B) = 60°, BC = 24. Aflaţi: C


a) sin B; b) cos B; c) tg B; d) ctg B.
A B

6. Efectuaţi:
π π π π π π π π
a) 2 sin + cos ; b) sin + 2 cos ; c) sin – 3 cos ; d) 3 sin – cos .
4 3 6 4 3 6 4 3

91
7. Efectuaţi:
π π π π π π π π
a) 2 tg – ctg ; b) ctg + 3 tg ; c) 6 tg – ctg ; d) 3ctg – 2 ctg .
3 3 6 4 6 4 3 4

π π π π
8. Efectuaţi: a) 6 tg 6 – ctg 4; b) 3 ctg 3 – 2 ctg 4.

9. Fie triunghiul dreptunghic ABC. Completaţi tabelul: C

AB AC BC sin B cos B sin C cos C A B

52 0,5

26 0,2

11 0,6

14 0,4

92
Cap 3. Triunghiul dreptunghic. Relaţii metrice
10. Fie triunghiul dreptunghic ABC. Completaţi tabelul: C

AB AC BC tg B ctg B tg C ctg C A B

12 2

16 4

7 3

15 6

11. Aflaţi măsurile unghiurilor ascuţite B şi C ale triunghiului dreptunghic ABC,


dacă: a) sin B ≈ 0,7; b) cos B ≈ 0,2; c) tg B ≈ 0,675; d) ctg B ≈ 1,732.

12. Fie triunghiul isoscel ABC cu baza BC. Aflaţi înălţimile triunghiului, dacă: A
a) BC = 8 şi m( B) = 30°; b) BC = 12 şi m( B) = 40°;
c) BC = 14 şi m( B) = 50°; d) BC = 16 şi m( B) = 25°. B C

93
13. Fie paralelogramul ABCD. Aflaţi înăl­ţimile paralelogramului, dacă:
a) AB = 5, BC = 7 şi m( A) = 30°; b) AB = 4, BC = 9 şi m( A) = 45°;
c) AB = 3, BC = 10 şi m( A) = 60°.

14. Aflaţi înălţimea copacului, dacă: P


a) MN = 35 şi m( M) = 30°; b) MN = 46 şi m( M) = 60°.

M N

Evaluare
I II
1. Fie triunghiul KLJ. J 1. Fie triunghiul DEG. G
Completaţi: Completaţi:
a) sin J = ...; K L a) sin G = ...; D E
20
b) cos J = ...; b) cos G = ...;

c) tg J = . d) ctg J = c) tg G = ...; d) ctg G = ...

94
Cap 3. Triunghiul dreptunghic. Relaţii metrice
2. Triunghiul dreptunghic C 2. Triunghiul dreptunghic V
10
CST are CS = 7, ST = 12. STV are VS = 9, ST = 15.
Aflaţi: S T 10 Aflaţi: S T
a) tg T; b) sin T. a) tg V; b) sin V.
3. Convertiţi în radiani 24°. 10
3. Convertiţi în radiani 36°.
4. Efectuaţi: 4. Efectuaţi:
a) sin 30° + tg 60°; a) cos 30° + tg 30°;
b) sin 60° + ctg 45°. b) sin 45° + ctg 60°.
10
5. Fie triunghiul dreptun- F 5. Fie triunghiul dreptun- F
ghic DEF cu EF = 18 şi ghic DEF cu EF = 15 şi
sin F = 0,2. Aflaţi: D E sin F = 0,3. Aflaţi: D E
a) DE; b) DF. a) DE; b) DF.
L 20 L
6. Fie triunghiul dreptun- 6. Fie triunghiul dreptun-
ghic BDL cu BD = 8 şi ghic BDL cu BD = 6 şi
ctg L = 1,2. Aflaţi: B D ctg L = 2,1. Aflaţi: B D
a) BL; b) DL. A 20
a) BL; b) DL. A
D D
7. Aflaţi înălţimile BD şi CE E 7. Aflaţi înălţimile BD şi CE E
ale triunghiului ABC, dacă B C ale triunghiului ABC, dacă B C
BC = 12, m( B) = 45° şi m( C) = 30°. BC = 16, m( B) = 30° şi m( C) = 45°.

Exerciţii suplimentare
1. Calculaţi:
a) diagonalele pătratului cu laturile de lungime a;
b) înălţimea triunghiului echilateral cu laturile de lungime a.

2. Triunghiul ABC are m( B) = 60°, m( C) = 45° şi AB = 24. Aflaţi:


a) BC şi AC; b) înălţimile din B şi C ale triunghiului.

95
BD AB 2
3. Fie triunghiul ABC (m( A) = 90°) cu înălţimea AD. Demonstraţi că: = .
DC AC 2

4. Triunghiul isoscel ABC are baza BC, AB = 12 şi m( B) = 30°. Aflaţi înălţimile


triunghiului ABC.

A
5. În desen triunghiul ABC are înălţimea AD, [BE] ≡ [DC],
[CF] ≡ [BD]. Demonstraţi că [AE] ≡ [AF].

E F
B D C

6. (Teorema lui Pitagora generalizată) Fie triunghiul ABC, D = prBC A.


1) Dacă B–D–C, atunci AB2 = AC 2 + BC 2 – 2BC · CD.
2) Dacă B–C–D, atunci AB2 = AC 2 + BC 2 + 2BC · CD.

96
Cap 3. Triunghiul dreptunghic. Relaţii metrice
7. (Lungimea înălţimii) Lungimile laturilor triunghiului ABC sînt AB = c, BC = a, AC
2
= b. Arătaţi că înălţimea din A a triunghiului este ha = p(p – a)(p – b)(p – c),
a√
unde p = 0,5(a + b + c).

8. (Teorema lui Stewart) Fie triunghiul ABC, B–M–C. Atunci: A


AB · MC + AC · BM = AM · BC – BC · BM · MC.
2 2 2

B C
M

9. (Lungimea medianei) Fie triunghiul ABC, AB = c, BC = a, AC = b, M mijlocul [BC].


Atunci, 4ma2 = 2(b 2 + c 2) – a 2.

97
Capitolul 4 Patrulatere
1 Poligoane
I
G
J
B D

H
A
C E F
Linie frîntă (poligonală) Linie frîntă (poligonală)
deschisă închisă cu autointersecţie

Linie frîntă (poligonală) des- Poligoane (linii poligonale)


chisă cu autointersecţie închise fără autointersecţie

O linie frîntă (poligonală) este o figură geometrică formată din două sau mai
multe segmente care au o extremitate comună, ca în una dintre situaţiile de mai sus.
O linie frîntă are vîrfuri şi laturi.
O linie frîntă poate fi:
deschisă fără autointersecţie; deschisă cu autointersecţie;
închisă fără autointersecţie; închisă cu autointersecţie.
Poligonul este o linie poligonală închisă fără autointersecţie.
A D H K

G L J
B E
C F M I
Patrulaterul Patrulaterul Pentagonul
convex ABCD concav EFGH convex IJKLM
R S Y A5
Q A4 A
T X A6 3
O
N
P U V A1 A2
Pentagonul con- Hexagonul con- Hexagonul con-
cav NOPQR vex STUVXY cav A1A2A3A4A5A6
Un poligon este convex, dacă niciun punct al interiorului poligonului nu aparţine
unei drepte ce conţine una dintre laturile poligonului.
Un poligon este concav, dacă nu este convex. Una dintre dreptele ce conţin latu-
rile poligonului concav intersectează interiorul poligonului.
Poligonul cu şapte laturi se numeşte heptagon; cu opt laturi se numeşte octogon;

98
Cap 4. Patrulatere
cu nouă laturi se numeşte nonagon; cu zece laturi se numeşte decagon; cu 12 laturi
se numeşte dodecagon; cu n laturi − poligon cu n laturi.
Diagonală a unui poligon este un segment determi-
Diagonală

nat de două vîrfuri neconsecutive ale poligonului.


Un poligon are: vîrfuri, laturi, diagonale. Vîrfuri ne-
Numărul diagonalelor unui poligon cu n laturi este: consecutive Vîrfuri conse-
cutive
n(n − 3)
.
2

Exerciţii
1. Completaţi propoziţiile:
a) Poligonul este fără autointer­secţii.
b) Patrulaterul are laturi.
c) Pentagonul are laturi.
d) Hexagonul are laturi.
e) Heptagonul are laturi.
f) Octogonul are laturi.
g) Nonagonul are laturi.
h) Decagonul are laturi.
i) Dodecagonul are laturi.
2. Recunoaşteţi fiecare poligon:
2
1 3 4

3. Reprezentaţi:
a) pentagonul concav ABCDE; b) heptagonul convex FGHIJKL;
c) patrulaterul concav MNPQ; d) hexagonul RSTUVX.

99
4. Completaţi propoziţia:
Diagonală a unui poligon este un segment determinat de

ale poligonului.

5. Aflaţi numărul diagonalelor unui:


a) patrulater; b) pentagon; c) hexagon; d) heptagon.

2 Patrulatere
A D A D
Vîrfuri A D

Lattive
CD

cu
Ex

opuse
secu ri con-

uri
AB D
tA

Ext
tive

con
Laturi Laturi con-
A opuse
Latu

secutive

se­
BC

B
CD C B
D

B C B C
In tA A D
Ex

Vîr

B
tive

Q
tA

fur
secu

C A D A D
i co
BC

i con

D
nse

Vîrfuri consecutive

BC
D

cut

xt A M
ur

P
iv
Vîrf

E
e

B C B Diagonale B
B imediane
C N C
Diagonalele unui patrulater sînt segmentele determinate de vîrfurile opuse ale
patrulaterului.
Bimedianele unui patrulater sînt segmentele determinate de mijloacele laturilor
opuse ale patrulaterului.
Teorema sumei măsurilor unghiurilor. Suma măsurilor unghiurilor unui patru-
later convex este 360°.
Demonstraţie
Diagonala BD descompune Int ABCD în Int ABC şi Int ABD, (1). A D
x z
Conform axiomei adunării unghiurilor, (1) implică: y
v

suma măsurilor unghiurilor A, B, C, D este egală cu suma B t


u
măsurilor unghiurilor triunghiurilor ABC şi BCD, C

100
Cap 4. Patrulatere
m(ÐA) + m(ÐB) + m(ÐC) + m(ÐD) = x + y + z + t, (2).
Conform teoremei sumei măsurilor unghiurilor triunghiului, (2) implică:
m(ÐA) + m(ÐB) + m(ÐC) + m(ÐD) = 360°.
Teorema sumei măsurilor unghiurilor. Suma măsurilor unghiurilor unui patrula-
ter concav este egală cu dublul măsurii unghiului patrulaterului, de mărime maximă.
Demonstraţie
Dreapta AB intersectează CD în E, (1). A
Conform teoremei a doua a unghiului
exterior, (1) implică: x
t = x + z şi α = y + t. (2)
(2) implică: α = x + y + z. (3) α B z D
(3) implică: m(ÐA) + m(ÐB) +
m(ÐC) + m(ÐD) = 2α. y t
E
C

Exerciţii
B
1. Patrulaterul ABCD are:
vîrfurile laturile A

unghiurile diagonalele
D C
laturile opuse
2. Completaţi propoziţia:
Suma măsurilor unghiurilor unui patrulater convex este
3. Aflaţi măsurile necunoscute ale un­ghiu­rilor patrulaterului convex:
a) cu unghiurile congruente;
b) cu unghiurile două cîte două congruente şi un unghi de 57°;
c) trei dintre unghiuri de măsuri 65°, 83°, 76°;
d) trei dintre unghiuri de măsuri 43°, 108°, 74°;
e) trei dintre unghiuri de măsuri 88°, 15°, 94°.

101
4. Aflaţi măsurile necunoscute ale unghiurilor unui patrulater convex:
a) dacă unghiurile sînt două cîte două congruente şi unul dintre ele are 35°;
b) dacă unghiurile sînt două cîte două congruente şi unul dintre ele are 79°;
c) dacă unghiurile sînt două cîte două congruente şi unul dintre ele are 64°;
d) dacă unghiurile sînt două cîte două congruente şi unul dintre ele are 107°.

5. Aflaţi măsurile unghiurilor unui patrulater convex:


a) dacă ele sînt direct proporţionale cu 3, 5, 7, 9;
b) dacă ele sînt direct proporţionale cu 4, 5, 7, 2, 9;
c) dacă ele sînt direct proporţionale cu 7, 9, 19, 13;
d) dacă ele sînt direct proporţionale cu 5, 11, 10, 13.

6. Aflaţi suma măsurilor unghiurilor unui pentagon convex cu ajutorul E

sugestiei din desen. D


A

B C

102
Cap 4. Patrulatere
7. Aflaţi suma măsurilor unghiurilor patrulaterului concav
A
ABCD cu ajutorul informaţiilor din desen.
B D
65°

3 Paralelogramul

Paralelogramul este patrulaterul cu laturile opuse paralele. D C

AB || DC, AD || BC dacă şi numai dacă ABCD este parale-


logram.
Teorema laturilor paralelogramului. Un patrulater con-
A B
vex este paralelogram dacă şi numai dacă are laturile opu-
se congruente.
Teorema directă (Necesitatea)
Ipoteza. Desenul, AB || DC, AD || BC. Concluzia. [AB] ≡ [DC], [AD] ≡ [BC].
Demonstraţie. AB || DC (Ipoteza) implică ÐB2 ≡ ÐD2 (al- D 2 C
terne interne), (1); AD || BC (Ipoteza) implică ÐD1 ≡ 1

ÐB1 (alterne interne), (2). Conform ULU, [BD] latură


comună, (1) şi (2) implică ∆ABD ≡ ∆DBC, de unde
1
[AB] ≡ [DC], [AD] ≡ [BC], q.e.d. 2
A B
Teorema reciprocă (Suficienţa)
Ipoteza. Desenul, [AB] ≡ [DC], [AD] ≡ [BC].
Concluzia. AB || DC, AD || BC.
Demonstraţie. Conform LLL, [BD] latură comună, [AB] D C
1 2
≡ [DC] şi [AD] ≡ [BC] implică ∆ABD ≡ ∆DBC, de unde
ÐB2 ≡ ÐD2 (1); AD || BC (Ipoteza) implică ÐD1 ≡ÐB1, (2).
(1) alterne interne implică AB || DC, iar (2) alterne interne 1
2
implică AD || BC, q.e.d. A B

103
Exerciţii
1. Completaţi propoziţia:
a) Paralelogramul este patrulaterul cu
b) Un patrulater convex este paralelogram dacă şi numai dacă are laturile opuse

2. Pentru fiecare paralelogram din desen notaţi că are laturile opuse paralele
D C
I L
H G

E F
A B J K

3. Controlaţi dacă fiecare dintre paralelogramele de mai sus are laturile opuse con-
gruente.

4. Aflaţi perimetrul unui paralelogram, dacă are:


a) o latură de 45 mm şi altă latură de 3,6 cm;
b) o latură de 38 mm şi altă latură de 4,7 cm;
c) o latură de 28 mm şi altă latură de 5,9 cm;
d) o latură de 55 mm şi altă latură de 6,3 cm.

5. Aflaţi lungimile celorlalte laturi, dacă:


a) perimetrul unui paralelogram este de 32 cm şi o latură are 4,3 cm;
b) perimetrul unui paralelogram este de 48 cm şi o latură are 7 cm;

104
Cap 4. Patrulatere
c) perimetrul unui paralelogram este de 58 cm şi o latură are 8,3 cm;
d) perimetrul unui paralelogram este de 64 cm şi o latură are 11 cm.

6. Aflaţi lungimile a două laturi opuse ale unui paralelogram, dacă:


a) ele sînt (în cm) 5x + 7 şi 7x − 1; b) ele sînt (în cm) 3x + 11 şi 5x − 3;
c) ele sînt (în cm) 9x − 5 şi 7x + 1; d) ele sînt (în cm) 13x − 4 şi 8x + 3.

4 Unghiurile paralelogramului
Teorema unghiurilor alăturate ale paralelogramului. Un patrulater este paralelo-
gram dacă şi numai dacă are unghiurile consecutive suplementare.
Teorema directă (Necesitatea)
Ipoteza. Desenul, AB || DC, AD || BC.
D C
Concluzia. m(ÐA) + m(ÐB) = 180º, m(ÐA) + m(ÐD) = 180º.
Demonstraţie. Conform criteriilor de paralelism: AB || DC
implică: m(ÐA) + m(ÐB) = 180º; AD || BC implică
m(ÐA) + m(ÐD) = 180º, q.e.d. A B
Teorema reciprocă (Suficienţa)
Ipoteza. Desenul, m(ÐA) + m(ÐB) = 180º, m(ÐA) + m(ÐD) = 180º.
Concluzia. AB || DC, AD || BC.
Demonstraţie. Conform criteriilor de paralelism: m(ÐA) + m(ÐB) = 180º implică:
AD || BC; m(ÐA) + m(ÐD) = 180º implică AB || DC, q.e.d.
Teorema unghiurilor opuse ale paralelogramului. Un patrulater este paralelo-
gram dacă şi numai dacă are unghiurile opuse congruente.
D C
Teorema directă (Necesitatea)
Ipoteza. Desenul, AB || DC, AD || BC.
Concluzia. ÐA ≡ ÐC, ÐB ≡ ÐD.
Demonstraţie. Conform criteriilor de paralelism: AB || DC, A B

105
AD || BC implică: m(ÐA) + m(ÐD) = 180º şi m(ÐC) + m(ÐD) = 180º, (1). Teorema
unghiurilor cu acelaşi suplement implică: ÐA ≡ ÐC. La fel se demonstrează:
ÐB ≡ ÐD, q.e.d.
Teorema reciprocă (Suficienţa)
Ipoteza. Desenul implică: ABCD este patrulater convex, (1). D C
y x
(1) implică: 2x + 2y = 360º, (2). (2) implică: x + y = 180º, (3).
(3) implică: m(ÐA) + m(ÐB) = 180º şi m(ÐA) + m(ÐD)
= 180º, (4). Conform criteriilor de paralelism, (4) implică: x y
AB || DC, AD || BC, q.e.d. A B

Exerciţii
1. Completaţi propoziţia:
Un patrulater este paralelogram dacă şi numai dacă are unghiurile consecutive

2. Pentru fiecare dintre paralelogramele din desen comparaţi unghiurile opuse.


D C
I L
H G

E F
A B J K

3. Completaţi propoziţia:
Un patrulater este paralelogram dacă şi numai dacă are unghiurile opuse

4. Aflaţi măsurile celorlalte unghiuri ale paralelogramului cu un unghi de:


a) 78°; b) 53°; c) 29°; d) 38°; e) 41°.

5. Construiţi paralelogramul ABCD cu:


a) AB = 5,2 cm, BC = 3,5 cm şi m(ÐB) = 27°;
b) AB = 6 cm, BC = 2,8 cm şi m(ÐB) = 39°;

106
Cap 4. Patrulatere
c) AB = 5,7 cm, BC = 4,3 cm şi m(ÐB) = 62°;
d) AB = 4,8 cm, BC = 3,7 cm şi m(ÐB) = 73°;
e) AB = 6,4 cm, BC = 5,8 cm şi m(ÐB) = 54°.

6. ��������������������������������������������������������������������������������
Aflaţi unghiurile unui paralelogram, dacă măsurile în grade a două unghiuri con-
secutive sînt:
a) 2x + 23 şi 4x + 7; b) 4x + 5 şi 5x − 23;
c) 5x − 17 şi 3x + 5; d) 7x − 10 şi 4x + 3.

7. Aflaţi unghiurile unui paralelogram, dacă măsurile în grade a două unghiuri opuse
sînt:
a) 5x + 33 şi 7x + 27; b) 15x + 22 şi 32x + 5;
c) 7x − 6 şi 5x + 7; d) 5x − 14 şi 3x + 14.

107
8. Aflaţi
��������������������������������������������������������������������������������
unghiurile unui paralelogram, dacă măsurile în grade a două unghiuri con-
secutive sînt direct proporţionale cu numerele:
a) 2 şi 7; b) 3 şi 15; c) 7 şi 11; d) 5 şi 13.

9. Aflaţi
��������������������������������������������������������������������������������
unghiurile unui paralelogram, dacă măsurile în grade a două unghiuri con-
secutive sînt invers proporţionale cu numerele:
a) 4 şi 5; b) 3 şi 7; c) 7 şi 11; d) 5 şi 13.

108
Cap 4. Patrulatere
5 Diagonalele paralelogramului

Teorema diagonalelor paralelogramului. Un patrulater este paralelogram dacă şi


numai dacă diagonalele lui au acelaşi mijloc (se înjumătăţesc).
Corolar. Intersecţia diagonalelor este centrul de simetrie al paralelogramului.

Teorema directă (Necesitatea) D C


1 1
Ipoteza. Desenul, AB || DC, AD || BC. O
Concluzia. [AO] ≡ [OC], [DO] ≡ [BO].
Demonstraţie. Desenul, AB || DC, AD || BC (Ipoteza) im- 1 1

plică [AB] ≡ [DC] (proprietate a paralelogramului), (1). A B


AB || DC (Ipoteza) implică ÐA1 ≡ ÐC1 (alterne interne) şi ÐB1 ≡ ÐD1 (alterne in-
terne), (2). Conform ULU, (1) şi (2) implică ∆AOB ≡ ∆COD, de unde [AO] ≡ [OC],
[DO] ≡ [BO], q.e.d.
Teorema reciprocă (Suficienţa) D C
1 1
Ipoteza. Desenul, [AO] ≡ [OC], [DO] ≡ [BO]. O
Concluzia. AB || DC, AD || BC.
Demonstraţie. Conform LUL, [AO] ≡ [OC], [DO] ≡ 1 1

[BO] şi ÐAOB ≡ ÐDOC implică ∆AOB ≡ ∆COD, de unde A B


[AB] ≡ [DC], (1). În acelaşi mod se demonstrează că ∆AOD ≡ ∆COB, de unde [AD]
≡ [BC], (2). (1) şi (2) implică: ABCD este paralelogram (proprietatea laturilor parale-
logramului), q.e.d.

Teorema unei perechi de laturi ale paralelogramului. Un patrulater este paralelo-


gram dacă şi numai dacă are două laturi opuse paralele şi congruente.

Teorema directă (Necesitatea) D C


Ipoteza. Desenul, AB || DC, AD || BC.
Concluzia. AB || DC, [AB] ≡ [DC].
Demonstraţie. Conform ipotezei AB || DC. Proprietatea latu- A B
rilor paralelogramului implică: [AB] ≡ [DC].
Teorema este demonstrată.
Teorema reciprocă (Suficienţa) D C
1 1
Ipoteza. Desenul, AB || DC, [AB] ≡ [DC]. O
Concluzia. AB || DC, AD || BC.
Demonstraţie. Conform ipotezei: [AB] ≡ [DC], (1); 1 1
A B
AB || DC implică ÐA1 ≡ ÐC1 (alterne interne) şi ÐB1
≡ ÐD1 (alterne interne), (2). Conform ULU, (1) şi (2) implică ∆AOB ≡ ∆COD, de
unde [AO] ≡ [OC], [DO] ≡ [BO], (3). (3) implică ABCD este paralelogram, q.e.d.

109
Exerciţii
1. Completaţi propoziţia:
a) Un patrulater este paralelogram dacă şi numai dacă diagonalele lui

b) Intersecţia diagonalelor este

2. Construiţi paralelogramul ABCD ale cărui diagonale se intersectează în O:


a) AC = 4,8 cm, CD = 6,4 cm şi m(ÐO) = 15°;
b) AC = 5,6 cm, CD = 4,4 cm şi m(ÐO) = 21°;
c) AC = 6,2 cm, CD = 5 cm şi m(ÐO) = 23°;
d) AC = 8 cm, CD = 6 cm şi m(ÐO) = 17°;
e) AC = 7,2 cm, CD = 5,4 cm şi m(ÐO) = 33°.

3. Prin vîrfurile triunghiului ABC se construiesc dreptele paralelele cu laturile opuse


şi se obţine triunghiul DEF. Enumeraţi paralelogramele ce le descoperiţi în construc-
ţie şi triunghiurile congruente.

110
Cap 4. Patrulatere
4. ABCD este paralelogram cu intersecţia diagonalelor (centrul de simetrie O) şi
P
M–O–P, Q–O–N ca îndesen. Demonstraţi: A D
a) ∆BOM ≡ ∆DOP; O
N
Q
b) O este mijlocul [MP];
c) MNPQ este paralelogram cu centrul O. B M C

5. Completaţi propoziţia:
Un patrulater este paralelogram dacă şi numai dacă are două laturi
şi

6. Completaţi patrulaterele cu două laturi paralele şi congruente şi controlaţi dacă sînt


paralelograme:
A H I
G
C L

B J
E
D F K

7. Demonstraţi că mijloacele laturilor patrulaterului convex ABCD P C


sînt vîrfurile unui paralelogram. D
Q N
A M B

111
8. Cercetaţi dacă patrulaterul concav are o proprietate similară celei descrise în ex. 7.

D
9. Punctele E, F, G, H, I, J sînt respectiv, mijloacele laturilor
G, H, I, J sînt respectiv, mijloacele laturilor şi diagonalelor G
patrulaterului convex ABCD. Demonstraţi că: H
a) EFGH, EIGJ şi FIGJ sînt paralelograme; J C
I
b) segmentele EG, FH şi IJ au acelaşi mijloc.
A
F
E
B

10. Punctele E, F, G, H, I, J sînt, respectiv, mijloacele laturilor şi diagonalelor patru-


laterului concav ABCD. Demonstraţi că:
a) EFGH, EIGJ şi FIGJ sînt paralelograme;
b) segmentele EG, FH şi IJ au acelaşi mijloc.

112
Cap 4. Patrulatere
6 Dreptunghiul
Paralelogramul cu un unghi drept se numeşte dreptunghi. D C
AB || DC, AD || BC, m(ÐA) = 90°.
Teorema. Dreptunghiul are toate proprietăţile paralelogramului:
Dreptunghiul are laturile paralele congruente.
A B
Unghiurile opuse ale dreptunghiului sînt congruente.
Unghiurile alăturate ale dreptunghiului sînt congruente.
Diagonalele dreptunghiului se înjumătăţesc.
Intersecţia diagonalelor dreptunghiului este centrul lui de simetrie.
Teorema unghiurilor dreptunghiului. Un patrulater este dreptunghi dacă şi numai
dacă are trei unghiuri drepte.
Corolar. Dreptunghiul are toate unghiurile drepte.
Teorema directă (Necesitatea) D C
Ipoteza. Desenul, AB || DC, AD || BC, m(ÐA) = 90°.
Concluzia. m(ÐA) = m(ÐD) = m(ÐB) = 90°.
Demonstraţie. AB || DC, AD || BC implică ABCD are unghiu-
rile consecutive suplementare, (1). (1) şi m(ÐA) = 90° implică A B
m(ÐD) = 90°, (2), şi m(ÐB)= 90°, (3). (2) şi (3) implică m(ÐA) = m(ÐD) = m(ÐB)
= 90°, q.e.d. D C
Teorema reciprocă (Suficienţa)
Ipoteza. Desenul, m(ÐA) = m(ÐD) = m(ÐB) = 90°.
Concluzia. AB || DC, AD || BC.
Demonstraţie. Conform Teoremei Paralelelor, m(ÐA) = A B
m(ÐD) = 90° (ipoteză) implică m(ÐA) + m(ÐD) = 180°, de unde AB || DC. În acelaşi
mod se deduce că AD || BC, q.e.d.
Teorema diagonalelor dreptunghiului. Un paralelogram este dreptunghi dacă şi
numai dacă are diagonalele congruente.
Teorema directă (Necesitatea) D C
Ipoteza. Desenul, AB || DC, AD || BC, m(ÐA) = 90°.
Concluzia. [AC] ≡ [BD].
Demonstraţie. Conform teoremei anterioare, ipoteza implică
m(ÐB)= 90°, (1). Ipoteza implică [AD] ≡ [BC], (2). A B
Conform CC, [AB] latură comună, (1) şi (2) implică ∆ABD ≡∆ABC, de unde [AC] ≡
[BD], q.e.d. D C
Teorema reciprocă (Suficienţa)
Ipoteza. Desenul, AB || DC, AD || BC, [AC] ≡ [BD].
Concluzia. ABCD este dreptunghi.
Demonstraţie. Conform LLL, (1), [AC] ≡ [BD] (ipoteză) şi A B
[AB] latură comună implică ∆ABD ≡ ∆BAC, de unde m(ÐA) = m(ÐD), (1). m(ÐA) +
m(ÐD) = 180° (proprietate a paralelogramului), (2). (1) şi (2) implică m(ÐA) = 90°,
(3), şi AB || DC, AD || BC, implică ABCD este dreptunghi, q.e.d.

113
Teoremă. Un paralelogram este dreptunghi dacă şi numai dacă mediatoarele a două
laturi opuse coincid.
Teorema directă (Necesitatea) A M D
Ipoteza. Desenul, AB || DC, AD || BC, m(ÐA) =
90°, M − mijlocul [AD], N − mijlocul [BC].
Concluzia. MN este mediatoarea laturilor
AD şi BC.
Demonstraţie. Ipoteza implică: ABNM are
laturrile opuse AM şi BN paralele şi congru-
ente, (1). m(ÐA) = 90° şi (1) implică: ABNM
este dreptunghi, (2). (2) implică MN este medi- B N C
atoarea laturilor AD şi BC, q.e.d.
Teorema reciprocă (Suficienţa) A M D
Ipoteza. Desenul, AB || DC, AD || BC, MN
este mediatoarea laturilor AD şi BC.
Concluzia. ABCD este dreptunghi.
Demonstraţie. Ipoteza implică: MNCB are
laturile opuse MD şi NC paralele şi congru-
ente, (1). m(ÐM) = 90° şi (1) implică: MNCB
este dreptunghi, (2). (2) implică ABCD este
dreptunghi, q.e.d. B N C

Teoremă axelor de simetrie ale dreptunghiului. Un paralelogram este dreptunghi


dacă şi numai dacă mediatoarele a două laturi opuse coincid.
Corolar. Mediatoarele laturilor opuse ale unui dreptunghi sînt
axele de simetrie ale dreptunghiului.
Corolar. Intersecţia diagonalelor şi intersecţia axelor de simetrie
ale unui dreptunghi coincid cu centrul cercului circumscris drept-
unghiului.

Exerciţii
1. Completaţi propoziţiile:
a) Paralelogramul cu un unghi drept se numeşte
b) Dreptunghiul are toate proprietăţile
1
2. Descoperiţi care dintre patrulaterele din 4
3
desen are trei un­ghiuri congruente.
2

114
Cap 4. Patrulatere
3. Identificaţi dreptunghiurile din desenul de la exerciţiul 2.

4. ABCD este un dreptunghi. Aflaţi numerele x, dacă măsurile în grade ale un­­ghiu­
rilor A şi B sînt:
a) 7x + 13 şi 11x − 31; b) 13x − 1 şi 9x + 27;
c) 12x − 54 şi 8x − 6; d) 19x − 5 şi 21x − 15.

5. Completaţi propoziţia:
Un paralelogram este dreptunghi dacă şi numai dacă are diagonalele

6. Construiţi dreptunghiul ABCD cu centrul de simetrie O:


a) AC = 8 cm, m(ÐO) = 21°; b) AC = 3,8 cm, m(ÐO) = 19°;
c) AC = 7,2 cm, m(ÐO) = 32°; d) AC = 4,6 cm, m(ÐO) = 43°.

115
7. Construiţi dreptunghiul ABCD cu:
a) AB = 7 cm şi BC = 4,6 cm; b) AB = 6,8 cm şi BC = 2,4 cm;
c) AB = 5,4 cm şi BC = 3,1 cm; d) AB = 6 cm şi BC = 2,5 cm.

8. Construiţi dreptunghiul ABCD cu: a) AC = 5,6 cm, m(ÐCAB) = 15°;


b) AC = 4,8 cm, m(ÐCAB) = 32°; c) AC = 5,2 cm, m(ÐCAB) = 23°.

116
Cap 4. Patrulatere
9. Construiţi dreptunghiul ABCD cu:
a) AB = 4,5 cm, m(ÐADB) = 30°; b) AB = 3,8 cm, m(ÐADB) = 26°;
c) AB = 3,4 cm, m(ÐADB) = 33°; d) AB = 5,2 cm, m(ÐADB) = 41°.

10. Aflaţi diagonalele în cm ale dreptun­ghiului ABCD, dacă:


a) AC = 2x + 1 şi BD = 3x − 6; b) AC = 5x − 4 şi BD = 3x + 8;
c) AC = 3x + 2 şi BD = 7x − 18; d) AC = 5x + 11 şi BD = 7x + 3.

11. Completaţi propoziţiile:


a) Un paralelogram este dreptunghi dacă şi numai dacă mediatoarea a două laturi
opuse este
b) Mediatoarele laturilor opuse ale unui dreptunghi sînt axele

c) Intersecţia diagonalelor unui dreptunghi este centrul cercului

117
12. Construiţi dreptunghiul ABCD cu:
a) AC = 8 cm, AB = 3,4 cm; b) AC = 5,8 cm, AB = 2 cm;
c) AC = 7 cm, AB = 2,5 cm; d) AC = 6,6 cm, AB = 3 cm.

13. Construiţi axele de simetrie ale dreptunghiului ABCD cu:


a) AC = 6 cm, m(ÐCAB) = 18°; b) AC = 7,2 cm, m(ÐCAB) = 22°;
c) AC = 5,8 cm, m(ÐCAB) = 36°; d) AC = 8 cm, m(ÐCAB) = 29°.

118
Cap 4. Patrulatere
7 Patrulatere ortodiagonale
D H

A C E
G

B F
AC ^ BD, EG ^ FH
Patrulaterul cu diagonalele perpendiculare se numeşte patrulater ortodiagonal.
Teoremă. Dacă un patrulater are două perechi de laturi consecutive congruente,
atunci el este patrulater ortodiagonal şi o diagonală formează unghiuri congruente
cu laturile congruente ale patrulaterului.
Ipoteza. Desenul, [AB] ≡ [AD] şi [BC] ≡ [CD]. A A
Concluzia. AC ^ BD, ÐBAC ≡ ÐCAD şi ÐBCA ≡
ÐACD.
Demonstraţie. Fie [AE] mediana corespunzătoa- C
re bazei triunghiului isoscel ABD, (1). Con-
E E
form proprietăţii triunghiului isoscel, AE ^ B D B D
BD, (2). (1) implică [CE] este mediana corespunzătoare bazei
triunghiului isoscel BCD, (3). (3) implică CE ^ BD, (4).
C
(2) şi (4) implică A – E – C, (5). (5) implică AC ^ BD. Conform
LLL, [AB] ≡ [AD], [BC] ≡ [CD] şi [AC] latură comună, ∆ABC ≡ ∆ADC, (6). (6)
implică: unghiurile BAC şi CAD sînt congruente; unghiurile BCA şi ACD sînt con-
gruente, q.e.d. În acelaşi mod se demonstrează teorema pentru patrulaterul concav.
Corolar. Dacă un patrulater are două perechi de laturi consecutive congruente,
atunci unghiurile formate de laturile necongruente ale patrulaterului sînt congru-
ente.
A A
Ipoteza. Desenul, [AB] ≡ [AD] şi [BC] ≡ [CD].
Concluzia. ÐABC ≡ ÐADC.
Demonstraţie. Conform LLL, [AB] ≡ [AD], C
[BC] ≡ [CD] şi [AC] latură comună, E E
B DB D
∆ABC ≡ ∆ADC.
Rezultă că: ÐABC ≡ ÐADC, q.e.d.
C
Corolar. Dacă un patrulater ortodiagonal are o pereche de laturi consecutive con-
gruente, atunci celelalte laturi ale patrulaterului sînt congruente.

119
Exerciţii
1. Completaţi propoziţiile:
a) Patrulaterul ortodiagonal are diagonalele
b) Dacă un patrulater are două pe­rechi de laturi consecutive congruente, atunci el
este patrulater
2. Identificaţi patrulaterele ortodiagonale:

1 4

3. Demonstraţi că mijloacele laturilor patrulaterului ortodiagonal convex ABCD sînt


vîrfurile unui dreptunghi.

4. Demonstraţi că mijloacele laturilor patrulaterului ortodiagonal concav ABCD sînt


vîrfurile unui dreptunghi.

5. ABCD este patrulater ortodiagonal. Aflaţi măsura unghiului CAB, dacă măsurile în
grade ale unghiurilor CAB şi DBA sînt:
a) 5x + 3 şi 7x + 3; b) 8x + 11 şi 29x + 5;
c) 9x + 13 şi 15x + 19; d) 13x + 5 şi 15x + 1.

120
Cap 4. Patrulatere
6. ABCD este patrulater ortodiagonal cu laturile AB şi AD congruente. Aflaţi BC,
dacă CD este: a) 17,2 cm; b) 29,25 cm; c) 38,5 cm; d) 57,28 cm.

7. ABCD este patrulater ortodiagonal cu laturile AB şi AD congruente. Aflaţi măsura


unghiului ABC, dacă măsurile în grade ale unghiurilor ABC şi ADC sînt:
a) 23x + 10 şi 25x + 2; b) 19x – 25 şi 17x – 11;
c) 15x + 8 şi 18x – 13; d) 16x + 9 şi 18x – 5.

8 Diagonalele rombului
Rombul este paralelogramul cu două laturi consecutive congruente. D
AB || CD, AD || DC, [AB] ≡ [BC].
A C
Teoremă. Un patrulater este romb dacă şi numai dacă are laturile
congruente.
B

121
Teorema directă (Necesitatea) D
Ipoteza. Desenul. AB || DC, AD || BC, [AB] ≡ [BC].
Concluzia. [AB] ≡ [BC] ≡ [CD] ≡ [AD].
Demonstraţie. Proprietatea laturilor paralelogramului implică A C
[AB] ≡ [DC] şi [AD] ≡ [BC], (1). Ipoteza şi (1) implică [AB] ≡
[BC] ≡ [CD] ≡ [AD], q.e.d. D
Teorema reciprocă (Suficienţa)
Ipoteza. Desenul. AB] ≡ [BC] ≡ [CD] ≡ [AD]. B
Concluzia. AB || DC, AD || BC, [AB] ≡ [BC]. A C
Demonstraţie. Conform criteriului LLL,
[AB] ≡ [BC] ≡ [CD] ≡ [AD] şi [AB] latură
comună implică ∆ ABC ≡ ∆ ADC, (1). (1) implică ÐABC ≡
ÐADC, (2). În acelaşi mod se deduce că ÐBAD ≡ ÐBCD, (3). B
Ipoteza, (2) şi (3) implică AB || DC, AD || BC, [AB] ≡ [BC], q.e.d.
Teorema diagonalelor rombului. Un paralelogram este romb dacă şi numai dacă
are diagonalele perpendiculare.
Teorema directă (Necesitatea)
Ipoteza. Desenul. [AB] ≡ [BC] ≡ [CD] ≡ [AD].
Concluzia. BD ^ AC.
Demonstraţie. Conform teoremei patrulaterelor ortodiagonale, ipoteza implică
BD ^ AC, q.e.d.
D
Teorema reciprocă (Suficienţa)
Ipoteza. Desenul. AB || DC, AD || BC, BD ^ AC.
Concluzia. [AB] ≡ [BC]. A C
Demonstraţie. Proprietatea diagonalelor paralelogramului şi ipo-
teza implică ABC este triunghi isoscel cu baza AC, AB || DC, AD
|| BC, (*). (*) şi ipoteza implică [AB] ≡ [BC], q.e.d. B
Consecinţe. 1) Un paralelogram este romb dacă şi numai dacă o diagonală este şi
bisectoare.
2) Diagonalele rombului sînt şi bisectoare.
3) Diagonalele rombului sînt axele lui de simetrie.

Exerciţii
1. Completaţi propoziţiile:
a) Rombul este paralelogramul cu două laturi consecutive
b) Un patrulater este romb dacă şi numai dacă are laturile
2. Construiţi rombul ABCD cu: a) BC = 4 cm, m(ÐA) = 62°;
b) AB = 4 cm, m(ÐA) = 29°; c) AD = 4 cm, m(ÐB) = 34°;
d) BC = 4 cm, m(ÐC) = 48°; e) CD = 4 cm, m(ÐD) = 51°.

122
Cap 4. Patrulatere
3. Construiţi rombul ABCD cu:
a) AB = 5 cm, AC = 5 cm; b) AB = 4,3 cm, AC = 4,3 cm;
c) AB = 5,2 cm, AC = 5,2 cm; d) AB = 4,5 cm, AC = 4,5 cm;
e) AB = 3,9 cm, BD = 3,9 cm; f) AB = 2,8 cm, BD = 2,8 cm.

4. Construiţi rombul ABCD cu:


a) AB = 2,5 cm, AC = 4 cm; b) AB = 3,3 cm, AC = 4,7 cm;
c) AB = 3,6 cm, AC = 5 cm; d) AB = 4,5 cm, AC = 5,2 cm;
e) AB = 4,1 cm, BD = 5,2 cm; f) AB = 3,7 cm, BD = 4,8 cm.

123
5. Completaţi propoziţia:
Un paralelogram este romb dacă şi numai dacă are diagonalele

6. Examinaţi romburile şi controlaţi dacă diagonalele lor au acelaşi mijloc şi sînt


perpendiculare:
D H L

K
A C E G
I

B F J

7. Construiţi rombul ABCD cu diagonalele:


a) AC = 5 cm, BD = 6 cm; b) AC = 4,4 cm, BD = 5 cm;
c) AC = 4 cm, BD = 2,2 cm; d) AC = 4,6 cm, BD = 3,8.

8. Completaţi propoziţiile:
a) Un paralelogram este romb dacă şi numai dacă o diagonală este şi

b) Diagonalele rombului sînt şi

c) Diagonalele rombului sînt şi axe

9. Construiţi rombul ABCD cu:


a) AB = 3,8 cm, AC = 5,2 cm; b) BC = 5,3 cm, AC = 6,3 cm;
c) CD = 2,8 cm, AC = 4 cm; d) AD = 5,1 cm, AC = 4,8 cm.

124
Cap 4. Patrulatere
10. Aflaţi măsurile unghiurilor rombului ABCD, dacă:
a) m(ÐBAC) = 15°; b) m(ÐBAC) = 21°; c) m(ÐBAC) = 26°;
d) m(ÐBAC)= 31°; e) m(ÐBAC) = 29°; f) m(ÐBAC) = 35°.

11. Construiţi rombul ABCD cu:


a) AC = 7,2 cm, m(ÐA) = 68°; b) AC = 6,4 cm, m(ÐA) = 76°;
c) AC = 5,3 cm, m(ÐA) = 84°; d) AC = 4,8 cm, m(ÐA) = 56°.

12. Construiţi rombul ABCD cu:


a) AC = 4,5 cm, m(ÐDBA) = 49°; b) AC = 3,7 cm, m(ÐDBA) = 27°;
c) AC = 5,3 cm, m(ÐDBA) = 51°; d) AC = 2,9 cm, m(ÐDBA) = 33°.

125
13. Aflaţi lungimile laturilor şi măsurile unghiurilor rombului ABCD, dacă:
a) BD = 5 cm şi m(ÐBAC) = 30°; b) BD = 3 cm şi m(ÐBAC) = 30°;
c) BD = 7 cm şi m(ÐBAC) = 30°; d) AC = 11 cm şi m(ÐBAC) = 60°;
e) AC = 9 cm şi m(ÐBAC) = 60°; f) AC = 13 cm şi m(ÐBAC) = 60°.

14. Construiţi rombul ABCD, dacă:


a) BD = 3,6 cm şi m(ÐBAC) = 25°; b) BD = 4,2 cm şi m(ÐBAC) = 20°;
c) BD = 5,2 cm şi m(ÐBAC) = 22°; d) AC = 7 cm şi m(ÐBAC) = 26°;
e) AC = 8,2 cm şi m(ÐBAC) = 50°; f) AC = 5,8 cm şi m(ÐBAC) = 53°.

15. Construiţi rombul ABCD, dacă:


a) AC = 4,8 cm şi m(ÐB) = 108°; b) AC = 5,4 cm şi m(ÐA) = 68°;
c) BD = 5,6 cm şi m(ÐB) = 38°; d) BD = 4,4 cm şi m(ÐC) = 76°.

126
Cap 4. Patrulatere
16. Demonstraţi că mijloacele laturilor dreptunghiu- A
Q
D
lui ABCD sînt vîrfurile unui romb.

M P

B C
N

17. [AD este bisectoarea unghiului A al triunghiului ABC; A


MD || AB, PD || AC. Demonstraţi că: PM ^ AD.
M
P

B D C

18. M, N, P sînt mijloacele laturilor triunghiului ABC; Q este B


simetricul punctului P faţă de M. Demonstraţi că intersecţia
M
segmentelor AQ şi MN este mijlocul segmentului MN. P Q

O
A C
N

19. ABC este triunghi isoscel cu baza BC, m(ÐA) = m(ÐDBA) = 20°, m(ÐACE) =
30°, DE || BC. Demonstraţi că triunghiul GBF este C
isoscel. D
F
A

E G
B

127
9 Înălţimile rombului
Înălţime a unui paralelogram este perpendiculara (segmen- A D
tul) dintr-un vîrf pe una dintre laturile lui. Înălţimile para-
lelogramului oarecare sînt necongruente. F
Distanţele dintre laturile opuse ale paralelogramului oare-
care sînt diferite. B C
E
AB || CD, AD || DC, E = prDC A, F = prBC A.
Teorema înălţimilor rombului. Un paralelogram este romb dacă şi numai dacă
înălţimile lui sînt egale.
Necesitatea A
Ipoteza. Desenul. AB || DC, AD || BC, [AB] ≡ [BC], AE ^ BC,
AF ^ CD. E F
Concluzia. [AE] ≡ [AF]. B D
Demonstraţie. AB || DC (Ipoteza) implică ÐEAB ≡ ÐFAD (ÐBAD
complementul comun), (1). Conform IU, ipoteza şi (1) implică
DABE ≡ DADF, de unde [AE] ≡ [AF], q.e.d. C
Suficienţa A
Ipoteza. Desenul. AB || DC, AD || BC, AE ^ BC, AF ^ CD,
E F
[AE] ≡ [AF].
Concluzia. [AB] ≡ [BC]. B D
Demonstraţie. AB || DC (Ipoteza) implică ÐEAB ≡ ÐFAD (ÐBAD
complementul comun), (1). Conform CU, ipoteza şi (1) implică
C
DABE ≡ DADF, de unde [AB] ≡ [AD], q.e.d.
Rigla negradată permite unei perechi de drepte paralele. Distanţa dintre ele este
egală cu lăţimea riglei.
Corolar. Proprietatea riglei negradate. Intersecţiile a două perechi de drepte para-
lele construite cu ajutorul riglei negradate sînt vîrfurile unui romb.

Exerciţii
1. Completaţi propoziţiile:
a) Înălţime a unui paralelogram este perpendiculara (segmentul) dintr-un vîrf pe
una dintre

128
Cap 4. Patrulatere
b) Înălţimile paralelogramului oare­care sînt

c) Distanţele dintre laturile opuse ale paralelogramului oarecare sînt

2. Construiţi înălţimile dintr-un vîrf al fie­cărui paralelogram:


A D
E H

L K

F G
I J
B C

3. Completaţi propoziţia:
Un paralelogram este romb dacă şi numai dacă înălţimile lui sînt
4. Controlaţi dacă fiecare romb are înăl­ţimile egale:
D H L

K
A C E G
I

B F J

5. Completaţi propoziţia:
Intersecţiile a două perechi de drepte paralele construite cu ajutorul riglei negra-
date sînt vîrfurile unui

10 Construcţii

l Construcţia bisectoarei unui unghi cu ajutorul riglei negradate.

129
l Construcţia mediatoarei unui segment cu ajutorul riglei negradate.

Exerciţii
1. Construiţi bisectoarele unghiurilor cu
ajutorul riglei negradate:

A B C

2. Construiţi mediatoarea segmentului cu ajutorul riglei negradate:

C
E

A B F

3. Construiţi cu ajutorul riglei negradate perpendiculara într-un punct pe o dreaptă.

d A

130
Cap 4. Patrulatere
4. Construiţi cu ajutorul riglei negradate paralela printr-un punct la o dreaptă.

5. Construiţi cu ajutorul riglei negradate perpendiculara dintr-un punct pe o dreaptă.

11 Pătratul
Pătratul este dreptunghiul cu două laturi consecutive congru-
ente sau rombul cu un unghi drept.
AB || CD, AD || DC, [AB] ≡ [AD], AB ^ AD.
Proprietăţile pătratului. Teoremă. Pătratul are proprietăţile
dreptunghiului şi ale rombului:
1) unghiurile drepte şi laturile congruente;
2) diagonalele congruente şi perpendiculare;
3) diagonalele sînt bisectoare;
4) diagonalele şi mediatoarele laturilor opuse sînt axele de simetrie ale pătratului;
5) intersecţia diagonalelor pătratului este centrul cercului circumscris şi centrul
cercul înscris (tangent laturilor) în pătrat.

131
Exerciţii
1. Completaţi propoziţiile:
a) Pătratul este dreptunghiul cu două laturi consecutive sau rombul cu un

b) Pătratul are proprietăţile şi ale


2. Construiţi pătratul ABCD cu:
a) AC = 4,6 cm; b) BD = 6,1 cm; c) AC = 3,4 cm; d) AC = 5,2 cm.

3. Construiţi pătratul ABCD cu:


a) AB = 4,2 cm; b) AB = 3,2 cm; c) BC = 4,6 cm; d) AD = 5,2 cm; e) CD = 2,8 cm.

132
Cap 4. Patrulatere
4. ABCD este pătrat, triunghiul ABE este echilateral. Aflaţi mă- D C
surile unghiurilor: AED, AEB, BEC, DEC.
E

A B

5. ABCD este pătrat, triunghiurile ABE şi BEC sînt D C


echilaterale.
E
a) Calculaţi m(ÐAED). b) Calculaţi m(ÐBEF).
c) Arătaţi că punctele D, E, F (unghiul DEF este F
alungit) sînt coliniare.
A B

133
12 Trapezul
l Trapezul este patrulaterul cu o pereche de laturi D C
paralele. Baza mică
AB || CD, AD şi DC nu sunt paralele.
l Trapezul are două laturi paralele numite baze

Înălţime
Laturi
(baza mare şi baza mică) şi două laturi nepara- neparalele
lele. Unghiurile alăturate fiecărei laturi nepara- Baza mare
lele sînt suplementare. A E B
Q P
l Trapezul cu un unghi drept este un trapez
dreptunghic.
MN || PQ, [MN] este baza mare, [PQ] este
baza mică, [NP] şi [MQ] sînt laturi nepara-
lele, [MQ] este înălţimea din Q.
Teoremă. Trapezul dreptunghic are două
unghiuri drepte. M N
l Construcţia trapezului dreptunghic ABCD cu baza mare dată m,
unghiul drept A, înălţimea dată h baza mică dată n.
D D n D n C
h

m m m
A B A B A B

Exerciţii
1. Completaţi propoziţiile:
a) Trapezul are două laturi paralele numite ( mare şi mică)
şi două
b) Unghiurile alăturate fiecărei laturi neparalele sînt

2. Enumeraţi elementele fiecărui trapez: A D


E H

L K

F G
I J
B C

134
Cap 4. Patrulatere
3. Construiţi înălţimile fiecărui trapez:
A D
E H

L K
F G

B C I J

4. Completaţi propoziţiile:
a) Trapezul cu un unghi drept este
b) Trapezul dreptunghic are un­ghiuri drepte.
5. Enumeraţi elementele fiecărui trapez: A D E H
L K

B C F G I J

6. Construiţi trapezul dreptunghic ABCD cu:


a) baza mare AB = 5 cm, înălţimea AD = 3 cm, CD = 2,4 cm;
b) baza mare AB = 5,3 cm, înălţimea AD = 4,1 cm, CD = 3,2 cm;
c) baza mare AB = 6,1 cm, înălţimea AD = 2,8 cm, CD = 3,5 cm;
d) baza mare AB = 4,8 cm, înălţimea AD = 2,5 cm, CD = 3,1 cm.

135
7. Construiţi trapezul dreptunghic ABCD cu:
a) baza mare AB = 6,4 cm, înălţimea AD = 3,5 cm, m(ÐB) = 76°;
b) baza mare AB = 6,2 cm, înălţimea AD = 2,2 cm, m(ÐB) = 45°;
c) baza mare AB = 4,6 cm, înălţimea AD = 2,3 cm,m(ÐB) = 47°;
d) baza mare AB = 5,1 cm, înălţimea AD = 3,2 cm, m(ÐB) = 55°.

8. Construiţi trapezul dreptunghic ABCD cu baza mare AB = 6,4 cm, CD = 3,2 cm,
m(ÐB) = 30°.

9. Construiţi trapezul dreptunghic ABCD cu baza mare AB = 5,1 cm, CD = 2,6 cm,
m(ÐD) = 60°.

136
Cap 4. Patrulatere
13 Trapezul isoscel
l Trapezul isoscel are laturile neparalele congruente. D C
AB || CD; AD şi DC nu sunt paralele; [AD] ≡ [DC].
Teorema unghiurilor trapezului isoscel. Un trapez este
isoscel dacă şi numai dacă are unghiurile alăturate unei
baze congruente.
Corolar. Unghiurile alăturate bazelor unui trapez isos- A B
cel sînt congruente.
Necesitatea
Ipoteza. Desenul. AB || DC, [AD] ≡ [BC]. Concluzia. ÐDAB ≡ ÐCBA.
Demonstraţie. Fie DE ^ AB, CF ^ AB, (1). Ipoteza şi (1) D C
implică DEFC dreptunghi, de unde [DE] ≡ [CF], (2).
Conform IC, (1) şi (2) implică DADE ≡ DBCF, de unde
ÐDAB ≡ ÐCBA, q.e.d.
Suficienţa
Ipoteza. Desenul. AB || DC, ÐDAB ≡ ÐCBA. A B
E F
Concluzia. [AD] ≡ [BC].
D C
Demonstraţie. Fie DE ^ AB, CF ^ AB, (1). Ipoteza şi (1) im-
plică DEFC dreptunghi, de unde [DE] ≡ [CF], (2).
Conform CU, ipoteza, (1) şi (2) implică DADE ≡ DBCF, de
unde [AD] ≡ [BC], q.e.d.

A E F B
Exerciţii
1. Completaţi propoziţiile:
a) Trapezul isoscel are laturile neparalele
b) Un trapez este isoscel dacă şi numai dacă are unghiurile alăturate unei baze

2. Construiţi trapezul isoscel ABCD cu:


a) baza mare AB = 6,4 cm, AD = 2,3 cm, m(ÐB) = 72°;
b) baza mare AB = 5,3 cm, AD = 2,5 cm, m(ÐB) = 81°;
c) baza mare AB = 5,8 cm, AD = 3,4 cm, m(ÐA) = 63°;
d) baza mare AB = 6,8 cm, AD = 3,7 cm, m(ÐA) = 69°.

137
3. Construiţi trapezul isoscel ABCD cu:
a) baza mare AB = 5,1 cm, AD = 2,1 cm, m(ÐD) = 96°;
b) baza mare AB = 6,9 cm, AD = 2,3 cm, m(ÐD) = 97°;
c) baza mare AB = 6,4 cm, AD = 2,5 cm, m(ÐD) = 98°;
d) baza mare AB = 7,4 cm, AD = 3,8 cm, m(ÐD) = 111°.

138
Cap 4. Patrulatere
4. Aflaţi măsurile unghiurilor B, C, D ale trapezului isoscel ABCD cu baza mare AB,
dacă: a) m(ÐA) = 36°; b) m(ÐA) = 47°; c) m(ÐA) = 58°; d) m(ÐA) = 41°.

5. Aflaţi măsurile unghiurilor B, C, D ale trapezului isoscel ABCD cu baza mică AB,
dacă: a) m(ÐA) = 114°; b) m(ÐA) = 108°; c) m(ÐA) = 127°; d) m(ÐA) = 151°.

6. Trapezul isoscel ABCD are baza mare AB. Aflaţi AD, dacă:
a) m(ÐA) = 30° şi înălţimea trapezului este de 21 mm;
b) m(ÐA) = 30° şi înălţimea trapezului este de 33 mm;
c) m(ÐA) = 30° şi înălţimea trapezului este de 27 mm;
d) m(ÐA) = 30° şi înălţimea trapezului este de 45 mm.

7. Trapezul isoscel ABCD are baza mare AB. Aflaţi AD, dacă:
a) m(ÐA) = 60° şi diferenţa lungimilor bazelor 24 mm;
b) m(ÐA) = 60° şi diferenţa lungimilor bazelor 38 mm;
c) m(ÐA) = 60° şi diferenţa lungimilor bazelor 24 mm;
d) m(ÐA) = 60° şi diferenţa lungimilor bazelor 37 mm.

139
8. Trapezul isoscel ABCD are baza mare AB. Aflaţi înălţimea trapezului, dacă:
a) m(ÐA) = 30° şi AD = 8,2 cm; b) m(ÐA) = 30° şi AD = 7,4 cm;
c) m(ÐA) = 30° şi AD = 9,8 cm; d) m(ÐA) = 30° şi AD = 5,6 cm.

9. Construiţi trapezul isoscel ABCD cu baza mare AB = 5,5 cm, AD = 3,2 cm,
înălţimea MD = 2,5 cm. Enumeraţi etapele.

10. Construiţi trapezul isoscel ABCD cu baza mare AB = 6,8 cm, CD = 3,4 cm,
m(ÐA) = 60°. Calculaţi perimetrul tra­pezului.

11. Construiţi trapezul isoscel ABCD cu baza mică CD = 4,2 cm, AD = 3,5 cm,
m(ÐA) = 60°. Calculaţi perimetrul tra­pezului.

140
Cap 4. Patrulatere
12. Construiţi trapezul isoscel ABCD cu baza mare AB = 6,4 cm, CD = 3,2 cm şi
m(ÐB) = 30°. Enumeraţi etapele.

13. Aflaţi perimetrul trapezului isoscel ABCD cu baza mare AB = 8,6 cm, m(ÐA) =
60°, m(ÐACD) = 90°.

14 Trapezul isoscel. Proprietăţi


Teorema diagonalelor trapezului isoscel. Un trapez este isoscel dacă şi numai dacă
are diagonalele congruente.
Necesitatea D C
Ipoteza. Desenul. AB || DC, [AD] ≡ [BC].
Concluzia. [AC] ≡ [BD].
Demonstraţie. Ipoteza implică ÐDAB ≡ ÐCBA, (1). Conform
LUL, (1), ipoteza ([AD] ≡ [BC]) şi [AB] latură comună implică
DABD ≡ DBAC, de unde [AC] ≡ [BD], q.e.d. A B
Suficienţa D C
Ipoteza. Desenul. AB || DC, [AC] ≡ [BD].
Concluzia. [BC] ≡ [AD].
Demonstraţie. Fie DE ^ AB, CF ^ AB, (1). Ipoteza şi (1)
implică DEFC dreptunghi, de unde [DE] ≡ [CF], (2).
Conform IC, ipoteza, (1) şi (2) implică DAFC ≡ DBED, de
unde ÐCAF ≡ ÐDBE, (3). Conform LUL, ipoteza şi (3) im- A E F B
plică DCAB ≡ DDBA, de unde [BC] ≡ [AD], q.e.d.
Teoremă axei de simetrie a trapezului isoscel. Un trapez este isoscel dacă şi numai
dacă mediatoarea unei baze coincide cu mediatoarea celeilalte baze.

141
Necesitatea
Ipoteza. Desenul. AB || DC, [AD] ≡ [BC], d mediatoarea [AB].
Concluzia. [DF] ≡ [FC].
Demonstraţie. Fie M intersecţia AD şi BC, (1). Ipoteza (AB || M
DC, [AD] ≡ [BC]) implică ÐDAB ≡ ÐCBA, de unde ABM este
triunghi isoscel cu baza AB, (2). Ipoteza (d mediatoarea [AB]) D C
F
implică d = ME, (3). Ipoteza (AB || DC) şi (2) implică MDC
d
este triunghi isoscel cu baza DC, (4). (3) şi (4) implică d este
mediatoarea [DC], de unde [DF] ≡ [FC], q.e.d. A E B
Suficienţa
Ipoteza. Desenul. AB || DC, d mediatoarea [AB] şi [CD]. D F C
Concluzia. [BC] ≡ [AD].
d
Demonstraţie. Fie DN ^ AB, CP ^ AB, (1). Ipoteza (AB ||
DC) şi (1) implică DNPC dreptunghi, de unde [NE] ≡
A N E P B
[EP] şi [DN] ≡ [CP], (2). Ipoteza (d mediatoarea [AB]
şi [CD]) şi (2) implică [AN] ≡ [PB], (3).
Conform CC, (1), (2) şi (3) implică DAND ≡ DBPC, de unde [AD] ≡ [BC], q.e.d.
l Construcţia trapezului isoscel ABCD cu baza mare de lungime m, unghiul A de
măsură u şi lungimea uneia dintre laturile neparalele n.
D D C D C
n
n n n
u
m u m u m
A B A B A B

l Construcţia trapezului isoscel ABCD cu baza mare AB de lungime m, lungimea une­


ia dintre diagonale de lungime d, lungimea bazei mici p.

D p C Se construieşte paralelo- D p C Executarea construcţiei. Se


gramul BECD. Rezultă construieşte: triunghiul isos-
[AC] ≡ [CE]. cel AEC cu baza de lungime
d d d d d AE = AB + BE = m + p şi
AC = d; paralelogramul
BECD; se completează
m m p ABCD.
A B A B E

Exerciţii
1. Completaţi propoziţia:
Un trapez este isoscel dacă şi numai dacă are diagonalele

142
Cap 4. Patrulatere
2. Identificaţi trapezele isoscele: H
E
A D L K

F G
I J
B C

3. Comparaţi unghiurile alăturate fiecărei L


A D E H
baze:
K

J
F G
B C
I

4. Comparaţi lungimile diagonalelor fie­că­rui trapez de la exerciţiul 3.

5. Construiţi înălţimile din fiecare vîrf al bazei mici:


E H
L
A D
K

F G J

I
B C

6. Construiţi trapezul isoscel ABCD cu:


a) baza mare AB = 5,1 cm, AC = 4,5 cm, BC = 3 cm;
b) baza mare AB = 5,6 cm, AC = 6,1 cm, BC = 4,8 cm;
c) baza mare AB = 7,4 cm, AC = 6,3 cm, BC = 2,5 cm;
d) baza mare AB = 6,5 cm, AC = 4,5 cm, BC = 3,1 cm.

143
7. Construiţi trapezul isoscel ABCD cu:
a) baza mare AB = 5,2 cm, AC = 4,1 cm, mÐA = 15°;
b) baza mare AB = 5,8 cm, AC = 4,2 cm, mÐA = 25°;
c) baza mare AB = 6,5 cm, AC = 5,3 cm, mÐA = 28°;
d) baza mare AB = 6,9 cm, AC = 5,6 cm, mÐA = 30°.

8. Construiţi trapezul isoscel ABCD cu:


a) baza mare AB = 5,2 cm, mÐA = 16°, mÐB = 75°;

144
Cap 4. Patrulatere
b) baza mare AB = 5,6 cm, mÐA = 18°, mÐB = 72°;
c) baza mare AB = 6,1 cm, mÐA = 23°, mÐB = 68°.

9. Construiţi trapezul isoscel ABCD cu:


a) baza mică DC = 3,3 cm, AD = 4,2 cm, mÐB = 123°;
b) baza mică DC = 4,4 cm, AD = 4,4 cm, mÐB = 112°;
c) baza mică DC = 1,9 cm, AD = 5,7 cm, mÐB = 104°.

145
10. Construiţi trapezul isoscel ABCD cu:
a) baza mică CD = 2,5 cm, AC = 6,4 cm, AD = 4,6 cm;
b) baza mică CD = 1,7 cm, AC = 5,6 cm, AD = 4,6 cm;
c) baza mică CD = 2,3 cm, AC = 6,8 cm, AD = 4,9 cm.

11. Construiţi trapezul isoscel ABCD cu baza mare AB = 6,5 cm, BC = 3,5 cm, mÐC
= 105°. Enumeraţi etapele.

146
Cap 4. Patrulatere
12. Construiţi trapezul isoscel ABCD cu baza mare AB = 6,5 cm, înălţimea DM = 4,2
cm, mÐA = 66°. Enumeraţi etapele.

13. Construiţi trapezul isoscel ABCD cu baza mare AB = 5,8 cm, CD = 2,6 cm,
înălţimea MD = 3,1 cm. Enumeraţi etapele.

14. Construiţi trapezul isoscel ABCD cu baza mare AB = 6,5 cm, CD = 3,9 cm,
AC = 6,2 cm. Enumeraţi etapele.

147
15. Construiţi trapezul isoscel ABCD cu baza mare AB = 7,4 cm, unghiul ACB este
drept, mÐA = 67°. Enumeraţi etapele.

16. Construiţi trapezul isoscel ABCD cu diagonalele perpendiculare, baza mare AB =


6,4 cm, CD = 4 cm. Enumeraţi etapele.

17. Construiţi trapezul isoscel ABCD cu AD ^ BC, baza mare AB = 5,8 cm, înălţimea
MD = 1,9 cm. Enumeraţi etapele.

148
Cap 4. Patrulatere
15 Linia mijlocie a trapezului

l Bimediană a unui patrulater este un segment determinat de mijloacele a două laturi


opuse. Orice patrulater (convex sau concav) are două bimediane.
Teorema bimedianelor. Bimedianele unui patrulater au acelaşi mijloc.

Demonstraţie. Teorema liniei mijlocii A E


implică: MNPQ este paralelogram (are Q L
două laturi opuse paralele şi congruente); D D K
H
IJKL este paralelogram (are două laturi M I
opuse paralele şi congruente). P J
Proprietatea diagonalelor paralelogra- B C
mului implică: bimedianele patrulaterului N G
convex ABCD au acelaşi mijloc; bimedianele patrulaterului concav DEGH au acelaşi
mijloc.

Teorema bimedianelor dreptunghiului şi pătratului. Bimedianele dreptunghiului


şi pătratului sînt conţinute de mediatoarele laturilor opuse.
l Linia mijlocie a unui trapez este segmentul determinat de mijloacele laturilor ne-
paralele.
Teorema liniei mijlocii. Linia mijlocie a trapezului este paralelă cu bazele trapezu-
lui şi lungimea ei este egală cu semisuma lungimilor bazelor.

Ipoteza. Desenul, AB || DC, M mijlocul [AD], N mijlocul [BC].


Concluzia. MN || AB, MN = 0,5(AB + CD). D
C
Demonstraţie. Fie MP || DC, (1). Proprie-
tatea liniei mijlocii a triunghiului implică-
[AP] ≡ [PC], (2). PQ || AB, implică [QB] P Q N
M
≡ [QD], (3). Conform proprietăţii liniei
mijlocii a triunghiului, ipoteza şi (3) im-
plică NQ || CD, (4). Conform Postulatu-
lui Paralelelor, AB || DC, (1), (2), (3), (4) A B
implică MN || AB, (5).
Conform proprietăţii liniei mijlocii a triunghiului, (1) implică MP = 0,5 • DC, (6).
În acelaşi mod rezultă PN = 0,5 • AB, (7). (6) şi (7) implică MN = 0,5(AB + CD),
q.e.d.
Corolar. 1) Linia mijlocie a trapezului conţine mijloacele diagonalelor.
2) Lungimea segmentului determinat de mijloacele diagonalelor este egală cu
semidiferenţa lungimilor bazelor trapezului.

149
Exerciţii
1. Completaţi propoziţiile:
a) Bimediană a unui patrulater este un segment determinat de mijloacele a două

b) Orice patrulater (convex sau concav) are bimediane.


2. Construiţi bimedianele:
D
E
H
L K
A

G I J
B C
F

3. Completaţi propoziţia:
Bimedianele unui patrulater au acelaşi

4. Verificaţi dacă bimedianele au acelaşi mijloc:


L
A D E H

F G J
I
B C
5. Completaţi propoziţiile:
a) Bimedianele paralelogramului sînt cu laturile paralelogramului
şi se intersectează în de simetrie.
b) Bimedianele dreptunghiului şi pă­tra­tului sînt conţinute de

c) Linie mijlocie a unui trapez este bimediana determinată de mijloacele

6. Recunoaşteţi proprietăţile bimediane­lor paralelogramelor:


A D K
E H
L

J
F G
B C
I

7. Recunoaşteţi proprietăţile bimediane­lor dreptunghiului şi pătratului:

150
Cap 4. Patrulatere
A D L K
E H

F G
B C
I J

8. Completaţi propoziţiile:
a) Linia mijlocie a trapezului este paralelă cu bazele trapezului şi lungimea ei este
egală cu
b) Linia mijlocie a trapezului conţine mijloacele
c) Lungimea segmentului determinat de mijloacele diagonalelor este egală cu
bazelor trapezului.
9. Trapezul ABCD are linia mijlocie MN, P şi Q mijloacele diago- D C
nalelor. Aflaţi lungimea liniei mijlocii şi PQ, dacă:
a) DC = 3,5 cm, AB = 5,7 cm; b) DC = 2,7 cm, AB = 4,3 cm; P Q
M N
c) DC = 1,1 cm, AB = 3,7 cm; d) DC = 4,1 cm, AB = 5,9 cm.
A B

10. Construiţi trapezul isoscel ABCD, dacă:


a) lungimea liniei mijlocii MN = 6,5 cm, distanţa dintre mijloacele diagonalelor PQ
= 2,5 cm şi înălţimea de 4,2 cm;
b) lungimea liniei mijlocii MN =
4,2 cm, distanţa dintre mijloacele diagonalelor PQ = 1,8 cm şi înălţimea de 3 cm;
c) lungimea liniei mijlocii MN = 5,3 cm, distanţa dintre mijloacele diagonalelor PQ
= 1,7 cm şi înălţimea de 4 cm;
d) lungimea liniei mijlocii MN = 5,8 cm, distanţa dintre mijloacele diagonalelor PQ
= 2,2 cm şi înălţimea de 2,5 cm.

151
11. Trapezul isoscel ABCD cu baza mare AB are distanţa dintre mijloacele diagona-
lelor şi înălţimea de 2,3 cm. Aflaţi măsura unghiului A.

12. Trapezul dreptunghic ABCD are baza mare AB, unghiul A drept, înălţimea de 2,6 cm şi
distanţa dintre mijloacele diagonalelor de 1,3 cm. Aflaţi măsura unghiului B.

152
Cap 4. Patrulatere
13. Construiţi trapezul dreptunghic ABCD cu baza mare AB = 8,6 cm, astfel încît
triunghiul ACB este un triunghi dreptunghic isoscel cu ipotenuza AB. Aflaţi lungimea
liniei mijlocii şi distanţa dintre mijloacele diagonalelor trapezului ABCD.

14. Construiţi trapezul dreptunghic ABCD cu baza mare AB = 6,4 cm, astfel încît
triunghiul ACB este un triunghi dreptunghic isoscel cu ipotenuza AB. Aflaţi lungimea
liniei mijlocii şi distanţa dintre mijloacele diagonalelor trapezului ABCD.

15. Construiţi trapezul dreptunghic ABCD cu baza mare AB = 7,4 cm, astfel încît
triunghiul ACB este un triunghi dreptunghic isoscel cu ipotenuza AB. Aflaţi lungimea
liniei mijlocii şi distanţa dintre mijloacele diagonalelor trapezului ABCD.

16. Construiţi trapezul isoscel ABCD cu baza mare AB = 5,4 cm, laturile neparalele
conţinute de drepte perpendiculare şi înălţimea de 3 cm.

153
Evaluare
I II
1. Patrulaterul con- C 1. Patrulaterul con- C
vex ABCD are mă- D vex ABCD are mă- D
surile unghiurilor 10 surile unghiurilor
A, B, C respectiv A, B, C respectiv
egale cu 47°, 86°, A B egale cu 42°, 93°, A B
59°. Aflaţi măsura unghiului D. 71°. Aflaţi măsura unghiului D.
2. Paralelogramul DEFG are DE = 5 cm 2. Paralelogramul DEFG are DE = 4,8
şi EF = 5,7 cm. Aflaţi lungimile celor- cm şi EF = 6 cm. Aflaţi lungimile celor-
10
lalte laturi ale paralelogramului. lalte laturi ale paralelogramului.
3. Paralelogramul DEFG are unghiul D 3. Paralelogramul DEFG are unghiul D
de 41°. Aflaţi măsurile celorlalte unghiuri 10 de 52°. Aflaţi măsurile celorlalte unghiuri
ale paralelogramului. ale paralelogramului.
4. Paralelogramul DEFG are centrul de 4. Paralelogramul DEFG are centrul de
simetrie O. Aflaţi lungimile segmen- simetrie O. Aflaţi lungimile segmen-
telor OD şi OE, dacă DF = 4,8 cm şi telor OD şi OE, dacă DF = 5,2 cm şi
GE = 3,8 cm. 10
GE = 2,4 cm. G F
G F O
O
D E
D E
5. Trapezul dreptunghic JKLM are un­ 5. Trapezul dreptunghic JKLM are
ghiul K de 24°. M L un­ghiul K de 37°. M L
Aflaţi măsurile Aflaţi măsurile
celorlalte un- celorlalte un-
ghiuri ale trape- 10 ghiuri ale trape-
J K J K
zului. zului.
6. Trapezul MNPQ are bazele MN de 6. Trapezul MNPQ are bazele MN de
5,4 cm şi PQ de 3,6 cm. Aflaţi: 8,8 cm şi PQ de 6,2 cm. Aflaţi:
a) lungimea liniei mijlocii a trapezu- a) lungimea liniei mijlocii a trapezu-
lui; 10
lui;
b) distanţa dintre mijloacele diago- b) distanţa dintre mijloacele diago-
nalelor lui. nalelor lui.
7. Trapezul isos- Q P 7. Trapezul isos- Q P
cel MNPQ are cel MNPQ are
NP = 3,2 cm, NP = 2,6 cm,
MP = 4,1 cm şi MP = 4,9 cm şi
10
unghiul P de M N unghiul P M N
115°. Aflaţi: de 123°. Aflaţi:
a) MQ şi NQ; a) MQ şi NQ;
b) măsurile celorlalte unghiuri. b) măsurile celorlalte unghiuri.

154
Cap 4. Patrulatere
8. Construiţi: 8. Construiţi:
a) paralelogramul cu diagonalele a) paralelogramul cu diagonalele
de 6 cm, 4,8 cm şi un unghi format 10 de 4 cm, 5,8 cm şi un unghi format
de ele de 34°; de ele de 23°;
b) rombul cu diagonalele de 4,6 cm b) rombul cu diagonalele de 5,8 cm
şi 5,2 cm. şi 4,4 cm.
9. Construiţi trapezul isoscel ABCD cu 9. Construiţi trapezul isoscel ABCD cu
baza mare AB de 3,8 cm, unghiul A de baza mare AB de 4,5 cm, unghiul A de
10
45° şi CD = 2,1 cm. 45° şi CD = 2,6 cm.
10. Construiţi trapezul isoscel ABCD cu 10. Construiţi trapezul isoscel ABCD cu
baza mare AB de 5,6 cm, unghiul B de 10
baza mare AB de 6,4 cm, unghiul B de
60° şi diagonalele perpendiculare. Aflaţi 60° şi diagonalele perpendiculare. Aflaţi
perimetrul trapezului. perimetrul trapezului.

Exerciţii suplimentare

1. (Teorema lui Ţiţeica) Trei cercuri congruente se intersectează în punc- A

tele A, B, C, D. Demonstraţi că cercul circumscris triunghiului ABC


D
este congruent cu cele trei cercuri.
B C

2. În desen triunghiul ABC este isoscel cu baza BC. Cu ajutorul informaţiilor din
desen, aflaţi măsura unghiului ADE. C
E
20°
20°
A
30°
D
B

155
Capitolul 5 Vectori în plan
1 Noţiunea de vector
l Lungimea unui segment, măsura unui unghi, aria unui triunghi sînt mărimi carac-
terizate numai printr-un număr. De aceea, lungimea segmentului, măsura unghiului,
aria triunghiului etc. sînt mărimi scalare.
l Forţa este o mărime fizică caracterizată de mărime, direc-
F

ţie şi sens. Forţa este o mărime fizică vectorială. v
l Pe reţeaua de pătrate ∆ABC ≡ ∆DEF. Triunghiul DEF se C
obţine din triunghiul ABC prin deplasarea fiecărui punct al → D → E
v v
lui cu 3 unităţi spre dreapta şi 2 unităţi în sus. Deplasarea

poate fi definită cu ajutorul vectorului v . A B

Un vector are: – mărime (modul); – direcţie (dreapta ce


conţine vectorul sau este paralelă cu această dreaptă); →
v
– sens (indicat de săgeată).
→ → → →
u şi v sînt vectori egali, u = v , dacă au acelaşi modul,
aceeaşi direcţie şi acelaşi sens.
l Pe reţeaua de pătrate sînt reprezentaţi vectorii egali y N(x2, y2)
→ → –
OA şi u . Aplicînd teorema lui Pitagora, OA = √ a2 + b2 . A →
b u
AOMN este paralelogram. Rezultă ∆AOB ≡ ∆AOB, de

unde x2 – x1 = a, y2 – y1 = b şi MN = √ a2 + b2 .
P
→ B M(x1, y1)

Dacă OA = u , A(a, b), atunci a şi b sînt coordonatele O a x

vectorului u. Se scrie u (a, b), iar modulul vectorului u
→ –
este |u | = √ a2 + b2 .

Exerciţii
1. Completaţi propoziţiile:
a) Un vector are
b) Direcţia unui vector
→ →
c) u şi v sînt vectori egali
2. Identificaţi vectorii egali.


→ b →
a c
→ →
d e

156
Cap 5. Vectori în plan
3. Completaţi propoziţiile:
→ →
a) Dacă OA = u , A(a, b), atunci

b) u (a, b) are modulul

4. Aflaţi coordonatele OA , dacă: y
a) A(–12; 3,2); b) A(–5,2;––11,9); –
c) A(8,13; 17,4); d) A(11√ 3 ; –2,5√ 6 ). b A

O a x


5. Desenaţi vectorul OA de coordonate:
a) (4, 3); b) (5, –3); c) (–5, 4); d) (–3, –5).


6. Desenaţi u (2, –4) cu originea în punctul:
a) M(1, 2); b) M(–2, 3); c) M(3, –1); d) M(–4, –2).

157

7. Aflaţi modulul u de coordonate: a) (2,5; 3); b) (5; –12): c) (7,1; –3); d) (–3,6; 4,1).


8. Aflaţi m astfel încît modulul u de coordonate:
a) (2, m + 1) să fie egal cu 13; b) (–3, m – 2) să fie egal cu 15;
c) (4, m – 3) să fie egal cu 17; d) (6, m + 4) să fie egal cu 21.


9. Reprezentaţi într-un sistem de axe de coordonate u cu originea în:
a) A(3, –2) şi coordonatele (3, 5); b) A(2, –3) şi coordonatele (4, –2);
c) A(–2, –4) şi coordonatele (–1, 3); d) A(–6, 3) şi coordonatele (–1, –2).

10. Aflaţi x şi y astfel încît:


→ → → →
a) u (x + 3, x – 2y + 5) = v (–x – 4, 3); b) u (2y – 1, 3x – 5y + 7) = v (3y, 4);
→ → → →
c) u (5x + 2, 3x – y + 2) = v (x – 7, 1); d) u (7y + 4, 5x – y + 1) = v (4y, 12).

158
Cap 5. Vectori în plan
2 Operaţii cu vectori
Regula paralelogramului. Suma a doi vectori este de- y
terminată de diagonala cu un capăt în originea comună b+d C
b A
a vectorilor. s→
→ →
Adunarea vectorilor. Suma vectorilor u (a, b) şi v (c, d) →
u
→ → → →
este u (a, b) + v (c, d) = s (a + c, b + d). d v B
→ x
Vectorul nul este 0 (0, 0). Vectorul opus vectorului O a c
→ → → → a+c
–u (–a, –b), u + (–u ) = 0 .
→ → →
l A, B, C sînt puncte coliniare. Atunci AB + BC = AC , C
B
→ → → → → → → A
iar AC + CA = 0 . Rezultă: AB + BC + CA = 0 .
→ →
Vectorii AC şi CA sînt opuşi.
→ → → →
Dacă A, B, C sînt puncte coliniare, atunci AB + BC + CA = 0 .
→ → →
l u (a, b) + u (a, b) = 2u (2a, 2b).

Înmulţirea vectorilor cu un număr. Înmulţind cu m vectorul u (a, b) se obţine vec-

torul mu (ma, mb).

Exerciţii
1. Aflaţi suma vectorilor:
→ → → →
a) u (3, –2) şi v (–12, 8); b) u (–17, –18) şi v (12, 27);
→ → → →
c) u (56, –37) şi v (–55, 85); d) u (39, –44) şi v (–62, 72).

2. Aflaţi opusul vectorului:


→ → → →
a) u (104, –271); b) u (89, 191); c) u (–252, 33); d) u (–57, –82).

159
3. Construiţi grafic suma vectorilor:
C C

A B A B
a) b)
C C

A
B A
B
c) d)

4. Aflaţi coordonatele vectorului u cu:

a) | u | = 12, dacă formează cu Ox un unghi de 30°;

b) | u | = 15, dacă formează cu Ox un unghi de 45°;

c) | u | = 18, dacă formează cu Ox un unghi de 60°;

d) | u | = 20, dacă formează cu Ox un unghi de 120°.

5. Completaţi propoziţiile:
→ → →
a) AC + CD + DA = dacă A–C–D.
→ → →
b) BE + EF + FB = dacă B–E–F.
→ → →
c) JK + KL + LJ = dacă J–K–L.
→ → → →
6. Aflaţi: a) 3u (4, –6) + 2v (–9, 5); b) 7u (9, –12) + 3v (–15, 16).

160
Cap 5. Vectori în plan
3 Descompunerea vectorilor
y


1) u = OA , A(a, b), M = prOxA, N = prOyA. OMAN este N(0, b) A

→ →
dreptunghi, deci u = OM + ON . →
→ → → → u
2) Vectorii i şi j au modulul 1, | i | = 1 şi | j | = 1. Prin →
→ → → → → → → j
urmare, OM = a i şi ON = b j , de unde u = a i + b j .
M(a,0) x

i O
Descompunerea unui vector după direcţiile axelor de coordonate. Versorii axelor
→ → → → →

de coordonate sînt i şi j , | i | = 1 şi | j | = 1. Dacă u = OA , A(a, b), M = prOx A, N = prOy

→ → → →
A, atunci u = OM + ON = a i + b j .
Descompunerea unui vector după două direcţii oarecare. D C
Prin C se construiesc paralelele la cele două direcţii AB şi AD. →

v
s
Se obţine paralelogramul ABCD şi descompunerea vectorului →
t
→ →
→ →
v , v = AB + AD . A B

Planul înclinat. Greutatea (G ) unui corp aflat
pe un plan înclinat se descompune în două for-
ţe (v. dreptunghiul OMNP). Conform criteriu-
lui CC, triunghiurile dreptunghice OMN şi CAB
sînt asemenea.
    
G= | G | sin α ,
Gt + Gn ,| Gt | =

    AB   AC
| G | cos α ,| Gt | =
| Gn | = |G |⋅ , | Gt | =
|G |⋅ .
BC BC

Exerciţii
1. Completaţi propoziţiile:
a) Versorii axelor de coordonate sînt

→ →
b) Dacă u = OA , A(a, b), M = prOx A, N = prOy A, atunci u =

2. Scrieţi u conform descompunerii lui pe axele de coordonate, dacă:

161
→ → → →
a) u (6, –9); b) u (–3, 17); c) u (–2, –18); d) u (13, –27).


3. Descompuneţi OA după direcţiile axelor:
y y y y
A A
A
A

O x O x O x O x


4. Descompuneţi vectorul v după direc­ţiile date:

→ → →
v v → v
v

5. Reprezentaţi vectorii:
           
a) u = 2i + 5 j ; c) u =
3i + 2 j ; b) u = 3i + 4 j ; d) u =
5i + 2 j .

     
6. Reprezentaţi vectorii: a) u =−2i + 7 j ; b) u =3i − 5 j ;
           
a) u =−2i + 7 j ; b) u =3i − 5 j ; c) u =−4i + 5 j ; d) u =6i − 7 j .
     
c) u =−4i + 5 j ; d) u =6i − 7 j .

162
Cap 5. Vectori în plan
     
7. Reprezentaţi vectorii: a) u =−5i − 6 j ; b) u =−2i − 7 j ;
           
a) u =−5i − 6 j ; b) u =−2i − 7 j ; c) u =−3i − 8 j ; d) u =−4i − 8 j .
     
c) u =−3i − 8 j ; d) u =−4i − 8 j .

8. Calculaţi modulele componentelor gre­u­tăţii unui corp aşezat pe un plan înclinat,


dacă:
a) unghiul planului înclinat este de 30° şi greutatea este de 78 N;
b) unghiul planului înclinat este de 45° şi greutatea este de 94 N;
c) unghiul planului înclinat este de 60° şi greutatea este de 118 N.

9. Aflaţi modulele vectorilor în care se descompune un vector cu modulul:


a) de 18 cm şi care formează cu axa Ox un unghi de 30°;
b) de 48 cm şi care formează cu axa Ox un unghi de 45°;
c) de 56 cm şi care formează cu axa Ox un unghi de 60°.

163
10. Aflaţi modulele vectorilor în care se descompune un vector cu modulul:
a) de 64 cm şi care formează cu axa Ox un unghi de 150°;
b) de 72 cm şi care formează cu axa Ox un unghi de 120°;
c) de 84 cm şi care formează cu axa Ox un unghi de 135°.

     
a) u cu
11. Aflaţi tangenta unghiului format de un vector = 9i +Ox,
−axa j ; b) u =
13 dacă: 92i − 18 j ;
           
a) u = −9i + 13 j ; b) u =92i − 18 j ; c) u = 12i − 15 j ; d) u = −35i − 15 j .
     
c) u = 12i − 15 j ; d) u = −35i − 15 j .

164
Cap 5. Vectori în plan
12. Aflaţi tangenta unghiului format de vectorul OA cu axa Ox, dacă:
a) B(0, 6) este proiecţia punctului A pe Ox şi OA = 15;
b) B(0, 12) este proiecţia punctului A pe Ox şi OA = 48;
c) B(0, –14) este proiecţia punctului A pe Ox şi OA = 36;
d) B(0, –17) este proiecţia punctului A pe Ox şi OA = 63.

4 Produsul scalar

→ → → → → →
Produsul scalar al vectorilor u = a i + b j şi v = c i + d j . Dacă măsura unghiului
→ →
format de vectori este notat (u , v ), atunci produsul lor scalar este:
→ → → → → →
u v = |u ||v | cos (u , v ).
Vectorii ortogonali formează un unghi drept.
Versorii axelor de coordonate sînt ortogonali, cos 0° = 1 şi cos 90° = 0 implică:
→ → → → → → → →
i i = 1, j j = 1, i j = 0 şi j i = 0.

→ → → → → →
1) Produsul scalar al vectorilor u = a i + b j şi v = c i + d j este:
→ → → → → → → → → → → → → →
u v = (a i + b j )(c i + d j ) = ac i i + ad i j + bc j i + bd j j = ac + bd.
→ → → → → → → → → →
2) Fie u = a i + b j şi v = c i + d j . Atunci ac + bd = |u ||v | cos (u , v ), de unde
– → –
cos (u , v ) = ac→+→bd , u = √ a2 + b2 , v = √ c2 + d 2 , cos (u , v ) = – ac + bd
→ → → → →
–.
|u ||v | √ a + b2 √ c2 + d 2
2

→ → → → → → → → → →
Dacă u = a i + b j şi v = c i + d j , atunci ac + bd = |u ||v | cos (u , v ), de unde
– → –
cos (u , v ) = ac→+→bd , u = √ a2 + b2 , v = √ c2 + d 2 , cos (u , v ) = – ac + bd
→ → → → →
–.
|u ||v | √ a + b2 √ c2 + d 2
2

165
Exerciţii
1. Completaţi propoziţiile:
a) Dacă unghiul format de vectori este notat atunci produsul lor scalar este

b) Vectorii ortogonali formează

c) cos 0° = şi cos 90° =


→ → → → → → → →
d) i i = j j = i j = şi j i =
→ → → → → →
6. Scrieţi u v , dacă: a) |u | = 16 şi |v | = 9; b) |u | = 13 şi |v | = 15;
→ → → →
c) |u | = 19 şi |v | = 21; d) |u | = 23 şi |v | = 24.

→ →
7. Calculaţi u v , dacă:
→ → → → → → → →
a) |u | = 4, |v | = 5 şi (u , v ) = 60°; b) |u | = 12, |v | = 8 şi (u , v ) = 30°;
→ → → → → → → →
c) |u | = 11, |v | = 9 şi (u , v ) = 45°; d) |u | = 13, |v | = 11 şi (u , v ) = 120°.

→ → → → → →
8. Calculaţi u v , dacă: a) u (15, 8) şi v (−7, 18); b) u (−17, −3) şi v (6, 15);
→ → → →
c) u (−8, 13) şi v (11, −21); d) u (−12, 15) şi v (−3, 18).

166
Cap 5. Vectori în plan

9. Aflaţi cosinusul unghiului format de vectorul u cu axa Ox, dacă:
→ → → →
a) u (−5, 18); b) u (−3, −23); c) u (17, −16); d) u (−21, −3).


10. Aflaţi cosinusul unghiului format de vectorul u cu axa Oy, dacă:
→ → → →
a) u (−2, 28); b) u (−11, −19); c) u (22, −35); d) u (−38, −2).

→ →
11. Aflaţi cosinusul unghiului format de vectorul u şi v , dacă:
→ → → →
a) u (11, 6) şi v (−28, 3); b) u (−8, −7) şi v (15, 12);
→ → → →
c) u (−30, 21) şi v (40, −9); d) u (−33, 5) şi v (−9, 31).

12. Aflaţi măsurile unghiurilor triunghiu­lui ABC, dacă:


a) A(−2, 5), B(7, −5), C(5, 9); b) A(−3, 2), B(9, −6), C(2, 11);
c) A(−7, 1), B(11, −2), C(9, 3); d) A(−15, 2), B(9, −1), C(12, 5).

167
→ →
13. Stabiliţi dacă u şi v sînt vectori ortogonali:
→ → → →
a) u (5, 12) şi v (−12, 5); b) u (13, −2) şi v (−4, −26);
→ → → →
c) u (15, 2) şi v (−4, 30); d) u (20, 60) şi v (−9, 3).

→ →
14. Aflaţi m astfel încît u v = p, dacă:
→ → → →
a) u (m − 2, m), v (m + 2, 2) şi p = 16; b) u (m − 1, m), v (m − 1, 5) şi p = 7;
→ → → →
c) u (m − 3, m), v (m − 3, 4) şi p = 21; d) u (m − 5, m), v (m + 5, 6) şi p = 22.

→ →
15. Aflaţi m astfel încît (u , v ) = α, dacă:
→ → → →
a) u (6, 1), v (m, 2) şi α = 30°; b) u (2, −m), v (−1, 3) şi α = 45°;
→ → → →
c) u (5, 2m), v (−2, 1) şi α = 60°; d) u (3m, −4), v (1, 2) şi α = 150°.

168
Cap 5. Vectori în plan
16. Aflaţi x şi y astfel încît:
→ → → → → → →
a) a (x − 3, 5), u (2x + 1, −3), v (7, y + 2), a v = 1 şi u v = −5;
→ → → → → → →
b) a (x − 5, −7), u (x + 2, −4), v (3, y − 4), a v = 5 şi u v = −1;
→ → → → → → →
c) a (3x − 2, 6), u (x + 5, −9), v (−2, y − 4), a v = −11 şi u v = −8;
→ → → → → → →
d) a (6x − 1, −2), u (3x − 2, −8), v (4, y + 1), a v = −2 şi u v = 0.

Exerciţii recapitulative

1. Desenaţi vectorul OA de coordonate:
a) (2, 8); b) (7, −3); c) (−2, 9); d) (−4, −2).

169

2. Desenaţi u (3, −2) cu originea în punctul:
a) M(3, 1); b) M(−4, 5); c) M(7, −3); d) M(−2, −8).


3. Aflaţi modulul u de coordonate:
– – – – –
a) (3√ 7 , −8); b) (7√ 2 , −3); c) (7√ 3 , −√ 8 ); d) (−6√ 6 , −5).


4. Aflaţi m astfel încît modulul u de coordonate:
a) (3, m − 1) să fie 22; b) (−2, m + 3) să fie 19;
c) (5, m − 2) să fie 25; d) (3, m + 5) să fie 27.

170
Cap 5. Vectori în plan

5. Reprezentaţi într-un sistem de axe de coordonate u cu originea în:
a) A(3, −2) şi coordonatele (3, 5); b) A(2, −3) şi coordonatele (4, −2);
c) A(−2, −4) şi coordonatele (−1, 3); d) A(−6, 3) şi coordonatele (−1, −2).

6. Aflaţi x şi y astfel încît:


→ → → →
a) u (7x + 1, 2x − y + 3) = v (x − 5, 9); b) u (9y − 2, 9x − 2y + 1) = v (4y, 3);
→ → → →
c) u (8x + 3, 7x − 3y + 1) = v (x − 3, 7); d) u (2y + 9, 8x − 2y + 3) = v (6y, 11).

→ → → →
7. Aflaţi suma vectorilor: a) u (8, −11) şi v (−3, 13); b) u (−21, −6) şi v (51, 37);
→ → → →
c) u (93, −56) şi v (−27, 77); d) u (28, −25) şi v (−16, 41).

171
8. Aflaţi opusul vectorului:
→ → → →
a) u (54, −401); b) u (31, −59); c) u (−71, 99); d) u (−38, −75).

9. Construiţi grafic suma vectorilor:


C A C
C
A
C
B B B
A A B
a) b) c) d)

10. Aflaţi coordonatele vectorului u cu:

a) | u | = 28, dacă formează cu Ox un unghi de 30°;

b) | u | = 34, dacă formează cu Ox un unghi de 45°;

c) | u | = 36, dacă formează cu Ox un unghi de 60°;

d) | u | = 46, dacă formează cu Ox un unghi de 150°.

11. Completaţi propoziţiile:


→ → →
a) AN + NB + BA = dacă A−N−B.
→ → →
b) CE + ED + DC = dacă C−E−D.
→ → → →
12. Aflaţi: a) 5u (6, −3) − 7v (−2, 4); b) 4u (6, −8) − 6v (−9, 13).

172
Cap 5. Vectori în plan

13. Scrieţi u conform descompunerii lui pe axele de coordonate, dacă:
→ → → →
a) u (11, −7); b) u (−9, 44); c) u (−5, −72); d) u (19, −51).


14. Descompuneţi OA după direcţiile axelor:
y y
A
A

O x O x

15. Descompuneţi vectorul v după direcţiile date:


v
v→

→ →
16. Scrieţi u v , dacă:
→ → → → → →
a) |u | = 15 şi |v | = 18; b) |u | = 26 şi |v | = 25; c) |u | = 31 şi |v | = 13.

173
→ → → → → →
17. Calculaţi u v , dacă: a) |u | = 13, |v | = 15 şi (u , v ) = 120°;
→ → → → → → → →
b) |u | = 12, |v | = 21 şi (u , v ) = 135°; c) |u | = 18, |v | = 26 şi (u , v ) = 150°.

→ → → →
18. Calculaţi u v , dacă: a) u (35, −8) şi v (−19, 29);
→ → → →
b) u (−41, −9) şi v (16, 21); c) u (−28, 15) şi v (34, −35).

→ →
19. Aflaţi cosinusul unghiului format de vectorul u şi v , dacă:
→ → → → → →
a) u (21, 2) şi v (−3, 49); b) u (−71, −2) şi v (4, 24); c) u (−30, 1) şi v (2, −120).

20. Aflaţi cosinusurile unghiurilor triunghiului ABC, dacă: a) A(−5, 2), B(9, −3),

174
Cap 5. Vectori în plan
C(10, 12); b) A(−4, 6), B(2, −11), C(15, 8); c) A(−14, 6), B(13, −8), C(18, 9).

Evaluare
→ →
1. Desenaţi vectorul OA de coordonate: 1. Desenaţi vectorul OA de coordonate:
a) (3, 4); b) (−5, −8). 20 a) (4, 3); b) (−8, −5).
→ →
2. Desenaţi u (3, −2) cu originea în punc- 2. Desenaţi u (−3, 2) cu originea în punc-
tul: 10 tul:
a) M(2, 4); b) M(−1, 3). a) M(4, 2); b) M(3, −1).
→ →
3. Aflaţi modulul

u de coordonate:

3. Aflaţi modulul

u de coordonate:

a) (4√ 3 , −5); b) (6√ 5 , −7). 10 a) (−5√ 3 , 4); b) (7√ 5 , −9).
4. Reprezentaţi într-un sistem de axe de 4. Reprezentaţi într-un sistem de axe de
→ →
coordonate u cu originea în: A(2, −3) şi 10 coordonate u cu originea în: A(3, −2) şi
coordonatele (4, 7). coordonatele (7, 4).
5. Aflaţi vectorul: 5. Aflaţi vectorul:
→ → → →
a) u (5, −16) + v (−15, 21); 10 a) u (17, −6) + v (−12, 32);
→ → → →
b) 2u (−14, −2) + v (19, 8). b) u (−8, 19) + 2v (−18, 3).
6. Construiţi grafic suma vectorilor: 6. Construiţi grafic suma vectorilor:
C A
C
B 10

B
A

7. Descompuneţi vectorul
u
7. Descompuneţi vectorul →
u


u după direcţiile date: 10 u după direcţiile date:
→ →
8. Calculaţi u v , dacă:
→ →
8. Calculaţi u v , dacă:
→ → → → → → → →
a) |u | = 9, |v | = 24 şi (u , v ) = 150°; a) |u | = 8, |v | = 27 şi (u , v ) = 150°;
→ → → →
b) u (11, −3) şi v (−14, 8). 10
b) u (14, −9) şi v (−12, 7).
9. Aflaţi m, dacă: 9. Aflaţi m, dacă:
→ → → → → → → →
u (3, −2), v (−2, m) şi (u , v ) = 60°. 10 u (2, −3), v (−3, m) şi (u , v ) = 60°.

175
Cuprins
Capitolul 1. Recapitulare şi completări................... 2 Exerciţii............................................................. 69
1. Elemente de logică. Definiţia, axioma, 6. Teorema lui Pitagora..................................... 71
teorema.............................................................. 2 Exerciţii............................................................. 72
Exerciţii............................................................. 3 7. Reciproca teoremei lui Pitagora.................... 75
2. Contraexemplu, teorema reciprocă, Exerciţii............................................................. 76
teorema de echivalenţă...................................... 5 Exerciţii recapitulative...................................... 78
Exerciţii............................................................. 7 Evaluare............................................................. 84
3. Congruenţa triunghiurilor. Paralelism........... 9 8. Elemente de trigonometrie............................ 84
Exerciţii............................................................. 10 Exerciţii............................................................. 85
4. Proprietăţi ale triunghiurilor.......................... 12 9. Valorile funcţiilor trigonometrice.................. 87
Exerciţii............................................................. 13 Exerciţii............................................................. 88
5, Cerc. Disc circular......................................... 16 Exerciţii recapitulative...................................... 90
Exerciţii............................................................. 16 Evaluare............................................................. 84
6. Poziţiile unei drepte faţă de un cerc.............. 17 Exerciţii suplimentare....................................... 84
Exerciţii............................................................. 18 Capitolul 4. Patrulatere............................................98
7. Poziţiile relative a două cercuri..................... 19 1. Poligoane........................................................98
Exerciţii............................................................. 20 Exerciţii............................................................. 99
8. Unghiuri........................................................ 21 2. Patrulatere..................................................... 100
Exerciţii............................................................. 22 Exerciţii............................................................ 101
9. Contraexenplu. Reciproca unei teoreme....... 22 3. Paralelogramul............................................. 103
Exerciţii............................................................. 23 Exerciţii............................................................ 104
Exerciţii recapitulative...................................... 24 4. Unghiurile paralelogramului........................ 105
Evaluare............................................................. 25 Exerciţii............................................................ 106
Capitolul 2. Asemănarea triunghiurilor.................. 26 5. Diagonalele paralelogramului...................... 109
1. Paralele echidistante...................................... 26 Exerciţii............................................................ 110
Exerciţii............................................................. 27 6. Dreptunghiul................................................. 113
2. Teorema paralelelor echidistante................... 29 Exerciţii............................................................ 114
Exerciţii............................................................. 30 7. Patrulatere ortodiagonale.............................. 119
3. Construcţii geometrice.................................. 32 Exerciţii............................................................ 120
Exerciţii............................................................. 33 8. Diagonalele rombului................................... 121
4. Teorema lui Thales........................................ 36 Exerciţii............................................................ 122
Exerciţii............................................................. 37 9. Înălţimile rombului....................................... 128
5. Reciproca teoremei lui Thales....................... 39 10. Construcţii.................................................. 129
Exerciţii............................................................. 40 Exerciţii............................................................ 130
6. Construcţii geometrice.................................. 43 11. Pătratul........................................................ 131
Exerciţii............................................................. 44 Exerciţii............................................................ 132
7. Asemănarea triunghiurilor............................. 45 12. Trapezul...................................................... 134
Exerciţii............................................................. 46 13. Trapezul isoscel.......................................... 137
8. Criterii de asemănare a triunghiurilor........... 48 14. Trapezul isoscel. Proprietăţi....................... 141
Exerciţii............................................................. 49 Exerciţii............................................................ 142
Exerciţii recapitulative...................................... 52 15. Linia mijlocie a trapezului.......................... 149
Evaluare............................................................. 55 Exerciţii............................................................ 150
Capitolul 3. Triunghiul dreptunghic. Relaţii............ 56 16. Linia mijlocie a trapezului.......................... 149
1. Proiecţii ortogonale....................................... 56 Exerciţii............................................................ 150
Exerciţii............................................................. 57 Evaluare............................................................ 154
2. Teorema înălţimii triunghiului dreptunghic..... 58 Exerciţii suplimentare...................................... 155
Exerciţii............................................................. 59 Capitolul 5. Vectori în plan..................................... 156
3. Reciproca teoremei înălţimii........................... 62 1. Noţiunea de vector......................................... 156
Exerciţii............................................................. 63 2.Operaţii cu vectori......................................... 159
4.Teorema catetei.............................................. 65 3. Produsul scalar............................................. 165
Exerciţii............................................................. 66 Exerciţii recapitulative..................................... 169
5. Reciproca teoremei catetei............................ 68 Evaluare............................................................ 175

S-ar putea să vă placă și